You are on page 1of 98

CHUYÊN ĐỀ

TOÁN HỌC
Tập san kỉ niệm 25 năm thành lập trường PTNK

SỐ 112
2 1
202

2
1
4 3
5
7 6
8 -
0 9
. ><
=
ĐẠI HỌC QUỐC GIA TP.HCM
TRƯỜNG PHỔ THÔNG NĂNG KHIẾU

Chuyên đề
Toán học
Số 12

TP.HCM – 2021
Mục lục

Ban biên tập


Lời nói đầu . . . . . . . . . . . . . . . . . . . . . . . . . . . . . . . . . . . . . . . . . 4

Trần Nam Dũng


Tâm tỷ cự và các định lý kinh điển của hình học phẳng . . . . . . . . . . . . . . . . . 5

Nguyễn Quang Minh


Tổng quát hóa tập tránh tỉ số và tập tránh tuyến tính . . . . . . . . . . . . . . . . . . 14

Trần Nguyễn Nam Hưng


Về một ý tưởng quy nạp theo số chữ số . . . . . . . . . . . . . . . . . . . . . . . . . 23

Trần Nguyễn Thanh Danh


Tổng quát hoá các bài hình bằng phép chia tỉ lệ . . . . . . . . . . . . . . . . . . . . . 31

Nhóm OMO PTNK


Giới thiệu kỳ thi OMO . . . . . . . . . . . . . . . . . . . . . . . . . . . . . . . . . . . 39

Ban biên tập


Lời giải đề thi chọn đội dự tuyển các năm . . . . . . . . . . . . . . . . . . . . . . . . 54

Ban biên tập


Lời giải đề thi chọn đội tuyển các năm . . . . . . . . . . . . . . . . . . . . . . . . . . 72

3
LỜI NÓI ĐẦU

Năm 2021, trường Phổ thông Năng Khiếu sẽ đón sinh nhật lần thứ 25 của mình. Đó là
dịp để thầy và trò Năng Khiếu nhìn lại chặng đường một phần tư thế kỷ đầy gian nan,
vất vả nhưng cũng nhiều vinh quang mà mình đã đi qua.

Chuyên đề Toán học cũng không nằm ngoài dòng chảy đó. Ngay từ những ngày đầu tiên
trường PTNK được thành lập, Chuyên đề Toán học đã trở thành một truyền thống không
thể thiếu của khối Toán. Suốt một phần tư thế kỷ đó, chuyên đề đã được vun đắp, gìn
giữ, kế thừa và phát huy từ thế hệ này sang thế hệ khác. Được ra mắt vào đúng dịp kỉ
niệm 25 năm thành lập trường PTNK, Chuyên đề Toán học Số 12 sẽ là nơi ghi chép lại
những nét đặc trưng nhất của học sinh chuyên Toán trường PTNK: đam mê, sáng tạo và
bền bỉ.

Ý tưởng cho Chuyên đề Toán học Số 12 đã được vun vén từ đầu năm 2019, ngay sau khi
Chuyên đề Toán học Số 11 được ra mắt. Tuy nhiên, vì sự non trẻ về chuyên môn lẫn kinh
nghiệm, đến nửa sau năm 2020, những bài viết đầu tiên mới được hoàn thành, và đến
tháng 4/2021, đội ngũ ban biên tập mới được thành lập. Đó cũng là lúc Chuyên đề Toán
học Số 12 tìm được hướng đi cho mình. Xuất phát từ mong muốn chia sẻ ý tưởng cho
một hoặc một vài bài toán tâm đắc, các bài viết trong chuyên đề đã được trau chuốt dần
để có thể truyền tải được vẻ đẹp thực sự của toán học, thoát ra được khỏi phạm vi các
kỳ thi Olympic. Ở mỗi bài viết, tác giả luôn cố gắng mở rộng, "thay áo mới" cho vấn đề
đặt ra theo những hướng bất ngờ, vì thế nên dù chủ đề có thể cũ và quen thuộc, nhưng
nội dung của bài viết thì luôn mới mẻ, thú vị và đậm chất Năng Khiếu.

Cuộc sống không phải lúc nào cũng bằng phẳng, và chặng đường để hoàn thành chuyên
đề này cũng như vậy. Nếu không có sự giúp đỡ của các thầy, các anh cựu học sinh, Chuyên
đề Toán học Số 12 sẽ không thể được hoàn tất theo đúng kế hoạch được đặt ra. Ban biên
tập xin chân thành cảm ơn các thầy Trần Nam Dũng, Nguyễn Tăng Vũ, các anh Nguyễn
Nguyễn, Nguyễn Tiến Hoàng đã đưa ra những nhận xét, lời khuyên hết sức chân thành,
thấu đáo. Cảm ơn bạn Bùi Mai Bảo Ngọc (học sinh lớp chuyên Anh niên khóa 2019 –
2022, thành viên ban Design của PTNK Photography Club) đã thiết kế nên mẫu bìa rất
vui tươi, dễ nhìn.

Trong quá trình viết bài và biên tập, sai sót là điều khó có thể tránh khỏi. Vì thế, ban
biên tập sẵn sàng đón nhận mọi ý kiến đóng góp từ bạn đọc. Hy vọng rằng bạn đọc vẫn
sẽ ủng hộ Chuyên đề Toán học trong thời gian tới, và thật sự thưởng thức được vẻ đẹp
của toán học, tìm thấy niềm say mê đối với môn toán thông qua những bài viết trong
chuyên đề.

Ban biên tập.

4
Tâm tỷ cự và các định lý kinh điển của hình học phẳng
Trần Nam Dũng
Giới thiệu. Trong bài viết này, chúng ta sẽ đến với những ứng dụng của tâm
tỷ cự trong việc chứng minh các định lý kinh điển của hình học phẳng, bao gồm
định lý Euler về tam giác pedal, công thức Euler, định lý Feuerbach và tiếp đó
là các mở rộng sang tứ giác lưỡng tâm cùng một số bài toán quỹ tích liên quan
đến định lý Poncelet.

1. Một số định lý cơ bản


Trước khi đến với một số định lý cơ bản, hãy cùng điểm qua các khái niệm về tâm tỷ cự.
Định nghĩa 1. Cho n điểm A1 , A2 , . . . , An và n số thực α1 , α2 , . . . , αn có tổng khác 0.
−−→ −−→ −−→ →

Khi đó tồn tại duy nhất điểm G sao cho α1 GA1 + α2 GA2 + · · · + αn GAn = 0 . Điểm
G được gọi là tâm tỉ cự của hệ điểm A1 , A2 , . . . , An với bộ trọng lượng α1 , α2 , . . . , αn
tương ứng.
Định nghĩa 2. Xét hệ điểm A1 , A2 , . . . , An với bộ trọng lượng α1 , α2 , . . . , αn tương ứng,
n
có tâm tỷ cự là G. Với điểm P trong mặt phẳng, đại lượng M (P ) = αi M A2i được gọi
P
i=1
là momen quán tính của điểm P đối với hệ điểm.
Từ hai định nghĩa nêu trên, ta có
n n −→ −−→2 n n n
−→ X −−→ X
M (P ) = αi P A2i = αi P G + GAi = αi P G2 + 2P G αi GAi + αi GA2i
X X X

i=1 i=1 i=1 i=1 i=1


= αP G + M (G).
2

trong đó α = α1 + α2 + · · · + αn .
Công thức này được gọi là định lý Lagrange và sẽ được ứng dụng để tính khoảng cách
giữa các điểm đặc biệt trong tam giác thông qua việc tính các momen quán tính.
Để tính momen quán tính của điểm G, ta có dùng công thức sau
1X
M (G) = αi αj Ai A2j .
α i<j

Ví dụ trường hợp n = 2, với hai điểm A1 , A2 với bộ trọng lượng α1 , α2 ta có

α1 P A21 + α2 P A22 = (α1 + α2 )P G2 + α1 GA21 + α2 GA22


α1 α2
= (α1 + α2 )P G2 + A1 A22 .
α1 + α2
Từ đó suy ra công thức
α1 α2 α1 α2
P G2 = P A21 + P A22 − A1 A22 ,
α1 + α2 α1 + α2 (α1 + α2 )2

được gọi là định lý Stewart, rất hữu ích trong việc tính khoảng cách từ một đỉnh của
tam giác đến một điểm nằm trên cạnh đáy khi biết vị trí của điểm này theo tỷ lệ chia.

5
Tâm tỷ cự và các định lý kinh điển của hình học phẳng

Ví dụ với trung điểm thì ta lấy α1 = α2 = 1 và ta có công thức tính độ dài đường trung
tuyến quen thuộc.
Với n = 3, xét tam giác ABC. Ta biết trọng tâm G là tâm tỷ cự của hệ điểm A, B, C với
bộ trọng lượng (1, 1, 1). Với một điểm M bất kỳ, ta có kết quả đẹp đẽ sau:
−−→ −−→ −−→ → −
SM BC · M A + SM CA · M B + SM AB · M C = 0 .
trong đó SM BC ký hiệu diện tích tam giác M BC. Nếu M nằm trong tam giác thì ta có
thể hiểu diện tích theo nghĩa thông thường. Trong trường hợp ngược lại, ta hiểu diện tích
theo nghĩa diện tích có dấu.
Chứng minh. Để chứng minh định lý này (xét trường hợp M nằm trong tam giác), ta
nối dài AM cắt BC tại D. Ta có
−−→ −−→ −−→ −−→ −−→ −−→
SM CA · M B + SM AB · M C = SM CA (M D + DB) + SM AB (M D + DC)
−−→ −−→ −−→
= (SM CA + SM AB )M D + SM CA · DB + SM AB · DC.
Kẻ các đường cao BH, CK xuống AM thì ta thấy
SM CA CK CD
= = .
SM AB BH BD
−−→ −−→
Từ đó suy ra SM CA · BD = SM AB · CD. Như thế hai véc-tơ SM CA · DB, SM AB · DC cùng


phương, có độ lớn bằng nhau và ngược hướng do đó có tổng bằng 0 .
Từ đây suy ra
−−→ −−→ −−→ −−→ −−→
SM BC · M A + SM CA · M B + SM AB · M C = SM BC · M A + (SM CA + SM AB ) · M D.
−−→ −−→ −−→ −−→
Do đó tổng SM BC · M A + SM CA · M B + SM AB · M C là một véc-tơ song song với M A.
−−→ −−→
Hoàn toàn tương tự ta suy ra tổng này là một véc-tơ song song với M B và M C. Điều
này chỉ có thể xảy ra khi
−−→ −−→ −−→ → −
SM BC · M A + SM CA · M B + SM AB · M C = 0 .

Định lý trên giúp chúng ta dễ dàng tìm "tọa độ tỷ cự" (tức là bộ trọng lượng tương ứng
biến nó thành tâm tỷ cự của các đỉnh) của các điểm đặc biệt trong tam giác đối với các
đỉnh của tam giác này, chẳng hạn tâm đường tròn nội tiếp I có tọa độ tỷ cự là (a, b, c),
tâm đường tròn bàng tiếp góc A có tọa độ tỷ cự là (−a, b, c), trực tâm có tọa độ tỷ cự là
(tan A, tan B, tan C), tâm đường tròn ngoại tiếp có tọa độ tỷ cự là (sin 2A, sin 2B, sin 2C).

2. Định lý Euler về tam giác pedal


Định nghĩa 3. Cho tam giác ABC, M là một điểm bất kỳ trong mặt phẳng tam giác và
D, E, F lần lượt là hình chiếu của M lên BC, CA, AB. Khi này, DEF được gọi là tam
giác pedal của điểm M đối với tam giác ABC.
Euler đã phát hiện ra rằng diện tích của tam giác pedal chỉ phụ thuộc vào khoảng cách
từ M đến tâm đường tròn ngoại tiếp tam giác ABC. Cụ thể ta có công thức
1

M O2
SDEF = 1 − SABC .
4 R2
Có nhiều cách chứng minh định lý này, chẳng hạn dùng phương tích hay dùng số phức.
Ở đây chúng ta sẽ trình bày cách chứng minh sử dụng tâm tỷ cự.

6
Chuyên đề Toán học Số 12

B1

E
C1
F
O
M

C
B D

A1

Chứng minh. Ta xét trường hợp M nằm bên trong tam giác ABC, các trường hợp khác
được chứng minh một cách tương tự.
Gọi A1 , B1 , C1 là các điểm đối xứng của M qua BC, CA, AB tương ứng. Khi đó các tam
giác M BC, M CA, M AB sẽ được "lật" ra bên ngoài, tạo thành lục giác AB1 CA1 BC1 có
diện tích gấp đôi diện tích tam giác ABC.
Mặt khác, lục giác này lại được tạo thành từ tam giác A1 B1 C1 và các tam giác cân
AB1 C1 , BC1 A1 , CA1 B1 . Tam giác A1 B1 C1 là ảnh của DEF qua phép vị tự tâm M tỷ số
2 do đó có diện tích bằng 4 lần diện tích tam giác DEF . Tam giác cân AB1 C1 có diện
tích bằng
1 1
AB1 · AC1 · sin(∠B1 AC1 ) = AM 2 · sin 2A.
2 2
Tương tự, diện tích tam giác BC1 A1 bằng 2 BM · sin 2B, diện tích tam giác CA1 B1 bằng
1 2
1
2
CM 2 · sin 2C. So sánh hai cách tính diện tích lục giác AB1 CA1 BC1 như trên, ta suy ra
1
2SABC = 4SDEF + (M A2 sin 2A + M B 2 sin 2B + M C 2 sin 2C).
2
Áp dụng định lý Lagrange vào tam giác ABC với hệ trọng lượng (sin 2A, sin 2B, sin 2C)
có O là tâm tỷ cự, ta có
M A2 sin 2A + M B 2 sin 2B + M C 2 sin 2C = (sin 2A + sin 2B + sin 2C)(M O2 + R2 )
M O2 M O2
! !
= 4 sin A sin B sin C(M O + R ) = ab sin C 1 +
2 2
= 2SABC 1+ .
R2 R2
Từ đó suy ra
M O2
!
4SDEF = SABC 1− .
R2
Tức là
1 M O2
!
SDEF = 1− SABC .
4 R2

Định lý Euler về tam giác pedal có một số hệ quả thú vị, trong đó có tính chất về đường
thẳng Simson: Ba điểm D, E, F thẳng hàng khi và chỉ khi M nằm trên đường tròn ngoại
tiếp tham giác ABC.

7
Tâm tỷ cự và các định lý kinh điển của hình học phẳng

3. Định lý Feuerbach
Đường tròn 9 điểm Euler có một tính chất độc đáo mà ít tài liệu chứng minh, đó là nó
tiếp xúc trong với đường tròn nội tiếp tam giác và tiếp xúc ngoài với các đường tròn bàng
tiếp. Để chứng minh kết quả đẹp đẽ này, ta cần đến công thức tính khoảng cách giữa các
điểm đặc biệt trong tam giác: I, O, G.

I O
G
H E

C
B

Bước 1. Tính IO.


Khoảng cách IO có thể tính dựa vào tọa độ tỷ cự của I. Thật vậy
1
(a + b + c)OI 2 = aOA2 + bOB 2 + cOC 2 − (bcBC 2 + caCA2 + abAB 2 ).
a+b+c
Hay
(a + b + c)OI 2 = (a + b + c)R2 − abc.
Từ đó
abc 2R sin Abc 4RS
OI 2 = R2 − = R2 − = R2 − = R2 − 2Rr.
a+b+c a+b+c a+b+c
Đây chính là công thức Euler nổi tiếng.
Bước 2. Tính OG, IG.
Khoảng cách OG tính đơn giản dựa vào trọng tâm G.
1
3OG2 = OA2 + OB 2 + OC 2 − (1 · 1 · BC 2 + 1 · 1 · CA2 + 1 · 1 · AB 2 ).
1+1+1
Suy ra
a2 + b2 + c2
OG2 = R2 − .
9
Tương tự như vậy
2(a2 + b2 + c2 ) p2
IG2 = r2 + − .
9 3
Bước 3. Tính IE.

8
Chuyên đề Toán học Số 12

Bây giờ ta đã đủ "chất liệu" để tính độ dài IE, trong đó E là tâm đường tròn Euler. Ta
biết đường tròn Euler là ảnh của đường tròn O qua phép vị tự tâm H tỷ số 12 , do đó E
−→ −−→
là trung điểm HO. Nhưng ta cũng biết O, G, H thẳng hàng và OG = 13 OH. Từ đấy thì
−−→ −−→ → −
EO − 3EG = 0 . Áp dụng công thức Stewart, ta có
α1 α2 α1 α2
P G2 = P A21 + P A22 − A1 A22
α1 + α2 α1 + α2 (α1 + α2 )2

1 −3 1 · (−3)
IE 2 = IO2 + IG2 − OG2
1−3 1−3 (1 − 3)2
1 2 3 2 2(a2 + b2 + c2 ) p2 3 a2 + b2 + c2
! !
= − (R − 2Rr) + r + − + 2
R −
2 2 9 3 4 9
1 3 a2 + b2 + c2 − 2p2
= R2 + Rr + r2 +
2 2 4
1 2 3 2 p2 − (ab + bc + ca)
= R + Rr + r + .
2 2 2

Đến đây ta cần đến một kết quả nhỏ liên quan đến biểu thức cuối cùng. Ta có

p2 r2 = S 2 = p(p − a)(p − b)(p − c).

Suy ra

pr2 = (p − a)(p − b)(p − c) = p3 − p2 (a + b + c) + p(ab + bc + ca) − abc


= −p3 + p(ab + bc + ca) − 4pRr.

Từ đó r2 = −p2 + ab + bc + ca − 4Rr. Suy ra −p2 + ab + bc + ca = r2 + 4Rr.


Thay vào biểu thức tính IE 2 , ta được

1 3 1 1
2
R

IE 2 = R2 + Rr + r2 − (r2 + 4Rr) = R2 − Rr + r2 = −r .
4 2 2 4 2

Từ công thức tính IO2 ta suy ra được R ≥ 2r. Do đó R2 − r ≥ 0 và ta có IE = R2 − r.


Từ đây, vì đường tròn Euler có bán kính bằng R2 nên đường tròn nội tiếp và đường tròn
Euler tiếp xúc trong với nhau.

Định lý Feuerbach có một hệ quả rất đẹp đẽ liên quan đến chùm tam giác nội tiếp (O) và
ngoài tiếp (I). Theo định lý Poncelet, nếu (O) là đường tròn ngoại tiếp và (I) là đường
tròn nội tiếp của tam giác ABC thì sẽ có vô số các tam giác nội tiếp (O) và ngoại tiếp
(I) và điểm nào trên (O) cũng là đỉnh của một tam giác như vậy. Một câu hỏi đặt ra là
khi các tam giác thay đổi nhưng luôn nội tiếp (O) và ngoại tiếp (I), trọng tâm của chúng
sẽ có quỹ tích thế nào?
Câu trả lời là khá bất ngờ nhưng lại vô cùng đơn giản dưới góc nhìn của định lý Feuerbach.
Gọi J là điểm trên đoạn IO sao cho OJ = 2JI. Khi đó J là điểm cố định và áp dụng
định lý Thales ta có JG = 2EI 3
= R−2r3
không đổi. Suy ra quỹ tích G là đường tròn tâm
J bán kính R−2r3
.

9
Tâm tỷ cự và các định lý kinh điển của hình học phẳng

4. Mở rộng sang tứ giác lưỡng tâm


Định nghĩa 4. Một tứ giác được gọi là tứ giác lưỡng tâm nếu nó vừa nội tiếp một
đường tròn, vừa ngoại tiếp một đường tròn khác.

Tứ giác lưỡng tâm có nhiều tính chất thú vị và đã được nghiên cứu khá nhiều. Ở đây ta
quan tâm đến vấn đề tính khoảng cách giữa hai tâm và đặt câu hỏi "Liệu có một công thức
tính khoảng cách này theo bán kính của hai đường tròn nội và ngoại tiếp hay không?",
tức là có một công thức tương tự như công thức d2 = R2 − 2Rr của tam giác hay không?
Câu trả lời được cho bởi định lý Fuss, có dạng
1 1 1
+ = 2,
(R − d)2 (R + d)2 r

trong đó như thường lệ R là bán kính đường tròn ngoại tiếp, r là bán kính đường tròn
nội tiếp còn d = IO là khoảng cách giữa hai tâm.
Điều thú vị là công thức Euler cho tam giác có thể viết lại thành
1 1 1
+ = ,
R−d R+d r

khá giống với dạng của định lý Fuss. Có nhiều cách chứng minh định lý Fuss, ở đây, ta
sẽ đưa ra cách chứng minh sử dụng tâm tỷ cự.
Như thường lệ, để áp dụng định lý Lagrange, ta cần tìm tọa độ tỷ cự của một trong hai
điểm O và I. Khác với trường hợp tam giác, ở đây ta không có một định lý tổng quát
giúp tìm tọa độ tỷ cự của một điểm đối với 4 đỉnh của một tứ giác. Ta sẽ tìm tọa độ tỷ
cự của điểm I của tứ giác ngoại tiếp ABCD dựa vào định lý con nhím (Định lý con
nhím phát biểu rằng với đa giác lồi P , nếu ta dựng các véc-tơ vuông góc với các cạnh,


hướng ra phía ngoài và có độ lớn bằng nhau thì tổng các véc tơ đã dựng bằng 0 . Các
véc-tơ dựng ra phía ngoài trông như lông nhím nên gọi là định lý con nhím).

Q
A
P

B N C

10
Chuyên đề Toán học Số 12

Chứng minh. Xét tứ giác ABCD ngoại tiếp đường tròn tâm I. Gọi M, N, P, Q là tiếp
điểm của AB, BC, CD, DA với (I). Đặt AM = AQ = x, BM = BN = y, CN = CP =
z, DP = DQ = t thì áp dụng định lý con nhím ta có
−−→ −→ −→ −→ → −
(x + y)IM + (y + z)IN + (z + t)IP + (t + x)IQ = 0 .
Ta có
−−→ −→ −−→ −
→ −−→ −→ −
→ −−→ −−→ −→ −

(x + y)IM = x(IB + BM ) + y(IA + AM ) = xIB + y IA + (xBM + y AM ) = xIB + y IA.
Ở đây hai véc-tơ cùng phương, ngược hướng và có độ lớn bằng nhau nên có tổng bằng


0 . Sử dụng các đẳng thức tương tự và thay vào, ta thu được

→ −→ −→ −→ →

(y + t)(IA + IC) + (x + z)(IB + ID) = 0 .
Đây chính là hệ thức mà ta cần. Chú ý là hệ thức này đúng cho mọi tứ giác ngoại tiếp
(không cần lưỡng tâm). Và hệ thức này có một hệ quả thú vị là đường thẳng Newton của
tứ giác ngoại tiếp đi qua tâm đường tròn nội tiếp của nó.
Bây giờ ta xét hệ điểm A, B, C, D với bộ trọng lượng (y + t, x + z, y + t, x + z) có tâm tỷ
cự là I. Áp dụng định lý Lagrange, ta có
(y + t)OA2 + (x + z)OB 2 + (y + t)OC 2 + (x + z)OD2
= 2(y + t + x + z)OI 2 + (y + t)IA2 + (x + z)IB 2 + (y + t)IC 2 + (x + z)ID2 .
Hay là
2(x + y + z + t)R2 = 2(x + y + z + t)OI 2 + (y + t)(2r2 + x2 + z 2 ) + (x + z)(2r2 + y 2 + t2 ).
       
Ta có x = r tan A2 , y = r tan B2 , z = r tan C2 , t = r tan D
2
. Do ∠A + ∠C =
180◦ , ∠B + ∠D = 180◦ , nên từ đây ta suy ra xz = yt = r2 .
Từ đây ta được
2(x + y + z + t)R2 = 2(x + y + z + t)OI 2 + (y + t)(2xz + x2 + z 2 ) + (x + z)(2yt + y 2 + t2 )
= 2(x + y + z + t)OI 2 + (x + z)(y + t)(x + y + z + t).
Từ đó ta suy ra
1
IO2 = R2 − (x + z)(y + t).
2
Ta có
A A B B
         
(x + z)(y + t) = r tan + r cot r tan + r cot
2 2 2 2
4r2 16r2 R2
= = ,
sin A sin B mn
trong đó m = AC, n = BD là độ dài các đường chéo của tứ giác lưỡng tâm. Đặt a =
AB, b = BC, c = CD, d = DA thì a + b + c + d và ta có các công thức quen thuộc sau
của tứ giác lưỡng tâm với các cạnh có độ dài a, b, c, d, diện tích S, các đường chéo m, n:
mn = ac + bd




a + c√= b + d



 S= abcd s
1 (ab + cd)(ac + bd)(ad + bc)

S


r = ,R =

.


a+c 4 abcd

11
Tâm tỷ cự và các định lý kinh điển của hình học phẳng

Từ các hệ thức này ta có

mn 4R2 (ac + bd)(a + c)2 (ab + cd)(ac + bd)(ad + bc)


− = −
4r2 mn 4abcd 4(ac + bd)abcd
(ac + bd)(a + c) 2
(ab + cd)(ad + bc)
= −
4abcd 4abcd
ac(b + d)2 + bd(a2 + 2ac + c2 ) − a2 bd − ab2 c − cad2 − bdc2 4abcd
= = = 1.
4abcd 4abcd
Từ đây tính được √
mn = 2r2 + 2r r2 + 4R2 .
Thay vào công thức ở trên ta có

16r2 R2 16r2 R2
(x + z)(y + t) = = 2 √
mn 2r + 2r r2 + 4R2
√ 
8rR2 r2 + 4R2 − r √ 
= = 2r r 2 + 4R2 − r .
r2 + 4R2 − r2

Từ đó suy ra đẳng thức sau đây, tương đương với định lý Fuss

IO2 = R2 + r2 − r r2 + 4R2 .


Qua chứng minh trên, ta còn thu nhận được một tính chất đặc biệt nữa là tích độ dài các
đường chéo của tứ giác lưỡng tâm tính được theo các bán kính R và r.
Chú ý là theo định lý Poncelet, nếu (I) và (O) là các đường tròn nội và ngoại tiếp của
một tứ giác lưỡng tâm thì sẽ có vô số tứ giác lưỡng tâm nội tiếp (O) và ngoại tiếp (I).
Như đã nói ở trên, tích độ dài các đường chéo không đổi và do đó diện tích tứ giác lưỡng
tâm này sẽ lớn nhất khi hai đường chéo vuông góc với nhau.
Một câu hỏi tự nhiên được đặt ra, tương tự như tình huống tam giác: Khi các tứ giác
thay đổi nhưng luôn nội tiếp (O) và ngoại tiếp (I) thì trọng tâm của tứ giác sẽ di chuyển
theo một quỹ tích nào?
Câu hỏi này cùng nhiều tính chất khác nữa của tứ giác lưỡng tâm (và n-giác lưỡng tâm
nói chung) đang chờ sự khám phá từ phía các bạn đọc yêu quý.

12
Tài liệu tham khảo
[1] Bicentric quadrilateral, Wikipedia : https://en.wikipedia.org/wiki/Bicentric_
quadrilateral

13
Tổng quát hóa tập tránh tỉ số và tập tránh tuyến tính
Nguyễn Quang Minh
Lớp Toán khóa 2018 – 2021

Giới thiệu. Các bài toán về tập tránh (tránh hiệu, tránh tổng, tránh tỉ số,
. . . ) vô cùng đa dạng, trải dài trên nhiều độ khó khác nhau. Trong phạm vi bài
viết này, chúng ta tìm hiểu về tập tránh tỉ số và một khái niệm rộng hơn là tập
tránh tuyến tính.

1. Sơ lược về tập tránh tỉ số

1.1. Các kết quả kinh điển


Định nghĩa 5. Với mỗi số hữu tỉ r, ta gọi một tập hợp X ⊂ Z+ là tập tránh tỉ số r
nếu không tồn tại 2 số thuộc X sao cho số này bằng r lần số kia.

Định nghĩa 6. Với n là số nguyên dương, kí hiệu gr (n) là số phần tử lớn nhất có thể
của một tập con tránh tỉ số r của tập hợp n số nguyên dương đầu tiên.

Việc nghiên cứu về chủ đề này, xuất phát từ những trường hợp khá cụ thể đã dẫn đến
nhiều kết quả kinh điển. Đầu tiên, Wang [1] chứng minh được rằng
 !
n n
 
g2 (n) = + g2 ,
2 4

trong đó g2 (0) = 0 và
2
g2 (n) = n + O(log(n)).
3
Tiếp theo, Leung và Wei [2] mở rộng tính chất trên sang tập hợp các số tự nhiên, và thu
được kết quả sau đây.

Định lý 1. Với mỗi số tự nhiên r > 1 thì


r
gr (n) = · n + O(log(n)).
r+1

Chưa dừng lại, Wakeham và Wood [3] tiếp tục quát sang các số hữu tỉ và chứng minh
được tính chất sau.

Định lý 2. Với a, b là các số nguyên dương nguyên tố cùng nhau và a < b thì

b
g b (n) = · n + O(log(n)).
a b+1
14
Chuyên đề Toán học Số 12

1.2. Tổng quát hoá tập tránh tỉ số


Ngoài cách tổng quát hóa như trên, ta cũng có thể tổng quát hóa tập tránh tỉ số theo
hướng nâng số lượng tỉ số. Thay vì chỉ yêu cầu tránh tỉ số k, ta sẽ yêu cầu tập hợp ta
đang xét cũng phải tránh các tỉ số k i , với i = 1, 2, . . . , q − 1 (với q > 1). Trong bài viết
này, chúng ta sẽ chủ yếu đề cập đến số phần tử cực đại của các tập như vậy.

Định nghĩa 7. Cho các số nguyên dương n, q và k (k ≥ 2, q ≥ 2). Tập con F của tập
hợp n số nguyên dương đầu tiên được gọi là tập tránh tỉ số (k, q) nếu không tồn tại 2
phần tử nào thuộc F mà tỉ số giữa số lớn và số bé là một trong các số k, k 2 , . . . , k q−1 . Ta
quy ước fk,q (n) là số phần tử lớn nhất có thể của tập F thoả mãn điều kiện trên.

Theo định nghĩa này thì fk,2 (n) = gk (n), còn fk,q (0) = 0. Ta thu được tính chất sau đây.
k−1
& '
n
 
Định lý 3. fk,q (n) = n + fk,q .
k kq
Chứng minh. Gọi X là tập các số nguyên dương không vượt quá knq và Y là tập hợp tất
cả các số nguyên dương lớn hơn knq và không vượt quá n.
Trước hết, ta nhận thấy mỗi phần tử của Y luôn có thể viết được dưới dạng t · k i , trong
đó i ∈ N, t là số nguyên dương không chia hết cho k. Các số t này, ứng với các phần tử
khác nhau trong F ∩ Y , cũng phải khác nhau, nếu không thì F ∩ Y sẽ chứa 2 số mà số
lớn bằng k j lầnl số bé,m trong đó 1 ≤ j < q (vô lí). l m
Từ 1 đến n có k−1 k
n số không chia hết cho K, suy ra |F ∩ Y | ≤ k−1
k
n , và

k−1
& '
n
 
|F | = |F ∩ Y | + |F ∩ X| ≤ n + fk,q .
k kq
Mặt khác, gọi tập con cực đại của X thoả mãn điều kiện đề bài là A1 . Thêm vào A1 các
số thuộc Y và lớn hơn nk , ta thu được tập A thoả yêu cầu đề bài. Vậy

k−1
& '
n n
   
fk,q (n) ≥ |A| = n − + |A1 | = n + fk,q .
k k kq
Từ hai đánh giá trên, ta suy ra được điều cần chứng minh. 
k q−1 (k − 1)
Định lý 4. fk,q (n) = n + O(log(n)).
kq − 1
Có thể chứng minh được hệ quả này dựa vào công thức
k−1 n
&  '
fk,q (n) =
X
,
i≥0 k k qi
j k j k
thu được khi áp dụng liên tục nhiều lần công thức truy hồi nêu trên (chú ý rằng bac
b
= ab
với mọi số thực a và số nguyên dương b). Bằng cách tổng quát trực tiếp cách chứng minh
của Wakeham và Wood, ta có thể mở rộng tính chất trên cho trường hợp k là số hữu tỉ.

Định lý 5. Với a, b là các số nguyên dương có ước chung lớn nhất là g và a < b thì
bq−1 (b − g)
f b ,q (n) = n + O(log(n)).
a bq − g q
15
Tổng quát hóa tập tránh tỉ số và tập tránh tuyến tính

2. Hai cách tiếp cận cho cùng một bài toán


Trên đây là một số công thức tính số phần tử lớn nhất của một tập tránh tỉ số trông khá
"trừu tượng". Vậy có cách nào để giải quyết triệt để, đưa ra một công thức tường minh
hơn cho bài toán này không? Khá thú vị, câu trả lời là có. Không những vậy, ta có thể
giải quyết nó theo hai hướng khác nhau hoàn toàn về bản chất. Để hiểu rõ hơn, hãy cùng
tìm hiểu 3 bài toán sau.

Bài 1. Chứng minh rằng với k, q là các số nguyên dương không nhỏ hơn 2 và n là số
nguyên dương tùy ý, ta luôn có

k q−1 (k n − 1)(k − 1)
$ %
fk,q (k ) =
n
+ 1.
kq − 1

Lời giải. Ta sẽ dùng phương pháp quy nạp theo n, cụ thể hơn là quy nạp nhảy cách từ
n lên n + q. Trước hết, xét 0 ≤ n < q. Rõ ràng khi n = 0 thì f (1) = 1. Nếu 1 ≤ n < q thì
từ Định lý 3, ta suy ra được
k−1 n k q−1 (k n − 1)(k − 1)
& ' $ %
fk,q (k ) =
n
k + fk,q (0) = k n−1 (k − 1) = + 1.
k kq − 1
Do đó mệnh đề đúng với n = 0, 1, . . . , q − 1. Cuối cùng, bước chuyển quy nạp từ n sang
n + q được thực hiện dựa trên đẳng thức:
k q−1 (k n − 1)(k − 1) k q−1 (k n−q − 1)(k − 1)
$ % $ %
+ 1 = + 1 + (k − 1)k n−1 .
kq − 1 kq − 1
Bài toán được giải quyết hoàn toàn.
Trên đây là công thức tính số phần tử của tập tránh tỉ số (k, q) thu được từ việc thiết
lập các hệ thức truy hồi. Bên cạnh cách tiếp cận đại số như trên, ta cũng có thể tiếp cận
bằng số học, cụ thể là phân hoạch tập n số nguyên dương đầu tiên thành các tập có dạng
{x, kx, k 2 x, . . . }, trong đó x là một số không chia hết cho k. Trên cơ sở đó, ta tiếp tục
thu được đẳng thức sau đây.

Bài 2. Chứng minh rằng với mọi k, q ∈ Z+ , k, q ≥ 2 và n ∈ Z+ thì


j  k
logk +1

n
t
fk,q (n) =
X
 .
1≤t≤n

 q 

t không chia hết cho k

Lời giải. Tương tự như trên, với mỗi t ≤ n, t không chia hết cho k, đặt St = {x ≤ n, x =
t · k i , i ∈ N}. Khi này, hợp của các tập St , với 1 ≤ t ≤ n, t 6≡ 0 (mod k) chính là tập hợp
n số nguyên dương đầu tiên. Do đó, ta chỉ cần xác định xem ta có thể chọn nhiều nhất
bao nhiêu số từ mỗi tập St là được. Trước hết, ta thấy
n n
 
t · k ≤ n ⇔ k ≤ ⇔ i ≤ logk
i i
.
t t

16
Chuyên đề Toán học Số 12

Do đó |St | = logk nt + 1 (St bao gồm các số t, kt, k 2 t, . . . , k blogk ( t )c t).


n
j  k

Tiếp theo, ta nhận thấy trong q phần tử liên tiếp của St , có tối đa 1 số được chọn vào
tập tránh tỉ số (k, q). Do đó có thể chọn tối đa
 
logk nt + 1
& '  
|St |
=
 
q  q 

số từ St . Dấu bằng xảy ra, chẳng hạn khi ta chọn các số có dạng
 
logk +1
 
n
t
t · k qi , i = 0, 1, . . . ,   − 1.

 q 

Đến đây, áp dụng tính chất nêu trên cho tất cả các số t ≤ n, t 6≡ 0 (mod k), ta suy ra
j  k
logk +1

n
t
fk,q (n) =
X
 .
1≤t≤n,t6≡0 (mod k) 

q 

Bài toán được giải quyết hoàn toàn.


Nhận xét. So với cách tiếp cận bằng truy hồi, cách tiếp cận này có thể chỉ ra một cách
tường minh thuật toán chọn tập tránh cực đại, thay vì chỉ xây dựng bằng quy nạp. Cụ
thể hơn, tập tránh cực đại ta chọn sẽ bao gồm tất cả các số nguyên dương không vượt
quá n và có thể viết được dưới dạng t·k qi , trong đó t là một số không chia hết cho k, i ∈ N.

Từ những nhận xét nêu trên, ta đi đến bài toán sau đây, thu được từ việc kết hợp
hai bài trên lại với nhau (số n trong bài 2 được thay bởi k n ).

Bài 3 (Problem 12252, Tạp chí AMM a ). Cho các số nguyên dương n, q và k (k ≥ 2).
Đặt P là tập hợp tất các các số nguyên dương nhỏ hơn k n và không là bội số của k.
Chứng minh rằng:

bn − logk (p)c k q−1 (k n−1 − 1)(k − 1)


& ' $ %
= + 1.
X

p∈P q kq − 1

a
Bài toán do tác giả đề xuất [4].

Nhận xét. Mặc dù cách đếm bằng hai cách cho ta đẳng thức trên, ta vẫn có thể dùng
biến đổi đại số và một ít kiến thức số học để chứng minh nó.
Lời giải. Trước hết, ta ký hiệu
bn − logk (p)c
& '
S=
X
.
p∈P q
l m l m l m
bn−logk (p)c
Để ý rằng q
chỉ có thể bằng 0, 1, 2, . . . , n
q
. Với i = 0, 1, 2, . . . , n
q
− 1, ta có

bn − logk (p)c
& '
= i ⇔ k n−q(i−1)−1 ≥ p > k n−qi−1 .
q
Ta xét hai trường hợp:

17
Tổng quát hóa tập tránh tỉ số và tập tránh tuyến tính
l m l m
Trường hợp 1. n − 1 không chia hết cho q. Trường hợp này, ta có n−1
q
= n
q
= t. Ta
có hai nhận xét quan trọng sau đây:
l m
• Số giá trị p ∈ P mà bn−logq k (p)c = i, với i = 0, 1, . . . , t − 1 chính là số số tự
nhiên không là bội của k nằm trong khoảng [k n−qi−1 , k n−q(i−1)−1 ]. Số lượng này bằng
k n−qi−2 (k q − 1)(k − 1).
l m
bn−logk (p)c
• Số giá trị p ∈ P mà q i
= t bằng với số số tự nhiên không là bội của k nằm
h
trong khoảng 1, k n−q(t−1)−1
. Số lượng này bằng k n−q(t−1)−2 (k − 1).
Vậy, S = A + B, trong đó
t−1 t−1

= k n−qi−2 (k q − 1)(k − 1)i = (k − 1)(k q − 1)k n−2 k −qi i
P P
A

i=1 i=1
B = k n−q(t−1)−2 (k − 1)t.

Ta lại có
t−1
k q − k q−qt tk q−qt
k −qi i =
X
− .
i=1 (k q − 1)2 kq − 1
Vậy
(k q − k q−qt )(k − 1)k n−2
A= − B.
kq − 1
Đặt n = qt − r (r ∈ N, r < q − 1) thì
(k − 1)k q−r−2 (k qt − 1)
S= .
kq − 1
kq−1 (kn−1 −1)(k−1)
j k
Để chứng minh S = kq −1
+ 1, ta chỉ cần chứng minh
(k − 1)k q−r−2 (k qt − 1) k q−1 (k n−1 − 1)(k − 1)
1≥ − > 0.
kq − 1 kq − 1
Bất đẳng thức này có thể được chứng minh bằng biến đổi tương đương.
Trường hợp 2. n = qt + 1 (t ∈ N)
Tương tự như trên, ta cũng có ba nhận xét quan trọng sau:
l m
• Với i = 1, 2, . . . , t−1, số giá trị p ∈ P mà bn−log
q
k pc
= i chính là số số tự nhiên không
là bội của k nằm trong khoảng [k n−qi−1 , k n−q(i−1)−1 ] và bằng k n−qi−2 (k q − 1)(k − 1).
l m
bn−logk pc
• Số giá trị p ∈ P mà = t bằng số số tự nhiên không là bội của k nằm trong
h i q
khoảng 2, k n−q(t−1)−1
và bằng k q−1 (k − 1) − 1.
l m
bn−logk pc
• Có đúng 1 số p ∈ P để q
= t + 1 là p = 1.
Vậy, S = C + D, trong đó
t−1 t−1

= k n−qi−2 (k q − 1)(k − 1)i = (k − 1)(k q − 1)k qt−1 k −qi i
P P
C

i=1 i=1
D = (t + 1) + (k q−1 (k − 1) − 1) t.

Tương tự như trong trường hợp trước, ta chỉ ra được


k q−1 (k qt − 1)(k − 1) k q−1 (k n−1 − 1)(k − 1)
$ %
S =C +D = + 1 = + 1.
kq − 1 kq − 1
Từ đây, ta suy ra được điều cần chứng minh.

18
Chuyên đề Toán học Số 12

3. Tập tránh tuyến tính


Trong phần này, ta sẽ đến với một khái niệm rộng hơn của tập tránh tỉ số là tập tránh
tuyến tính.
Định nghĩa 8. Cho k, b là các số nguyên, trong đó k > 1, b ≥ 0. Ta gọi tập hợp X ⊂ Z+
là tập tránh tuyến tính (k, b) nếu như với mọi x ∈ X thì kx + b ∈ / X.
Như vậy, tập tránh tỉ số là một trường hợp của tập tránh tuyến tính, trong trường hợp
b = 0.
Định nghĩa 9. Cho k > 1, b ≥ 0 là các số nguyên và số nguyên dương n. Khi này, số
phần tử lớn nhất có thể của tập con tránh tuyến tính (k, b) của tập hợp n số nguyên
dương đầu tiên được ký hiệu là gk,b (n).
Bạn đọc có thể tự chứng minh công thức truy hồi sau.
n(k − 1) + b n − b(k + 1)
& ' $ %!
Định lý 6. gk,b (n) = + gk,b với mọi n ≥ b.
k k2
(Quy ước gk,b (n) = 0 nếu n là số nguyên âm, và gk,b (n) = n nếu 0 ≤ n ≤ b.)
Tận dụng triệt để phương pháp chứng minh này, ta tổng quát tập tránh tuyến tính như
sau. Trước hết, đặt p(x) = kx + b là một biểu thức theo x. Ta định nghĩa p0 (x) = x và
pi (x) = p(pi−1 (x)) với mọi số nguyên dương i. Nói cách khác, pi (x) = p(p(. . . p(x) . . . ))
(vế phải bao gồm i dấu ngoặc), còn p−i (x) là hàm ngược của pi (x). Ta cũng đồng thời
quy ước số mũ của x chính là số tự nhiên i lớn nhất sao cho p−i (x) ∈ Z+ .
Định lý 7. pi (x) = k i x + (k i−1 + k i−2 + · · · + k + 1)b.
Định nghĩa 10. Cho số nguyên dương q (q ≥ 2). Tập con F của Z+ được gọi là tập
tránh tuyến tính (k, b, q) nếu với mọi x ∈ F thì p1 (x), p2 (x), . . . , pq−1 (x) ∈
/ F.
Định nghĩa 11. Với k, b, q ∈ Z thỏa mãn điều kiện k > 1, b ≥ 0, q ≥ 2, số phần tử lớn
nhất có thể của tập con tránh tuyến tính (k, b, q) của tập hợp n số nguyên dương đầu
tiên được kí hiệu là fk,b,q (n).
Như vậy, theo định nghĩa này thì fk,0,q (n) = fk,q (n). Ta cũng quy ước fk,b,q (n) = 0 trong
trường hợp n là số nguyên âm và fk,b,q (n) = n nếu 0 ≤ n ≤ b.
n(k − 1) + b b(k q − 1)
& ' $ %!
n
Định lý 8. fk,b,q (n) = + fk,b,q − với mọi n ≥ b.
k k q k q (k − 1)
Chứng minh. Tương tự như trong mục 2, tập tránh tuyến tính (k, b, q) cực đại sẽ bao
gồm tất cả các số có số mũ chia hết cho q. Có tổng cộng
k−1 n(k − 1) + b
& ' & '
b + (n − b) · =
k k
số có số mũ bằng 0. Hơn nữa, với các số còn lại, giá trị của hàm số p−q tại chúng cũng
phải là số nguyên dương và có số mũ chia hết cho q (có thể bằng 0). Do các số này đều
không vượt quá n nên p−q của chúng không vượt quá
n − (k q−1 + k q−2 + · · · + k + 1)b b(k q − 1)
$ % $ %
n
= − .
kq k q k q (k − 1)

19
Tổng quát hóa tập tránh tỉ số và tập tránh tuyến tính

q q
j k j k
−1) −1)
Từ 1 đến knq − kb(k
q (k−1) có đúng fk,b,q knq − kb(k
q (k−1) số có số mũ chia hết cho q. Cộng
các số ở 2 nhóm (nhóm có số mũ bằng 0 và nhóm có số mũ khác 0) lại, ta suy ra được
điều cần chứng minh. 

4. Một số bài toán minh họa


Các bài toán ở mục này nhằm phục vụ hai mục đích: một là để làm ví dụ cho các định lý
và hệ quả có phần rắc rối ở các mục trước, và hai là để đưa ra một số hướng khai thác,
mở rộng khác trong tương lai để bạn đọc cùng tiếp tục tìm hiểu.
Bài tập 1. Có thể chọn được nhiều nhất bao nhiêu số từ tập 2021 số nguyên dương đầu
tiên, sao cho với mỗi số x được chọn thì các số 2x + 1 và 4x + 3 không được chọn?
Lời giải. Ta cần tính f2,1,3 (2021). Ta có
1(23 −1)
 l m j k
f2,1,3 (2021) = 2021+1
+ f2,1,3 2021
− = 1011 + f2,1,3 (251)
2 23 23 (2−1)

2,1,3 (251) = 126 + f2,1,3 (30) = 126 + 16 + f2,1,3 (2) = 126 + 16 + 2 = 144.
f

Suy ra f2,1,3 (2021) = 1011 + 144 = 1155.


Bài tập 2. Có thể chọn được nhiều nhất bao nhiêu số tự nhiên từ tập {101, 102, . . . , 1000},
sao cho với mỗi số x được chọn thì số 3x không được chọn?
Lời giải. Ta tịnh tiến các số trên trục toạ độ về bên trái 100 đơn vị, đưa bài toán
đặt ra về bài toán sau đây: Có thể chọn được nhiều nhất bao nhiêu số tự nhiên từ tập
{1, 2, 3, . . . , 900}, sao cho với mỗi số x được chọn thì số 3x + 200 không được chọn?
Kết quả của bài toán là
900 · (3 − 1) + 200 900 − 200 · (3 + 1)
& ' $ %!
g3,200 (900) = + g3,200
3 32
= 667 + g3,200 (11) = 667 + 11 = 678.
Bài tập 3. Trong mặt phẳng toạ độ Oxy, cho các điểm A(26, 0), B(26, 101), C(0, 101).
Tường đánh dấu một số điểm nguyên nằm trong hình chữ nhật OABC (không tính các
điểm biên). Biết rằng nếu điểm X đã được đánh dấu thì trung điểm của OX không được
đánh dấu nữa. Hỏi Tường đã đánh dấu tối đa bao nhiêu điểm?
Bài tập 4. ([6]) Cho các số nguyên k, b, q (k > 1, b ≥ 0, q ≥ 2). Với n là số nguyên dương,
một tập con S của tập hợp n số nguyên dương đầu tiên được gọi là n-đầy đủ nếu như S
là tập tránh tuyến tính (k, b, q), tuy nhiên nếu thêm vào S bất kỳ số nguyên dương t ≤ n
nào thì tập mới thu được không còn là tập tránh tuyến tính (k, b, q) nữa. Kí hiệu ak,b,q (n)
là số phần tử nhỏ nhất có thể của một tập n-đầy đủ. Chứng minh rằng
a) Với mọi số nguyên dương n ≥ b thì
n(k − 1) + b b(k 2q−1 − 1)
& ' $ %!
n
ak,b,q (n) = + ak,b,q − 2q−1 .
k k 2q−1 k (k − 1)

Quy ước ak,b,q (n) = 0 nếu n < 0 và ak,b,q (n) = n, trong trường hợp 0 ≤ n ≤ b. Từ
đó, hoặc bằng cách khác, hãy đưa ra công thức tính ak,b,q (n).

20
Chuyên đề Toán học Số 12

b) Với mỗi x ∈ [ak,b,q (n), fk,b,q (n)], luôn tồn tại tập hợp n-đầy đủ có đúng x phần tử.

Gợi ý. Ta có thể sử dụng bổ đề sau:


Tập hợp S được gọi là tập giãn cách q nếu hiệu giữa 2 phần tử phân biệt bất kỳ
trong S không nhỏ hơn q. Xét tập hợp Hq (n) gồm các tập hợp S là tập con của tập
hợp n số nguyên dương đầu tiên thoả mãn điều kiện S là tập giãn cách q, nhưng khi
thêm vào S bất kỳ số t ≤ n nào thì tập mới thu được không còn là tập giãn cách q
nữa. Khi đó, tồn tại tập hợp A ∈ Hq (n) có đúng x phần tử khi và chỉ khi
& ' & '
n n
≥x≥ .
q 2q − 1

Bài tập 5. (Lee [7]) Với mỗi số thực p ∈ (0, 1), đặt [n]p là tập các số tự nhiên thu được
bằng cách chọn mỗi phần tử thuộc tập hợp n số nguyên dương đầu tiên một cách độc lập
và ngẫu nhiên với xác suất là p. Giả sử a, b là các số nguyên dương nguyên tố cùng nhau
và a < b. Đặt g b ([n]p ) là kích cỡ lớn nhất của tập tránh tỉ số ab cực đại mà đồng thời là
a
tập con của [n]p . Chứng minh rằng giá trị kì vọng của g b ([n]p ) được cho bởi công thức
a

  b
E g b ([n]p ) = pn + O (log(n)) .
a b+p

21
Tài liệu tham khảo
[1] Edward T. H. Wang. On double-free sets of integers. Ars Combinatoria, 28:97–100,
1989

[2] J. Y.-T. Leung and W.-D. Wei. Maximal k-multiple-free sets of integers. Ars Combi-
natoria, 38:113–117, 1994

[3] D. Wakeham and D. Wood. On multiplicative Sidon sets. arXiv:1107.1073

[4] Nguyen Quang Minh. (2021) Problems 12252, The American Mathematical Monthly,
128:5, 467

[5] Lihua You, Gexin Yu, Bolian Liu. On maximal (k, b)-linear-free sets of integers and
its spectrum. Australasian Journal of Combinatorics, 23 (2001), pp.211-215

[6] Nguyen Quang Minh. A Generalisation of Maximal (k,b)-Linear-Free Sets of Integers.


Journal of Combinatorial Mathematics and Combinatorial Computing (to appear).

[7] Sang June Lee. On constant-multiple-free sets contained in random subsets of integers.
Ars Combinatoria 113A (2014) 247-256.

22
Về một ý tưởng quy nạp theo số chữ số
Trần Nguyễn Nam Hưng
Lớp Toán khóa 2018 – 2021

Giới thiệu. Bài viết sẽ giới thiệu đến bạn đọc một ý tưởng quy nạp khá tự
nhiên và đẹp đẽ trong số học. Dù khá đơn giản về mặt kỹ thuật, ý tưởng này
lại có những ứng dụng và mở rộng bất ngờ, vượt xa khuôn khổ của sơ đồ giải
toán ban đầu.

1. Bài toán mở đầu


"Cội nguồn" của ý tưởng quy nạp theo số chữ số là bài toán sau đây.
Bài toán mở đầu (Mỹ 2003).

Chứng minh rằng với mỗi số nguyên dương n, luôn tồn tại một số nguyên dương
có n chữ số, có các chữ số đều lẻ và chia hết cho 5n .

Lời giải. Ta sẽ giải quyết bài toán bằng phương pháp quy nạp theo n.
Khi n = 1, dễ thấy mệnh đề đúng vì số 5 thỏa mãn yêu cầu đề bài. Giả sử ta xây dựng
được số S thỏa mãn yêu cầu đề bài với n = k. Xét 5 số sau đây

S + 10k , S + 3 · 10k , . . . , S + 9 · 10k

thu được từ việc thêm vào bên trái S các chữ số 1, 3, 5, 7, 9. Dễ thấy 5 số này đều chia
hết cho 5k và không có 2 số nào có cùng số dư khi chia cho 5k+1 . Số dư khi chia chúng
cho 5k+1 sẽ là
0, 5k , 2 · 5k , 3 · 5k , 4 · 5k

theo thứ tự nào đó. Vậy trong 5 số trên có một số chia hết cho 5k+1 , do đó tồn tại một
số có k + 1 thỏa mãn yêu cầu đề bài, và mệnh đề đúng với n = k + 1.
Theo nguyên lí quy nạp toán học, mệnh đề đúng với mọi n ∈ Z+ . Lời giải hoàn tất.

2. Một số mở rộng trực tiếp


Bản chất của ý tưởng quy nạp nêu trên là xây dựng một dãy gồm vô hạn chữ số.
Từ đây, cộng với việc kết hợp các bài toán nhỏ lại với nhau một cách khéo léo, ta có thể
dễ dàng giải quyết hai bài toán sau đây.

Bài 1. Tìm tất cả các số nguyên dương n sao cho tồn tại một bội số nguyên dương
của n mà trong biểu diễn thập phân của nó chỉ chứa các chữ số 1 hoặc 2.

Lời giải. Trước hết, ta sẽ chứng minh tính chất sau đây.

23
Về một ý tưởng quy nạp theo số chữ số

Tính chất 1. Với mỗi số nguyên dương n, tồn tại một số có n chữ số sao cho nó chỉ chứa
các chữ số 1 và 2, đồng thời chia hết cho 2n .

Chứng minh. Với n = 1, dễ thấy ta có số 2 thỏa đề. Hơn nữa, với số x có n chữ số chia
hết cho 2n , ta có thể thu được số y có n + 1 chữ số và chia hết cho 2n+1 theo cách sau :

• Kiểm tra xem 2n+1 có chia hết cho x hay không.

• Nếu có, ta thêm chữ số 2 vào bên trái x để thu được y, ngược lại ta thêm chữ số 1
vào bên trái x để thu được y.

Từ đây, theo nguyên lý quy nạp, ta suy ra điều cần chứng minh. 
Trở lại bài toán, trước hết, ta nhận thấy một số có chữ số tận cùng là 1 hoặc 2 thì không
chia hết cho 5. Do đó, các số chia hết cho 5 không thỏa đề.
Xét trường hợp n = 2k · l, trong đó k ∈ N và l ∈ Z+ , (l, 10) = 1. Theo nhận xét trên, ta
suy ra tồn tại số S có k chữ số, mỗi chữ số bằng 1 hoặc 2 và chia hết cho 2k . Hơn nữa,
với mỗi số nguyên dương a, ta luôn có

SS . . . S (các chữ số của S được lặp lại a lần)


10ak − 1
= S · (1 + 10k + · · · + 10(a−1)k ) = S · .
10k − 1
Bây giờ, ta chỉ cần chọn a sao cho

10ak − 1
chia hết cho l ⇔ 10ak − 1 chia hết cho l(10k − 1)
10k − 1

là được. Theo định lý Euler, ta có thể chọn a = ϕ(l(10k − 1)). Dễ thấy biểu diễn thập
phân của số này chỉ chứa các chữ số 1 hoặc 2. Từ đây, ta suy ra n thỏa đề.
Vậy tất cả các số n cần tìm là những số không chia hết cho 5.

Bài 2 (Brazil 1994). Ta gọi một siêu số nguyên là một dãy gồm vô hạn các
chữ số trong hệ thập phân: . . . dn . . . d3 d2 d1 . Cho 2 siêu số nguyên . . . cn . . . c3 c2 c1 và
. . . dn . . . d3 d2 d1 , tích của chúng . . . pn . . . p3 p2 p1 được xác định như sau: pn . . . p3 p2 p1 là
n chữ số tận cùng của tích của cn . . . c3 c2 c1 và dn . . . d3 d2 d1 . Hỏi ta có thể tìm được hay
không hai siêu số nguyên khác 0 có tích bằng 0 (nghĩa là tất cả các chữ số trong tích
này đều bằng 0)?

Lời giải. Từ các kết quả thu được trong hai bài toán trên, ta suy ra có thể chọn 2 dãy
các chữ số khác 0 (cn )n≥1 và (dn )n≥1 sao cho với mọi số nguyên dương n, ta luôn có

5n | cn cn−1 . . . c1 , 2n | dn dn−1 . . . d1 .

Chọn c = . . . cn . . . c3 c2 c1 , d = . . . dn . . . d3 d2 d1 , ta suy ra

10n | cn cn−1 . . . c2 c1 · dn dn−1 . . . d2 d1

và pn = 0 với mọi n, hay p = 0, mặc dù c, d khác 0. Vậy câu trả lời là khẳng định.

24
Chuyên đề Toán học Số 12

3. Xử lý các điều kiện phức tạp hơn về biểu diễn thập


phân
Trong các bài toán ở phần trước, từ các điều kiện về biểu diễn thập phân, ta có thể dễ
dàng đi đến thuật toán quy nạp theo số chữ số. Tuy nhiên, trong nhiều trường hợp khác,
các điều kiện này lại là trở ngại lớn trong việc định hướng. Để xử lý được chúng, bên
cạnh việc sử dụng sơ đồ quy nạp nêu trên, cần thiết lập các điều kiện bổ sung để có thể
xây dựng thuật toán một cách tường minh, chặt chẽ.

Bài 3 (IMO Shortlist 1994). Một số nguyên dương được gọi là lung lay nếu như các
chữ số của nó, theo thứ tự từ trái qua phải, luân phiên nhau bằng 0 và khác 0, đồng
thời chữ số tận cùng của nó phải khác 0. Ví dụ, 201 và 30507 là các số lung lay. Tìm
tất cả các số nguyên dương sao cho nó không là ước của bất kỳ số lung lay nào.
Nhận xét. Vì ta chỉ có thể kiểm soát các chữ số ở vị trí lẻ của số lung lay nên ở mỗi
bước nhảy, ta sẽ thêm hai chữ số, thay vì một chữ số như trong hai bài toán trước đó.

Lời giải. Xét các trường hợp sau đây.

Trường hợp 1. (10, n) = 1. Khi này, dễ thấy (100, 99n) = 1. Với mỗi a ∈ Z+ , ta có

100a − 1
Sa = 10101 . . . 01 (bao gồm a chữ số 1) = .
99
Theo định lý Euler, với a = ϕ(99n), ta có

99n | 100a − 1 ⇒ n | Sa .

Từ đây, ta suy ra S chia hết cho n, do đó n không thoả đề.

Trường hợp 2. n chia hết cho 5. Ta xét các trường hợp nhỏ sau đây:

• n chia hết cho 25 hoặc 10. Nếu n chia hết cho 25, mọi bội của n đều có 2 chữ số tận
cùng là 25, 50, 75, 00 và không là số lung lay. Nếu n chia hết cho 10, các bội của n
đều có chữ số tận cùng bằng 0 và cũng không là số lung lay.

• n = 5l, trong đó (l, 10) = 1. Tương tự trường hợp 1, tồn tại a ∈ Z+ sao cho

Sa = 101
| {z. . . 01} chia hết cho l ⇒ 5Sa = 505 . . . 05 chia hết cho n.
a chữ số 1

Do đó n không thoả đề.

Trường hợp 3. (10, n) = 2. Khi này, n = 2k · l, trong đó k ∈ Z+ và (10, l) = 1. Tồn tại


một số có 2k − 1 chữ số, lung lay và chia hết cho 22k . Chứng minh. Thật vậy, với k = 1,
ta có thể chọn số 4. Hơn nữa, giả sử với k, ta chọn được số S = d2m−1 . . . d2 d1 thoả mãn.
Khi này, 4 số

10d2m−1 . . . d2 d1 , 20d2m−1 . . . d2 d1 , 30d2m−1 . . . d2 d1 , 40d2m−1 . . . d2 d1

25
Về một ý tưởng quy nạp theo số chữ số

đều chia hết cho 22k , đồng thời không có 2 số nào trong chúng có cùng số dư khi chia cho
22k+2 . Do đó, trong 4 số này, luôn có 1 số chia hết cho 22k+2 , và mệnh đề đúng tại k + 1.
Theo nguyên lý quy nạp, ta suy ra mệnh đề đúng với mọi k ∈ Z+ . 
Trở lại bài toán, nếu l = 1 thì áp dụng nhận xét trên, ta suy ra ngay điều cần chứng
minh, do 2k | 22k . Với l > 1, tương tự, ta cũng chọn được số S lung lay, có 2k − 1 chữ số
và chia hết cho 22k . Dễ thấy

102ak − 1
Sa = S(1 + 102k + 104k + · · · + 102(a−1)k ) =
102k − 1

cũng là 1 số lung lay. Tương tự bài 1, ta chỉ cần chọn a sao cho

102ak − 1
l| ⇔ l(102k − 1) | 102ak − 1.
10 − 1
2k

 
Theo định lý Euler, ta có thể chọn a = ϕ l(102k − 1) .

Từ đây, ta suy ra các số cần tìm là những số chia hết cho ít nhất 1 trong 2 số 25 hoặc 10.

Bài 4 (IMO Shortlist 1998). Chứng minh rằng với mỗi số nguyên dương n, tồn tại
một số nguyên dương có đúng n chữ số, không chứa chữ số 0, đồng thời chia hết cho
tổng các chữ số của nó.
Nhận xét. Yêu cầu lỏng lẻo về biểu diễn thập phân vừa là cái dễ vừa là cái khó của bài
toán này. Vậy ta phải làm chặt hơn các điều kiện này, để dễ dàng xây dựng thuật toán
một cách tường minh.

Lời giải. Xét các trường hợp sau đây.

Trường hợp 1. n ≤ 9. Khi này, số A = 11 . . . 1a, trong đó a = 10 − n thoả đề.

Trường hợp 2. n ≥ 10. Khi này, tồn tại duy nhất các số nguyên dương k và l sao cho
n = 9 · 2k−1 + l (l ≤ 9 · 2k−1 ). Ta sẽ chứng minh rằng tồn tại số x có n chữ số sao cho :

• n chia hết cho 2k+1

• Tổng các chữ số của x bằng 9 · 2k+1 .

Thật vậy, theo nhận xét ở bài 1 thì ta có thể chọn k + 1 chữ số cuối cùng của x để 2k+1 | x,
đồng thời các chữ số được chọn không vượt quá 2. Điều quan trọng bây giờ là chứng minh
rằng ta có cách chọn các chữ số còn lại để x thoả mãn các điều kiện còn lại. Thật vậy,
gọi M là số chữ số ta chưa lựa chọn và N là tổng của chúng. Khi này

9 · 2k−1 + 1 ≤ M ≤ 9 · 2k − (k + 1)
9 · 2k+1 − (2k + 2) ≤ N ≤ 9 · 2k+1 − (k + 1)

nên ta suy ra
9 · 2k+1 − (2k + 2) N 9 · 2k+1 − (k + 1)
≤ ≤ .
9 · 2k − (k + 1) M 9 · 2k−1 + 1 − (k + 1)
26
Chuyên đề Toán học Số 12

Bằng các biến đổi thông thường, ta suy ra được


9 · 2k+1 − (2k + 2) 9 · 2k+1 − (k + 1)
1≤ < ≤ 9.
9 · 2k − (k + 1) 9 · 2k−1 + 1 − (k + 1)
Do đó, ta có thể chọn các chữ số còn lại để thu được số x thoả các điều kiện trên. Với x
này, dễ thấy x chia hết cho tổng các chữ số của nó.
Từ hai trường hợp nêu trên, ta suy ra được điều cần chứng minh.

4. Mở rộng dưới góc nhìn các phân môn khác


Việc nhìn bài toán gốc dưới con mắt của giải tích và tổ hợp, đi sâu hơn vào các kết
quả thu được từ sơ đồ quy nạp ban đầu đã giúp tác giả tạo ra được nhiều bài toán thú
vị, mới mẻ cả về mặt hình thức lẫn nội dung.

4.1. Tổ hợp
Ta có thể lồng vào bài toán chứng minh ban đầu các dạng toán cơ bản của tổ hợp nhằm
làm mới, tạo ra nhiều phiên bản khác nhau của thuật toán lựa chọn chữ số.

Bài 5. Với mỗi số nguyên dương n, kí hiệu Sn là tập hợp tất cả các số nguyên dương
có n chữ số, không chứa chữ số 0 và 9, đồng thời chia hết cho 2n . Hãy tính theo n số
phần tử của tập hợp Sn .

Lời giải. Để số S = an an−1 . . . a1 thuộc Sn , ta phải có

ai ai−1 . . . a1 chia hết cho 2i , i = 1, 2, . . . , n.

Để thuận lợi trong việc lựa chọn, ta sẽ chọn theo thứ tự từ phải sang. Trước hết, ta cần
có a1 chẵn. Có 4 cách chọn a1 (a1 = 2, a1 = 4, a1 = 6, a1 = 8). Đến đây, với mỗi i < n,
khi đã có ai ai−1 . . . a2 a1 chia hết cho 2i , ta sẽ tính số cách chọn ai+1 sao cho ai+1 ai . . . a2 a1
chia hết cho 2i+1 . Thật vậy, ta xét 2 trường hợp sau :
Trường hợp 1. S ≡ 0 (mod 2i+1 ). Khi này, có 4 giá trị có thể cho ai+1 là 2, 4, 6, 8.
Trường hợp 2. S ≡ 2i (mod 2i+1 ). Khi này, tương tự, ta có thể chọn ai+1 ∈ {1, 3, 5, 7}.
Từ đây, ta suy ra có 4 cách chọn ai+1 , khi đã chọn được các chữ số a1 , a2 , . . . , ai thỏa đề.
Do đó, với cách chọn như vậy, ta sẽ có 4 cách chọn a1 , 4 cách chọn a2 , . . . , 4 cách chọn
an . Từ đây, ta suy ra |Sn | = 4n . Lời giải hoàn tất.
Nhận xét. Ký hiệu Si,n là tập các số thuộc Sn và chia hết cho 2n+i . Khi này, từ kết quả
của bài toán trên, ta dễ dàng thu được hai tính chất sau đây.
Tính chất 2. |S1,n | = 22n−1 .
Tính chất 3. |S2,n | = 4n−1 .
Với i ≥ 3, ta có thể tính số phần tử của Si,n bằng phương pháp này hay không ? Có cách
nào khác tốt hơn hay không ? Xin nhường lại cho bạn đọc tự tìm hiểu.

27
Về một ý tưởng quy nạp theo số chữ số

Bài 6. Trung và Tường chơi trò chơi sau đây. Trong trò chơi này, họ phải tạo ra một
số có 2n chữ số, có các chữ số đều khác 0 và không vượt quá c. Trung chọn các chữ số
ở vị trí lẻ (từ trái sang), sau đó Tường sẽ chọn các chữ số còn lại. Tường thắng nếu số
thu được chia hết cho 22n , ngược lại thì Trung thắng. Tìm số c nhỏ nhất để Tường có
chiến thuật thắng.

Lời giải. Nếu c = 1, dễ thấy Trung luôn thắng. Với c = 2 và c = 3, để chiến thắng, Trung
chỉ cần chọn chữ số hàng chục là 2.
Ta sẽ chứng minh rằng với c = 4, Tường có chiến thuật để thắng trò chơi. Thật vậy, gọi
n chữ số mà Trung điền (từ trái sang) là an , an−1 , . . . , a1 . Khi này, Tường cần chọn các
chữ số bn , bn−1 , . . . , b1 ∈ {1, 2, 3, 4} sao cho an bn . . . a1 b1 chia hết cho 22n . Để an bn . . . a1 b1
chia hết cho 22n , ta cần có

ai bi . . . a1 b1 chia hết cho 22i với mọi i = 1, 2, . . . , n.

Như vậy, ta cần có a1 b1 chia hết cho 4. Khi này, nếu a1 ∈ {2, 4} thì Tường chọn b1 = 4,
ngược lại thì Tường chọn b1 = 2. Hơn nữa, khi đã chọn các chữ số b1 , b2 , . . . , bi , Tường cần
chọn chữ số bi+1 sao cho ai+1 bi+1 . . . a2 b2 a1 b1 chia hết cho 22i+2 . Đặt S = ai+1 0ai bi . . . a1 b1 .
Khi này, ứng với bi+1 = 1, 2, 3, 4, ta thu được các số

S + 102i , S + 2 · 102i , S + 3 · 102i , S + 4 · 102i .

Trong 4 số này luôn có một số chia hết cho 22i+2 , do đó Tường có thể chọn tiếp chữ số
bi+1 thoả mãn. Cứ tiếp tục quá trình trên, Tường sẽ chọn được các chữ số b1 , b2 , . . . , bn
sao cho an bn . . . a1 b1 chia hết cho 22n , và thắng trò chơi.

4.2. Giải tích


Vì kết quả thu được từ sơ đồ quy nạp là một dãy gồm các chữ số, ta sẽ thử khảo sát một
vài tính chất của dãy số này.

Bài 7. Xét dãy số (an )n≥1 thỏa mãn điều kiện với mọi n ∈ Z+ thì an ∈ {1, 3, 5, 7, 9}
và an an−1 . . . a1 chia hết cho 5n . Chứng minh rằng dãy số (an )n≥1 không tuần hoàn từ
bất kỳ số hạng nào.

Lời giải. Giả sử ngược lại, tức tồn tại các số nguyên dương N, T sao cho an = an+T với
mọi số nguyên dương n > N . Đặt

Sn = an an−1 . . . a2 a1


SN +T − SN
A = aN +T aN +T −1 . . . aN +1 = .
10N
Khi này, ta có

SnT +N = SN + 10N · A · (1 + 10T + · · · + 10(n−1)T )


10nT +N · A − A · 10N + SN (10T − 1)
= .
10T − 1

28
Chuyên đề Toán học Số 12

Dễ thấy SN (10T − 1) là số lẻ còn A · 10N là số chẵn, suy ra SN (10T − 1) − A · 10N khác


0, do đó tồn tại số nguyên dương k sao cho

5k - SN (10T − 1) − A · 10N .

Chọn n sao cho nT + N > k, ta có 5nT +N - SnT +N (vô lí). Từ đây, ta suy ra điều giả sử
sai, và thu được điều cần chứng minh.

5. Bài tập tự luyện


Bài tập 1. Với a, b là 2 chữ số khác 0 và khác nhau, đặt Sa,b là tập các số nguyên dương
n sao cho tồn tại một bội của n mà các chữ số của nó đều thuộc tập {a, b}. Biết rằng tồn
tại 2 chữ số a0 , b0 sao cho Sa,b ⊂ Sa0 ,b0 với mọi a, b, đồng thời a0 < b0 . Xác định a0 , b0 .
Bài tập 2. Một số nguyên dương được gọi là tốt nếu như các chữ số trong biểu diễn thập
phân của nó, theo thứ tự từ trái sang phải, tạo thành một dãy số không giảm. Chứng
minh rằng tồn tại số nguyên dương n sao cho mọi số tốt đều không chia hết cho 2n .
Bài tập 3. Hai số nguyên dương a = a1 a2 . . . am và b = b1 b2 . . . bn được gọi là liên kết nếu
a1 a2 . . . am b1 b2 . . . bn hoặc b1 b2 . . . bn a1 a2 . . . am chia hết cho 2m+n . Xét đồ thị G = (V, E)
có các đỉnh là các số nguyên dương thoả mãn hai đỉnh kề nhau khi và chỉ khi hai số tương
ứng là hai số liên kết. Chứng minh rằng G là đồ thị liên thông.
Bài tập 4 (IMO 2004). Một số nguyên dương được gọi là luân phiên nếu như hai chữ
số kề nhau bất kỳ trong biểu diễn thập phân của nó khác tính chẵn lẻ. Tìm tất cả các số
nguyên dương n sao cho tồn tại một bội số của n là số luân phiên.
Bài tập 5 (Chọn đội tuyển KHTN 2016). Cho n là số nguyên dương. Chứng minh rằng
tồn tại số nguyên dương m có n chữ số, chỉ chứa các chữ số thuộc tập hợp {1, 2, 3} trong
biểu diễn thập phân và chia hết cho tổng các chữ số của nó.
Bài tập 6. Cho số nguyên dương n. Sơn và Việt chơi trò chơi sau đây. Trong trò chơi
này, họ cần tạo ra một số có 3n chữ số, gọi là a3n a3n−1 . . . a2 a1 sao cho ai ∈
/ {0, 9} với mọi
i. Đầu tiên, Việt sẽ chọn ra trước một tập con k phần tử tùy ý của tập hợp 3n số nguyên
dương đầu tiên, gọi là S. Sau đó, Sơn chọn tất cả các chữ số ai với i thuộc S. Cuối cùng,
Việt chọn các chữ số còn lại. Việt thắng nếu số tạo thành chia hết cho 23n , ngược lại thì
Sơn thắng. Tìm số k lớn nhất để Việt có chiến thuật thắng trò chơi.
Bài tập 7. Ta nói một bảng ô vuông n × n (bảng gồm n hàng, n cột) là hài hòa nếu
như mỗi ô vuông của bảng được điền một chữ số khác 0, hơn nữa nếu đọc các số thuộc
một hàng theo thứ tự từ trái sang phải hoặc đọc các số thuộc cùng một cột theo thứ tự
từ trên xuống dưới, ta thu được một số chia hết cho 2n .
Ví dụ, bảng sau đây là bảng hài hòa, vì 312, 136, 264 đều chia hết cho 8.
3 1 2
1 3 6
2 6 4
Chứng minh rằng với mỗi số nguyên dương n, luôn tồn tại một bảng n × n hài hòa.

29
Tài liệu tham khảo
[1] IMO Shortlist các năm 1994, 1998, 2004

[2] Diễn đàn Art Of Problem Solving : https://artofproblemsolving.com

[3] Olympic KHTN (2014 - 2019) và Các bài toán chọn lọc từ kỳ thi đội tuyển (Lê Phúc
Lữ)

[4] Thirty Years Of Brazilian Math Olympiads (Carlos Yuzo Shine)

30
Tổng quát hoá các bài hình bằng phép chia tỉ lệ
Trần Nguyễn Thanh Danh
Lớp Toán khóa 2020 – 2023

1. Bài toán mở đầu


Bài toán sau đây đã tạo nên ý tưởng tổng quát hoá các bài toán hình học cho tác giả.
Bài toán mở đầu
Cho tam giác ABC nhọn nội tiếp đường tròn (O). Một đường thẳng d bất kỳ cắt
3 đường tròn (BOC), (COA), (AOB) lần lượt tại A0 , B 0 , C 0 . A0 , B0 , C0 là 3 điểm
nằm trên AO, BO, CO và chia các đoạn này theo tỷ lệ bằng nhau. Chứng minh
rằng A0 A0 , B0 B 0 , C0 C 0 đồng quy.

2. Các ý tưởng chứng minh


Trong quá trình giải các bài toán, thì các phương pháp chứng minh chủ yếu là sử dụng
định lý Thales, điểm liên hợp đẳng giác và tỉ số kép để kết thúc vấn đề, sau khi đã chứng
minh xong các trường hợp đặc biệt.
Ta sẽ bắt đầu với công cụ đơn giản nhất là định lý Thales.

2.1. Định lý Thales

Bài 1. Cho tam giác ABC không cân nội tiếp (O) có ba đường cao AD, BE, CF cắt
nhau tại H. A0 , B0 , C0 lần lượt nằm trên các đoạn HA, HB, HC và chia các đoạn này
theo tỷ lệ bằng nhau. Gọi Ma , Mb , Mc lần lượt là trung điểm của BC, CA, AB. Đường
thẳng qua A0 , vuông góc HMa cắt BC tại A1 . Các điểm B1 , C1 được xác định tương
tự. Chứng minh rằng A1 , B1 , C1 thẳng hàng.

Lời giải. Đặt


HA0 HB0 HC0
k= = = .
HA HB HC
Đường thẳng qua H, vuông góc với HMa cắt BC tại A3 . Đường thẳng qua A, vuông góc
với HMa cắt BC tại A2 . Các điểm B2 , B3 , C2 , C3 được xác định tương tự.
Trước hết, ta sẽ chứng minh rằng A2 , B2 , C2 và A3 , B3 , C3 thẳng hàng. Nói cách khác, ta
sẽ giải quyết bài toán trong trường hợp k = 0 và k = 1.

Tính chất 1. A2 , B2 , C2 thẳng hàng, đồng thời đường thẳng (d2 ) đi qua A2 , B2 , C2 vuông
góc với đường thẳng Euler của tam giác ABC.

Chứng minh.

31
Tổng quát hoá các bài hình bằng phép chia tỉ lệ

Gọi X, Y, Z lần lượt là giao điểm của AA2 , BB2 , CC2 với (O). Theo tính chất quen thuộc,
ta có X, H, Ma thẳng hàng và X thuộc (AH). Áp dụng bổ đề tâm đẳng phương cho (BC),
(AH) và (O), ta có AX, EF và BC đồng quy tại A2 , đồng thời

PA2 /(O) = A2 B · A2 C = A2 E · A2 F = PA2 /(ω) ,

với (ω) là đường tròn Euler của tam giác ABC. Từ đây, ta suy ra X nằm trên trục đẳng
phương của (O) và (ω). Tương tự, Y, Z cũng nằm trên trục đẳng phương của (O) và
(ω). Do đó X, Y, Z thẳng hàng, đồng thời đường thẳng (d3 ) đi qua X, Y, Z vuông góc với
đường thẳng Euler của tam giác ABC. 

Tính chất 2. A3 , B3 , C3 thẳng hàng, đồng thời đường thẳng (d3 ) đi qua 3 điểm này
vuông góc với đường thẳng Euler của tam giác ABC.

Chứng minh.

32
Chuyên đề Toán học Số 12

Xác định X, Y, Z tương tự như trong chứng minh tính chất 1. Xét phép nghịch đảo tâm
H, phương tích HD · HA biến Ma thành X, A3 thành A4 và BC thành (AH). Do A3
nằm trên đường thẳng BC nên A4 thuộc đường tròn (AH). Hơn nữa, ta có HA3 vuông
góc với HMa nên XA4 là đường kính của (AH).
Do đó, AXHA4 là hình chữ nhật. Gọi Ta là trung điểm của AH thì OTa là trung trực
của AX (dây cung chung) nên cũng là trung trực của HA4 , suy ra OH = OA4 .
Xác định B4 , C4 tương tự thì OH = OA4 = OB4 = OC4 nên H, A4 , B4 , C4 thuộc
(O, OH). Theo phép nghịch đảo thì A3 , B3 , C3 cùng thuộc đường thẳng (d3 ) và phép
nghịch đảo tâm H biến (O, OH) thành (d3 ) nên OH vuông góc với (d3 ). 
Trở lại bài toán. Ta có
HA0 HB0 HC0 A3 A1 B3 B1 C3 C1
k= = = = = = .
HA HB HC A3 A2 B3 B2 C3 C2
Theo định lý Thales trong hình thang A2 B2 B3 A3 thì A1 B1 , B1 C1 song song với (d2 ), (d3 ),
từ đó suy ra A1 , B1 , C1 và đường thẳng qua ba điểm đó vuông góc với đường thẳng Euler.
Lời giải hoàn tất.

Nhận xét. Ta có thể chọn các vị trí đặc biệt của A0 , B0 , C0 và giấu bằng các tính chất
của các điểm đó để tạo ra các bài toán mới. Ví dụ:

33
Tổng quát hoá các bài hình bằng phép chia tỉ lệ

Bài tập 1 (Ý tưởng từ VNTST 2021). Cho tam giác ABC có các đường cao AA0 , BB 0 ,
CC 0 đồng quy tại H và Ma , Mb , Mc lần lượt tại trung điểm BC, CA, AB. Lấy Bc , Cb theo
thứ tự là điểm đối xứng của B 0 , C 0 qua CC 0 , BB 0 . (HBc Cb ) cắt lại AA0 tại A0 . Đường
thẳng qua A0 , vuông góc với HMa cắt BC tại A1 , các điểm B1 , C1 xác định tương tự.
Chứng minh A1 , B1 , C1 thẳng hàng.

Hơn nữa, trong chứng minh bài toán trên, thay vì lấy trường hợp đặc biệt như trên, ta
có thể lấy A0 , B0 , C0 lần lượt là trung điểm HA, HB, HC. Với trường hợp này, ta dựng
ba đường thẳng da , db , dc qua H song song với BC, CA, AB cắt cặp cạnh (AB, AC) tại
(Bc , Cb ), các cặp cạnh kia tương tự thì Bc Cb qua A0 và vuông với HMa , chứng minh sử
dụng tính chất liên hợp đẳng giác trong tứ giác, từ đó có thể xử lý tiếp bằng cách sử dụng
định lý Menelaus.

2.2. Hàng điểm điều hoà

Bài 2. Cho tam giác ABC nhọn nội tiếp đường tròn (O). Một đường thẳng d bất kỳ
cắt 3 đường tròn (BOC), (COA), (AOB) lần lượt tại A0 , B 0 , C 0 . A0 , B0 , C0 là 3 điểm
nằm trên AO, BO, CO và chia các đoạn này theo tỷ lệ bằng nhau. Chứng minh rằng
A0 A0 , B0 B 0 , C0 C 0 đồng quy.

Lời giải.

Đặt k = OA
OD
. Xét các trường hợp sau đây.

Trường hợp 1. k = −1. Kẻ các đường kính AD, BE, CF của (O). Khi này, ta cần
chứng minh DA0 , EB 0 , F C 0 đồng quy.

34
Chuyên đề Toán học Số 12

Xét phép nghịch đảo z : IOA


O
2 :

A0 ↔ X(X ∈ BC) (BOC) ↔ BC


B 0 ↔ Y (Y ∈ AC) (AOC) ↔ AC
C 0 ↔ Z(Z ∈ BA) (BOA) ↔ AB
Lấy M là điểm Miquel tứ giác toàn phần BCY Z.XA thì M nằm trên (O). Khi đó
(M X, M D) ≡ (M X, M B) + (M B, M D)
≡ (ZX, ZB) + (AB, AO)
≡ (ZO, ZA) + (AZ, AO)
≡ (OX, OA) ≡ (OX, OD) (mod π).
Vì vậy (DOX) đi qua M . Tương tự thì (EOY ) và (F OZ) đi qua M .
Lại có
z :(DOX) ↔ DA0
(EOY ) ↔ EB 0
(F OZ) ↔ F C 0
M ↔M
Vì vậy DA0 , EB 0 , F C 0 đồng quy tại M .
Trường hợp 2. k = 1
Ta cần chứng minh AA0 , BB 0 , CC 0 đồng quy là xong.
Trước hết, ta chứng minh AC 0 cắt CA0 trên (ABC). Thật vậy, gọi B1 là giao điểm của
AC 0 và CA0 . A1 , C1 được xác định theo cách tương tự. Ta có
(B1 C, B1 A) ≡ (B1 A0 , B1 C 0 )
≡ (C 0 A, C 0 O) + (A0 O, A0 C)
≡ (BA, BO) + (BO, BC)
≡ (BA, BC) (mod π).
Vậy B1 ∈ (ABC). Chứng minh tương tự, ta cũng suy ra được A1 , C1 ∈ (ABC).
Bây giờ, ta chứng minh X, B1 , C1 . Thật vậy, ta có
(OB1 , OC1 ) ≡ (OB1 , OA) + (OA, OC1 )
≡ 2(CB1 , CA) + 2(BA, BA0 )
≡ 2(CA0 , CB) + 2(CB, CA) + 2(BA, BC) + 2(BC, BA0 )
≡ 2(A0 C, A0 B) + 2(AB, AC)
≡ 2(A0 C, A0 B) + 2(OB, OC)
≡ 2(A0 C, A0 B) + (A0 B, A0 C)
≡ (A0 C, A0 B) ≡ (A0 B1 , A0 C1 ) (mod π).
Vì vậy O, A0 , B1 , C1 đồng viên. Tương tự trường hợp 1, xét phép nghịch đảo z : IOA
O
2.

Phép nghịch đảo này biến A thành X, biến B1 và C1 thành chính nó nên ta suy ra được
0

X, B1 , C1 thẳng hàng. Ta có :

35
Tổng quát hoá các bài hình bằng phép chia tỉ lệ

• A0 B cắt AB 0 tại C1

• A0 C cắt AC 0 tại B1

• BC cắt C 0 B 0 tại X

Áp dụng định lý Desargues cho 4A0 BC và 4AB 0 C 0 , do X, B1 , C1 nên ta suy ra được


AA0 , BB 0 , CC 0 đồng quy tại một điểm G.

Trường hợp 3. k bất kỳ.


C0 C 0 cắt B0 B 0 tại H. Ta có

C 0 (M HGB 0 ) = C 0 (F C0 CO) = (F C0 CO) = (EB0 BO) = B 0 (EB0 BO) = B 0 (M HGC 0 ).

Suy ra M, H, G thẳng hàng. Chứng minh tương tự thì A0 A0 , B0 B 0 , C0 C 0 đồng quy trên
M G. Vậy ta có điểu phải chứng minh.

2.3. Điểm liên hợp đẳng giác

Bài 3. Cho tam giác DEF ngoại tiếp đường tròn (O). (O) tiếp xúc EF , DF , DE
theo thứ tự tại A, B, C. A1 , B1 , C1 nằm trên OD, OE, OF và lần lượt chia 3 đoạn
này theo tỉ lệ bằng nhau. Chứng minh AA1 , BB1 , CC1 đồng quy

Lời giải.

Ta sẽ trước hết giải quyết vấn đề sau.

Tính chất 3. Cho tam giác ABC nội tiếp (O) có Ma , Mb , Mc lần lượt là trung điểm
của BC, CA, AB. A0 , B0 , C0 trên OMa , OMb , OMc chia các đoạn theo tỷ lệ bằng nhau.
Chứng minh AA0 , BB0 , CC0 đồng quy trên đường thẳng Euler tam giác ABC.

36
Chuyên đề Toán học Số 12

Chứng minh. Đặt


OA0 OB0 OC0
= = = k.
OMa OMb OMc
Dựng H là trực tâm tam giác ABC. AA0 cắt OH tại X. Khi này

XH OA0 OA0 1 OA0 −1


= = =− · = k.
XO HA 2OMa 2 OMa 2

Tương tự, nếu BB0 cắt OH tại Y thì YY H O


= −1
2
k nên X ≡ Y , từ đó ta được AA0 , BB0 ,
CC0 đồng quy trên đường thẳng Euler tam giác ABC. 
Trở lại bài toán. Từ bài trên với A1 , ta lấy A0 thoả mãn

OMa · OD = OA1 · OA0 = OA2 .

Xác định tương tự với B0 ,C0 . Khi đó 4OA0 A v 4OAA1 , suy ra ∠OAA0 = ∠OA1 A =
∠HAA1 và AA0 , AA1 đẳng giác trong góc BAC. Chứng minh tương tự thì cặp đường
thẳng (BB0 , BB1 ), (CC0 , CC1 ) đẳng giác theo góc ABC, ACB.
Từ tính chất trên, AA0 , BB0 , CC0 đồng quy tại T nên AA1 , BB1 , CC1 cũng đồng quy
tại T 0 là điểm liên hợp đẳng giác của T .

Nhận xét. Do AA0 , BB0 , CC0 đồng quy trên đường thẳng Euler nên AA1 , BB1 , CC1
đồng quy trên liên hợp đẳng giác của đường thẳng Euler, tức là hyperbol Jerabek, độc
giả có thể tự tìm hiểu.

3. Bài tập
Bài tập 2 (Định lý Kariya). Cho tam giác ABC ngoại tiếp đường tròn (I). (I) tiếp xúc
BC, CA, AB tại M , N , P . X, Y , Z lần lượt nằm trên các đoạn IM , IN , IP và chia
chúng theo tỷ lệ bằng nhau. Chứng minh AX, BY , CZ đồng quy.

Bài tập 3. Cho tam giác ABC nội tiếp (O) có các đường cao AA0 , BB 0 , CC 0 đồng quy
tại H. Kẻ các đường cao A0 A0 , B 0 B0 , C 0 C0 của tam giác A0 B 0 C 0 . A1 , B1 , C1 theo thứ tự
nằm trên A0 A0 , B 0 B0 , C 0 C0 và chia chúng theo tỷ lệ bằng nhau. Chứng minh AA1 , BB1 ,
CC1 đồng quy.

Bài tập 4 (Ý tưởng từ VNTST 2020). Cho tam giác ABC nhọn ngoại tiếp đường tròn
(I), nội tiếp đường tròn (O). (I) tiếp xúc BC, CA, AB theo thứ tự tại D, E, F . Gọi L,
M , N lần lượt là trung điểm cung nhỏ BC, CA, AB. P , Q, R là 3 điểm trên các đoạn
OP , OQ, OR và chia chúng theo các tỉ lệ bằng nhau. Gọi X, Y , Z lần lượt là trung điểm
của EF , DF , DE. Chứng minh rằng đường thẳng qua A0 , song song IP ; đường thẳng
qua B 0 , song song IQ; đường thẳng qua C 0 , song song IR đồng quy, trong trường hợp :

a) A0 , B 0 , C 0 là 3 điểm nằm trên DP , EQ, F R và chia chúng theo tỉ lệ bằng nhau.

b) A0 , B 0 , C 0 là 3 điểm nằm trên DX, EY , F Z và chia chúng theo tỉ lệ bằng nhau.

37
Tổng quát hoá các bài hình bằng phép chia tỉ lệ

Bài tập 5 (Ý tưởng từ PTNK TST 2019). Cho tam giác ABC có 3 dường cao AA0 , BB 0 ,
CC 0 . Hạ AX, BY , CZ lần lượt vuông góc với B 0 C 0 , A0 C 0 , A0 B 0 . X 0 , Y 0 , Z 0 chia A0 X,
B 0 Y , C 0 Z theo tỉ lệ là k1 . A1 , B1 , C1 chia AA0 , BB 0 , CC 0 theo tỉ lệ là k2 . Chứng minh
rằng A1 X, B1 Y , C1 Z đồng quy.

Với k1 = k2 = 1
2
thì bài toán trở thành PTNK TST 2019.

Bài tập 6. Cho tam giác ABC. Đường tròn (O) qua B, C cắt AB tại D, E. X, Y , Z
lần lượt là tâm của (OBC), (OCE), (OBD). X 0 , Y 0 , Z 0 lần lượt chia OX, OY , OZ theo
tỉ lệ bằng nhau. Chứng minh rằng AX 0 , BY 0 , CZ 0 đồng quy.

Thực chất việc tổng quát hoá bài này là gợi ý cho một hướng chứng minh. Bài toán
bắt nguồn từ trường hợp X 0 , Y 0 , Z 0 theo thứ tự trùng với X, Y , Z.

Bài tập 7 (Định lý Conway). Cho tam giác ABC và một đường tròn (K) bất kỳ. Gọi
X, Y , Z lần lượt là cực của các đường thẳng BC, CA, AB đối với (K). Chứng minh AX,
BY , CZ đồng quy.

38
Giới thiệu kỳ thi OMO
Nhóm OMO PTNK

1. Thông tin về kỳ thi


1.1. Thời gian tổ chức - Đối tượng dự thi
OMO (Online Math Open) là một kỳ thi giải toán đồng đội qua mạng do một số thầy
giáo huấn luyện đội tuyển Toán quốc gia và sinh viên làm việc tại Hoa Kỳ, nhiều trong
số đó từng tham gia kì thi IMO, tổ chức từ năm 2012. Đây là kỳ thi hoàn toàn miễn phí,
và ai cũng có thể lập nhóm để tham gia. Hai lần một năm, đầu tháng 4 và cuối tháng
10, mỗi nhóm tối đa 4 người, bao gồm cả trưởng nhóm, sẽ tham gia giải các bài toán vui
có, thử thách có, trong 12 ngày. Sau khi kì thi kết thúc một ngày sẽ có kết quả, và phần
thưởng lớn nhất của các đội là . . . được đắm mình vào đam mê giải toán. Ở đợt thi mùa
thu năm 2019, có 303 đội nộp bài; đến đợt thi mùa xuân năm 2020, số đội tranh tài đã
tăng lên thành 565.

1.2. Cách thức tham gia - Một số quy định


Việc đăng ký thực hiện bởi trưởng nhóm trên trang web https://internetolympiad.
org/. Tài khoản đã được tạo ở các đợt thi trước vẫn có thể sử dụng cho các đợt thi sau.
Mặc dù bất kỳ ai cũng có thể tham gia, ban tổ chức yêu cầu các thí sinh tự giác tuân thủ
các quy định mà nổi bật hơn hẳn là:

• Chỉ được sử dụng máy tính để cộng, trừ, nhân, chia các số, mọi việc tính toán sử
dụng các phần mềm hoặc công cụ, thiết bị khác đều bị cấm.

• Chỉ được vẽ hình bằng tay.

• Không được sử dụng các tài liệu khác trong lúc làm bài.

• Không được trao đổi với các nhóm khác.

Ngoài ra, sau khi kết quả được công bố cùng lời giải chi tiết các bài toán, các đội có quyền
khiếu nại về đáp án hoặc bài làm của mình.

1.3. Cơ cấu đề thi


Thông thường, đề thi một đợt thi OMO sẽ bao gồm 30 bài toán điền đáp số (trừ đợt thi
mùa đông hai năm 2012 và 2013 có đến 50 bài), được sắp xếp theo thứ tự tăng dần độ
khó. Độ khó của các bài toán được tính theo một thang điểm chạy từ 1 đến 9.5. Các bài
toán trong đề thi sẽ có độ khó như sau :

39
Giới thiệu kỳ thi OMO

Bài Độ khó
1–5 1–3
6 – 10 3–4
11 – 15 4.5 – 5.5
16 – 20 6
21 – 25 7–8
26 – 30 8 – 9.5

Nhìn chung, độ khó các bài toán trong đề thi rất đa dạng. Độ khó 1 nghĩa là học sinh lớp
7, 8 bình thường cũng có thể làm được. Còn các bài có độ khó lớn hơn 8 thường rất khó
(khó hơn cả các bài toán trong đề thi IMO) và có thể có yếu tố toán cao cấp (ví dụ như
các bài toán về ma trận, hàm số, . . . ).
Ngoài ra, đáp số của mỗi bài toán trong đề thi là một số nguyên không âm không vượt
quá 231 − 1 = 2147483647. Vì thế, để các bài toán chỉ có một đáp số (và là số nguyên
không âm), đề thi sẽ được thiết kế phù hợp, và thường lồng vào các yếu tố tính toán đại
số, số học. Nhìn chung, các bài toán trong đề thi OMO sẽ yêu cầu tính toán rất nhiều.

2. Đề thi các năm gần đây


2.1. Mùa thu 2020
Bài 1. Một hình tròn có bán kính r có diện tích là 505. Tính diện tích của hình tròn có
độ dài đường kính là 2r.

Bài 2. Với mỗi số nguyên dương x, đặt f (x) = xx . Giả sử rằng n là một số nguyên dương
sao cho tồn tại một số nguyên dương m khác 1 thỏa mãn điều kiện f (f (f (m))) = mm .
n+2020

Tìm giá trị nhỏ nhất có thể của n.

Bài 3. Tìm số cách điền các số tự nhiên từ 1 đến 9 vào các ô trong bảng 3 × 3 thỏa các
điều kiện sau:

• Mỗi ô chứa đúng 1 số.

• Mỗi số nằm trong đúng 1 ô.

• Trong mỗi hàng, các số tăng theo chiều từ trái sang.

• Trong mỗi cột, các số tăng theo chiều trên xuống.

• Các số trong đường chéo từ chéo từ góc phải trên đến góc trái dưới tăng theo chiều
từ trên xuống.

Bài 4. Một người từ hành tinh OMO Centauri viết 10 số nguyên tố đầu tiên theo thứ
tự bất kì là U, W, XW, ZZ, V, Y, ZV, ZW, ZY và X. Trong đó, mỗi chữ cái đại diện cho
một chữ số, và các chữ số được đại diện đôi một phân biệt. Đồng thời, hệ cơ số người này
sử dụng khác hệ thập phân. Người này viết thêm một số nữa là U ZW X. Xác định số đó
(biểu diễn theo hệ thập phân).

40
Chuyên đề Toán học Số 12

Bài 5. Xác định số lượng các bộ ba số nguyên (a, b, c) với a, b, c là các số nguyên trong
khoảng từ 1 đến 12 thỏa mãn khi viết

1 1
q =a+ −
b b+ 1
c
dưới dạng phân số tối giản, tử số của q chia hết cho 13.

Bài 6. Cho x, y và z là ba số thực không âm thỏa x + y + z = 120. Tìm giá trị lớn nhất
của trung vị trong ba số 2x + y, 2y + z, 2z + x.

Bài 7. Trên một cái hồ hình vuông 9 × 9 tạo thành từ những hình vuông đơn vị, một
tảng băng tàng hình có dạng hình chữ nhật 2 × 4 hoặc 4 × 2 đồng thời tạo thành từ những
hình vuông đơn vị, có các cạnh song song với bờ hồ. Lily cố gắng bắn hạ tảng băng bằng
cách bắn các tên lửa lên hồ. Nếu Lily bắn trúng tảng băng với bất kì tên lửa nào, cô
thắng. Cô mua n tên lửa và bắn tất cả cùng lúc. Gọi N là giá trị nhỏ nhất của n sao cho
Lily có thể chắc chắn bắn trúng tảng băng và M là số cách để Lily bắn N tên lửa sao cho
đảm bảo trúng đích. Xác định 100M + N .

Bài 8. Cho λ là một số thực thoả mãn điều kiện nếu ABCD là một tứ giác lồi nội tiếp
bất kì có AC = 4, BD = 5, AB ⊥ CD thì diện tích của ABCD không nhỏ hơn λ. Khi
m
này, giá trị lớn nhất có thể của λ là , với m và n là các số nguyên dương nguyên tố
n
cùng nhau. Xác định 100m + n.

Bài 9. Hong và Song mỗi người có một bộ bài có 8 lá bài, 4 đen và 4 đỏ. Mỗi lượt, mỗi
người chơi đặt xuống 2 lá trên cùng của bộ bài của họ. Xác suất để Hong và Song tung
m
ra cùng cặp (2 đỏ, 2 đen hay 1 đỏ 1 đen) trong cả 4 lượt là với m, n là các số nguyên
n
dương nguyên tố cùng nhau. Tính 100m + n.

Bài 10. Cho w, x, y, z là các số phức khác 0, và n là một số nguyên dương thỏa các điều
kiện sau:
1 1 1 1
• + + + = 3,
w x y z
• wx + wy + wz + xy + xz + yz = 14,

• (w + x)3 + (w + y)3 + (w + z)3 + (x + y)3 + (x + z)3 + (y + z)3 = 2160,



• w + x + y + z + i n ∈ R.

Xác định n.

Bài 11. Cho tam giác ABC thỏa AB = 5, AC = 8, ∠BAC = 60◦ . P là một điểm trong
tam giác thỏa ∠AP B = ∠BP C = ∠CP A. Các đường thẳng BP và AC cắt nhau tại E,
CP và AB cắt nhau tại F . Đường tròn ngoại tiếp của các tam giác BP F và CP E cắt
m
nhau tại điểm Q 6= P . Khi đó QE + QF = , với m, n là các số nguyên dương nguyên
n
tố cùng nhau. Tính 100m + n.

41
Giới thiệu kỳ thi OMO

Bài 12. Ở một buổi tiệc có 100 con mèo. Mỗi cặp mèo tung đồng xu 1 lần, và chúng bắt
tay nếu và chỉ nếu đồng xu ra mặt ngửa. Biết rằng có 4900 cặp mèo bắt tay. Sau buổi
tiệc, mỗi con mèo được gán cho một chỉ số hạnh phúc trong khoảng [0, 1]. Ta gọi một
con mèo là tốt nếu chỉ số hạnh phúc của nó lớn hơn tất cả các con mèo mà nó bắt tay.
m
Giá trị kì vọng của số mèo tốt là với m, n là các số nguyên dương nguyên tố cùng
n
nhau. Tính 100m + n.
Bài 13. Cho a, b, c, x, y, z là các số nguyên dương thỏa mãn

a2 − 2 b2 − 37 c2 − 41
= = = a + b + c.
x y z
Đặt S = a + b + c + x + y + z. Tính tổng tất cả các trị có thể của S.
Bài 14. Cho hình chữ nhật BCB 0 C 0 , M là trung điểm B 0 C 0 , A là một điểm nằm trên
đường tròn ngoại tiếp của hình chữ nhật. Gọi H là trực tâm tam giác ABC, T là chân
đường cao hạ từ H xuống AM . Biết rằng AM = 2, [ABC] = 2020, BC = 10. Khi đó
m
AT = , với m, n là các số nguyên dương nguyên tố cùng nhau. Tính 100m + n.
n
Bài 15. Cho m, n là các số nguyên dương nguyên tố cùng nhau và
2020
2020
!
m
(−1) k
cos(2020 cos−1 ( 2020 )) = .
X
k

k=0 k n
Ta viết lại n dưới dạng tích của một bộ các số nguyên tố (không nhất thiết đôi một phân
biệt). Tính tổng các số trong bộ số đó. Ví dụ, nếu n = 12 = 2 × 2 × 3 thì đáp số là
2 + 2 + 3 = 7.
Bài 16. Cho trước số nguyên dương n, ta gọi dãy số a1 , a2 , . . . an với ai ∈ {1, 2, . . . , n}
với mọi i là n–tốt nếu với mọi ước nguyên dương d của n và mỗi số nguyên không âm k
n
bé hơn ta có
d

(k+1)d
X
d
ai .
i=kd+1

Đặt χ(n) là xác suất dãy a1 , a2 , . . . , an với ai được chọn bất kỳ trong tập 1, 2, . . . , n là
dãy n–tốt. Tìm tổng của tất cả giá trị của n thỏa mãn: tồn tại số nguyên dương m không
chia hết cho 3 sao cho 340 χ(n) = m1 .
Bài 17. Cho tam giác ABC có AB = 11, BC = 12, CA = 13, gọi M, N lần lượt là trung
điểm của các cạnh CA, AB, đường tròn nội tiếp tam giác ABC lần lượt tiếp xúc CA,√AB
k m
tại X, Y . Gọi R, S, T lần lượt là trung điểm M N, BX.CY . Khi đó sin ∠SRT = ,
n
với k, m, n là các số nguyên dương thỏa k, n nguyên tố cùng nhau và m là số square–free.
Tính 100k + 10m + n.
Bài 18. Có một hàng người dài vô tận được đánh số 1, 2, 3, . . . . Sau đó, mỗi người nói
"Karl" hoặc "Lark". Đặt S là tập các số nguyên dương i thỏa mãn những người thứ
m
i, i + 1, i + 2 đều nói "Karl", và đặt X = 2 thì giá trị của X 2 là , với m, n là các
P −i
i∈S n
số nguyên dương nguyên tố cùng nhau. Tính 100m + n.

42
Chuyên đề Toán học Số 12

Bài 19. Tìm số nguyên dương M nhỏ nhất sao cho tồn tại số nguyên dương n thỏa mãn:
• M là tổng bình phương của n số nguyên dương liên tiếp nào đó.
• 2M là tổng bình phương của 2n số nguyên dương liên tiếp.
Bài 20. Cho một chuỗi gồm ít nhất một phần tử, trong đó mỗi phần tử là A hoặc B,
Kathryn có thể thực hiện các biến đổi sau:
• Chọn một chữ A, xóa đi và thay BB vào vị trí đó.
• Chọn một chữ B, xóa đi và thay AA vào vị trí đó.
Kathryn bắt đầu với chuỗi "A". Đặt an là số chuỗi độ dài n Kathryn có thể tạo được qua
∞ an m
một loạt các phép biến đổi. Khi đó = với m, n là các số nguyên dương nguyên
P
n=1 5
n n
tố cùng nhau. Tính 100m + n.
Bài 21. Trong tất cả các hình elip có tâm nằm tại gốc tọa độ, có duy nhất một elip tiếp
xúc với với đồ thị của đường cong x3 − √ 6x2 y + 3xy 2 + y 3 + 9x2 − 9xy + 9y 2 = 0 tại 3
kπ m
điểm phân biệt. Diện tích elip này là với k, m, n là các số nguyên dương sao cho
n
(k, n) = 1 và m là số square–free. Tính 100k + 10m + n.
Bài 22. Ba điểm P1 , P2 , P3 và 3 đường thẳng l1 , l2 , l3 nằm trên một mặt phẳng sao cho
không có điểm nào trong ba điểm nằm trên đường thẳng nào trong ba đường. Với hai
số nguyên i, j không nhất thiết phân biệt nằm từ 1 đến 3, ta gọi một đường thẳng l là
(i, j)–tốt nếu ảnh đối xứng của Pi qua l nằm trên lj , và gọi nó là rất tốt nếu có hai cặp
(i1 , j1 ) và (i2 , j2 ) mà nó đều tốt. Tìm số lớn nhất các đường thẳng rất tốt có thể có.
Bài 23. Với một số nguyên dương k > 1 với (k, 2020) = 1, ta nói một số nguyên dương
N là k–tệ nếu không tồn tại các số nguyên không âm x, y sao cho N = 2020x + ky. Tìm
tổng của tất cả số nguyên dương k thỏa k > 1, (k, 2020) = 1 và điều kiện sau: nếu m, n
là các số nguyên dương với m + n = 2019(k − 1), m ≥ n và m là k–tệ thì n là k–tệ.
Bài 24. Trong lý thuyết đồ thị, một tam giác là một tập hợp ba đỉnh sao cho hai điểm
bất kì trong chúng được nối bởi một cạnh. Với một số nguyên n ≥ 3, nếu một đồ thị n
đỉnh không chứa hai tam giác không có đỉnh chung, đặt f (n) là số tam giác tối đa có thể
có. Tính f (3) + f (4) + · · · + f (100).
Bài 25. Cho n là một số nguyên dương với chính xác 12 ước nguyên dương 1 = d1 <
· · · < d12 = n. Ta nói n tầm thường nếu

5 + d6 (d6 + d4 ) = d7 d4 .
Tính tổng hai số nguyên dương tầm thường nhỏ nhất.
Bài 26. Cho dãy hàm hai biến f0 , f1 , f2 , f3 , . . . được xác định bới f0 (x, y) = 0 và
fn+1 (x, y) = x + |y + fn (x, y)| với mọi số nguyên n không âm. Với x0 , y0 ∈ [−2, 2] được

chọn ngẫu nhiên, đặt pn là xác suất fn (x0 , y0 ) < 1. Đặt a, b, c, d là các số nguyên dương
π2 + a
sao cho giới hạn của dãy số p1 , p3 , p5 , . . . là và giới hạn của dãy số p2 , p4 , p6 , . . . là
b
π +c
2
. Tính 1000a + 100b + 10c + d.
d
43
Giới thiệu kỳ thi OMO

Bài 27. Cho ABC là một tam giác không cân, không vuông. Gọi ω là đường tròn nội
tiếp và σ là đường tròn chín điểm của 4ABC với tâm lần lượt là I và N . Giả sử ω và
σ tiếp xúc với nhau tại F . D là chân đường cao đỉnh A và M là trung điểm cạnh BC.
Tiếp tuyến chung của ω và σ tại F cắt AB, AC lần lượt tại P và Q. Cho DP và DQ lần
lượt cắt lại σ tại P1 và Q1 . Giả sử tồn tại điểm Z trên cạnh P1 Q1 sao cho ∠M F Z =√90◦
k m
và M Z ⊥ DF . Biết rằng σ có bán kính 11 và ω có bán kính là 5. Khi đó DI =
n
với k, m, n là các số nguyên dương thỏa mãn (k, n) = 1 và m là số square–free. Tính
100k + 10m + n.
Bài 28. Julia làm một chiếc bánh với hình dạng một hình vuông đơn vị. Mỗi phút, Julia
cắt hai nhát lên cái bánh như sau:
• Cô chọn một phần hình vuông S không có vết cắt nào bên trong.

• Cô cắt cái bánh theo hai đường qua tâm S song song với cạnh bánh.
Trong suốt quá trình, cô không di chuyển miếng bánh. Sau tám phút, cô có một lưới gồm
92 = 81 miếng bánh (các miếng có thể có kích thước khác nhau. Tìm số lưới phân biệt có
thể tạo thành, hai lưới giống nhau nếu có cùng tập những vết cắt; đặc biệt, hai lưới nhận
được từ nhau qua phép quay hay đối xứng là hai lưới phân biệt.
Bài 29. Cho tam giác không cân ABC. Cho I0 = A và với mỗi số nguyên dương t, It
là tâm nội tiếp của tam giác It−1 BC. Giả sử I0 , I1 , . . . đều nằm trên một hyperbol H
với hai đường tiệm cận l1 , l2 . Đường thẳng qua A vuông góc BC cắt l1 , l2 lần lượt tại
12
P, Q. Giả sử AC 2 = AB 2 + 1. Khi đó diện tích nhỏ nhất có thể của tứ giác BP CQ là
√ √ 7
j k+l m
với j, k, l, m, n thỏa (j, l, n) = 1, k, m là các số square–free, và j > l. Tính
n
10000j + 1000k + 100l + 10m + n.
Bài 30. Giả sử F là một miền với đúng 514 phần tử. Ta nói một hàm f : F → F là vui
nếu với mọi x, y ∈ F ,

(f (x + y) + f (x)) (f (x − y) + f (x)) = f (y 2 ) − f (x2 ).


Tìm số phần tử z của F sao cho tồn tại hai hàm vui phân biệt h1 và h2 sao cho h1 (z) =
h2 (z).

2.2. Mùa xuân 2020


Bài 1. Cho l là một đường thẳng và A, B, C là ba điểm nằm trên l sao cho AB = 7 và
BC = 5. Gọi m là đường thẳng đi qua A và vuông góc với l. Tìm giá trị nhỏ nhất của
P B + P C.
Bài 2. Po viết năm số nguyên liên tiếp và sau đó xóa đi một số trong chúng. Tổng của
bốn số còn lại là 153. Hãy tìm số mà Po đã xóa đi.
Bài 3. Biết rằng câu trả lời cho bài toán này có thể biểu diễn được dưới dạng abc, trong
đó a, b và c là các số nguyên dương đôi một nguyên tố cùng nhau, đồng thời b = 10, tính
1000a + 100b + 10c.

44
Chuyên đề Toán học Số 12

Bài 4. Cho ABCD là hình vuông có độ dài cạnh là 16 và tâm O. Cho S là một nửa
đường tròn với đường kính AB và nằm ngoài hình vuông ABCD. Đồng thời, gọi P là
một điểm nằm trên S sao cho OP = 12. Tính diện tích của tam giác CDP .

Bài 5. Tìm số nguyên dương n nhỏ nhất không thể biểu diễn được dưới dạng n = x3 +3y 3 ,
trong đó x, y là các số nguyên.

Bài 6. Alexis có 2020 bức tranh, bức tranh thứ i có dạng một hình chữ nhật 1 × i với
mỗi i = 1, 2, . . . , 2020. Tìm số nguyên dương n nhỏ nhất sao cho Alexis có thể đặt tất cả
các bức tranh đó lên một cái bàn hình vuông kích thước n × n sao cho các bức tranh này
không chồng lên nhau và không nằm ngoài cái bàn đó.

Bài 7. Ta đặt hai chiếc xe ô tô một cách ngẫu nhiên lên lưới ô vuông 5 × 5, mỗi chiếc xe
chiếm đúng một ô vuông và hướng về một trong bốn hướng cơ bản. Biết rằng hai chiếc xe
này không được đặt tại cùng một ô vuông. Tại mỗi bước di chuyển, ta có thể chọn một
chiếc xe ô tô và di chuyển chiếc xe ô tô đó về phía trước 1 ô theo hướng của nó. Xác suất
để tồn tại một số hữu hạn các bước di chuyển để hai xe ô tô chiếm cùng một ô vuông là
m
, trong đó m, n là hai số nguyên dương nguyên tố cùng nhau. Tính 100m + n.
n
a! + 1
Bài 8. Cho a > b là hai số nguyên dương. Tìm giá trị nguyên nhỏ nhất có thể của .
b! + 1
Bài 9. Một ảo thuật gia có một cái nón chứa a con thỏ trắng và b con thỏ đen bên trong.
Ảo thuật gia này liên tục lấy ra các cặp thỏ một cách ngẫu nhiên. Ta gọi một cặp thỏ là
cặp ca rô nếu như trong cặp này có đúng một con thỏ trắng và một con thỏ đen. Biết
rằng ảo thuật gia lấy ra hết tất cả các con thỏ từ cái nón và không có con thỏ nào bị lẻ,
đồng thời giá trị kỳ vọng của số cặp ca rô là 2020. Hãy tính số cặp (a, b) có thể.

Bài 10. Hỏi có tổng cộng bao nhiêu hàm số f : {1, 2, . . . , 15} → {1, 2, . . . , 15} sao cho
với mọi số x ∈ {1, 2, . . . , 15}, biểu thức
f (f (x)) − 2f (x) + x
15
có giá trị là một số nguyên ?

Bài 11. Một tủ sách làm bằng gỗ gụ có bốn cuốn sách trông giống hệt nhau và có lần
lượt 200, 400, 600 và 800 trang. Velma lấy ra một cách ngẫu nhiên một quyển sách từ
tủ sách đó, lật hú họa một trang để đọc, và sau đó để quyển sách lại vị trí cũ trên tủ
sách. Sau đó, Daphne cũng lấy ra một cách ngẫu nhiên một quyển sách từ tủ sách đó,
lật hú họa một trang để đọc. Biết rằng Velma đọc trang 122 của quyển sách cô ấy lấy
xuống và Daphne đọc trang thứ 304 của quyển sách Daphne chọn. Xác suất để cả hai
m
chọn cùng một quyển sách là , với m, n là hai số nguyên dương nguyên tố cùng nhau.
n
Tính 100m + n.

Bài 12. Cho ngũ giác lồi ABCDE nội tiếp đường tròn γ. Biết rằng AB = 14, BE =
10, BC = CD = DE và SABCDE = 3SACD . Khi này, có hai giá trị có thể cho bán kính

của γ. Tổng của hai giá trị này là n, với n là số nguyên dương. Tính n.

Bài 13. Với p, q, r là các số nguyên không âm, đặt

45
Giới thiệu kỳ thi OMO

f (p, q, r) = (p!)p (q!)q (r!)r .

Tìm số nguyên dương n nhỏ nhất sao cho với mọi bộ ba số nguyên không âm (a, b, c) và
(x, y, z) thỏa mãn điều kiện a + b + c = 2020 và x + y + z = n, f (x, y, z) chia hết cho
f (a, b, c).

Bài 14. Gọi S và T là các tập hợp hữu hạn, khác rỗng bao gồm các số nguyên dương.
b
Ta nói số tự nhiên a là tốt đối với số tự nhiên b nếu như a ≥ + 7. Ta nói một cặp số
2
có thứ tự (a, b) ∈ S × T (nghĩa là a thuộc S còn b thuộc T ) là có thể thỏa mãn nếu
như a và b đôi một tốt đối với với nhau. Một tập con R của S được gọi là không chấp
nhận được nếu như có ít hơn |R| phần tử b của T sao cho (a, b) là có thể thỏa mãn với
a ∈ R nào đó. Giả sử không tồn tại tập con không chấp nhận được của S, hơn nữa
S chứa các số 14, 20, 16, 32, 23 và 31, hãy xác định giá trị nhỏ nhất của tổng các phần tử
của T , biết rằng |T | ≥ 20.

Bài 15. Cho tam giác ABC với AB = 20 và AC = 22. Giả sử đường tròn nội tiếp của
tam giác này tiếp xúc BC, CA, và AB lần lượt tại D, E và F , và P là chân đường vuông
góc hạ từ D xuống EF . Nếu ∠BP C = 90◦ , tính BC 2 .

Bài 16. Hỏi có bao nhiêu cặp số nguyên dương có thứ tự (m, n) sao cho (2m − 1)(2n − 1)
là ước của 210! − 1 ?

Bài 17. Tính số số nguyên 1 ≤ n ≤ 1024 sao cho dãy số dne, dn/2e, dn/4e, dn/8e, . . .
không chứa bất kỳ bội số nào của 5.

Bài 18. Vincent có một con xúc xắc cân bằng với các mặt được đánh số từ 1 đến 6. Ban
đầu, anh ta sẽ thả con xúc xắc và ghi chú kết quả thu được vào một mảnh giấy. Sau đó, cứ
mỗi giây trôi qua, anh ta lại thả con xúc xắc đó. Nếu kết quả thu được của Vincent trong
một lượt nào đó khác với kết quả anh ta thu được trong lượt trước đó, anh ta sẽ ghi chú
kết quả đó và tiếp tục thả trong những lượt tiếp theo. Nếu kết quả thu được bằng đúng
với kết quả thu được trong lượt ngay trước đó thì anh ta sẽ dừng lại, không ghi chú kết
quả của lần thảy cuối cùng và tính trung bình cộng của các kết quả trước đó. Biết rằng
trong lượt đầu tiên, Vincent thu được kết quả là 1, và E là giá trị kỳ vọng của kết quả mà
anh ta thu được. Khi này, tồn tại các số hữu tỉ dương r, s, t sao cho E = r − s · ln(t), hơn
m
nữa t không phải là lũy thừa (mũ lớn hơn 1) của một số hữu tỉ nào. Nếu r + s + t = ,
n
trong đó m, n là các số nguyên dương nguyên tố cùng nhau, tính 100m + n.

Bài 19. Cho ABC là một tam giác không cân. Đường tròn nội tiếp tam giác này tiếp
xúc BC, AC và AB lần lượt tại D, E và F , và đường tròn bàng tiếp góc A của tam giác
tiếp xúc các đường thẳng BC, AC và AB lần lượt tại D1 , E1 và F1 . Giả sử các đường
thẳng AD, BE và CF đồng quy tại G, và các đường thẳng AD1 , BE1 và CF1 đồng quy
tại G1 . Đường thẳng GG1 cắt đường phân giác trong góc ∠BAC tại√ X. Giả sử AX = 1,
√ √ j + k m
cos ∠BAC = 3 − 1 và BC = 8 4 3. Khi này, AB · AC = , với j, k, m, n là
n
các số nguyên dương sao cho gcd(j, k, n) = 1 và m không chia hết cho bất kỳ số chính
phương nào lớn hơn 1. Tính 1000j + 100k + 10m + n.

46
Chuyên đề Toán học Số 12

Bài 20. Reimu phát minh một hệ đếm mà ở đó, mỗi số trong hệ có đúng 5 chữ số.
Số 0 trong hệ đếm này sẽ được biểu diễn là 00000, hơn nữa với mỗi số tự nhiên n,
Reimu sẽ thu được biểu diễn của số n + 1 bằng cách xác định chữ số bên trái ngoài cùng
trong dãy các chữ số bằng với chữ số bên phải ngoài cùng của n, sau đó tăng chữ số
đó lên một đơn vị, còn các chữ số bên phải chữ số đó thì được gán giá trị 0 (các chữ
số còn lại được giữ nguyên). Hệ đếm tương tự cho 3 chữ số sẽ được bắt đầu với các số
000, 100, 110, 111, 200, 210, 211, 220, 221, 222, 300, . . . Hãy xác định biểu diễn thập phân
của số mà trong hệ đếm của Reimu được biểu diễn là 98765.

Bài 21. Với số nguyên dương i = 2, 3, . . . , 2020, đặt



3i2 + 2i − 1
ai = .
i3 − i
1
Cho x2 , . . ., x2020 là các số thực dương sao cho x42 +x43 +· · ·+x42020 = 1− .
1010 · 2020 · 2021
Gọi S là giá trị lớn nhất có thể của

ai xi ( ai − 2−2.25 xi )
P2020
i=2

và gọi m là số nguyên dương nhỏ nhất sao cho S m là một số hữu tỉ. Khi S m được biểu diễn
dưới dạng phân số tối giản, mẫu số của nó có thể được biểu diễn dưới dạng pα1 1 pα2 2 · · · pαk k
với p1 < · · · < pk là các số nguyên tố và αi là các số nguyên dương với i = 1, k. Tính
p1 α1 + p2 α2 + · · · + pk αk .

Bài 22. Cho ABC là một tam giác không cân với tâm đường tròn nội tiếp I và K là giao
điểm của ba đường đối trung trong tam giác. Ngoài ra, giả sử BC = 1099. Gọi P là một
điểm nằm trên mặt phẳng chứa tam giác ABC, và gọi D, E, F là chân các đường cao hạ
từ P lần lượt xuống các đường thẳng BC, CA, AB. Gọi M và N lần lượt là trung điểm
của các đoạn thẳng EF và BC. Giả sử các bộ ba điểm (M, A, N ) và (K, I, D) đều thẳng
hàng, và diện tích của tam giác DEF lớn gấp 2020 lần diện tích tam giác ABC. Tính giá
trị lớn nhất có thể của dAB + ACe.

Bài 23. Trong ngân hàng Shower, một người khách đang chán đặt n đồng xu thành một
hàng. Cứ mỗi giây, người khách lại thực hiện Quy Trình. Trong Quy Trình, các đồng
xu nằm cạnh đúng 1 đồng xu đang ở mặt ngửa trước Quy Trình sẽ được đặt ngửa lên (ở
đúng vị trí của nó) và các đồng xu còn lại sẽ được đặt úp xuống.
Hàm f được định nghĩa như sau: Nếu tồn tại một trạng thái khởi đầu của các đồng xu
sao cho người khách không dừng thì f (n) = 0. Ngược lại, f (n) là số giây trung bình để
Quy Trình kết thúc tính trên tất cả các trạng thái ban đầu. Biết rằng khi n = 2k − 1 với
k nguyên dương thì f (n) > 0.
Gọi N là số nguyên dương nhỏ nhất mà

M = 2N (f (22 − 1) + f (23 − 1) + f (24 − 1) + · · · + f (210 − 1))

là số nguyên dương. Giả sử M = bk bk−1 . . . b0 trong hệ nhị phân, tính N + b0 + b1 + · · · + bk .

Bài 24. Cho A, B là hai đỉnh đối nhau của một hình vuông đơn vị với đường tròn ngoại
tiếp Γ. Gọi C là một điểm di động trên Γ. Nếu C 6∈ {A, B}, gọi ω là đường tròn nội tiếp

47
Giới thiệu kỳ thi OMO

tam giác ABC, và gọi I là tâm của đường tròn ω. Gọi C1 là điểm tiếp xúc của đường
thẳng AB với đường tròn ω, và gọi D là điểm đối xứng của C1 qua đường thẳng CI. Nếu
C ∈ {A, B}, đặt D = C. Khi C di chuyển trên Γ, D di chuyển trên một đường cong C
tạo thành một vùng có diện tích là A. Tính b104 Ac.

Bài 25. Đặt S là tập các dãy số nguyên dương (có ít nhất hai số hạng) có tổng bằng
2019. Với một dãy a1 , a2 , . . . , ak , giá trị của nó được định nghĩa là:
a
aa2 aa3 . . . akk+1
V (a1 , a2 , . . . , ak ) = 1 2 .
a1 !a2 ! . . . ak !
m
Tổng các giá trị của tất cả các dãy thuộc S là với m, n nguyên tố cùng nhau. Tính số
n
dư khi chia m + n cho 1000.

Bài 26. Cho ABC là một tam giác với đường tròn ngoại tiếp ω và tâm đường tròn
ngoại tiếp O. Giả sử AB = 15, AC = 14, và P là một điểm nằm trong 4ABC sao
cho AP = 13 2
, BP 2 = 409
4
, và P gần AC hơn AB. Gọi E, F là giao điểm của BP , CP
với ω (E 6= B, F 6= C), và gọi Q là giao điểm của EF với tiếp tuyến của ω tại A. Giả
a √
sử tứ giác AQOP nội tiếp và CP 2 có thể được biểu diễn dưới dạng − c d sao cho
b
gcd(a, b) = 1 và d không chia hết cho bình phương của bất kỳ số nguyên tố nào. Tính
1000a + 100b + 10c + d.

Bài 27. Một xích đạo đại số được định nghĩa là sự trang bị ba phép toán nhị phân \,
], [ trên tập hợp các số thực sao cho với mọi x, y ∈ R, ta có

x \ y = x + y, x ] y = max{x, y}, x [ y = min{x, y}.

Một biểu diễn xích đạo thông qua ba biến số x, y, z, cùng với độ phức của các biểu
diễn này, được định nghĩa theo các quy tắc sau:

• x, y và z là các biểu diễn xích đạo với độ phức bằng 0

• Khi P và Q là các biểu diễn xích đạo với độ phức lần lượt là p và q, các biểu diễn
P \ Q, P ] Q, P [ Q đều có độ phức bằng 1 + p + q

Hãy tính số hàm số khác nhau f : R3 → R (R3 là tập hợp các bộ số (x, y, z), trong đó
x, y, z ∈ R) có thể được biểu diễn dưới dạng biểu diễn xích đạo với độ phức không vượt
quá 3.

Bài 28. Cho A0 BC0 D là một tứ giác lồi nội tiếp trong một đường tròn, gọi là ω. Với mỗi
số nguyên i ≥ 0, gọi Pi là giao điểm của hai đường thẳng Ai B và Ci D, Qi là giao điểm của
hai đường thẳng Ai D và BCi , Mi là trung điểm của đoạn thẳng Pi Qi . Các đường thẳng
Mi Ai và Mi Ci lần lượt cắt lại ω tại Ai+1 và Ci+1 . Đường tròn ngoại tiếp của 4A3 M3 C3
và 4A4 M4 C4 cắt nhau tại hai điểm U và V .
Nếu A0√B = 3, BC0 = 4, C0 D = 6, DA0 = 7, khi này U V có thể được biểu diễn dưới
a b
dạng , trong đó a, b, c là các số nguyên dương sao cho a, c nguyên tố cùng nhau và b
c
là một số square–free. Tính 100a + 10b + c.

48
Chuyên đề Toán học Số 12

Bài 29. Cho x0 , x1 , . . ., x1368 là các số phức. Với mỗi số nguyên m, gọi d(m), r(m) là
các số nguyên duy nhất sao cho 0 ≤ r(m) < 37 và m = 37d(m) + r(m). Cho ma trận kích
thước 1369 × 1369 A = {ai,j }0≤i,j≤1368 được xác định như sau:



 x37d(j)+d(i) r(i) = r(j), i 6= j

d(i) = d(j), i 6= j

−x37r(i)+r(j)


ai,j = .



 x38d(i) − x38r(i) i=j

0 trong các trường hợp còn lại

Ta nói ma trận A là r–murine nếu tồn tại một ma trận kích thước 1369 × 1369 M sao
cho r cột của M A − I1369 gồm toàn số 0, trong đó I1369 là ma trận đơn vị kích thước
1369 × 1369 (là ma trận ở đó các số ở đường chéo chính bằng 1, các số còn lại bằng 0).
Gọi rk(A) là giá trị lớn nhất của r sao cho A là ma trận r–murine. Gọi S là tập hợp tất
cả các giá trị có thể của rk(A) khi các số {xi } thay đổi. Tính tổng của 15 số nhỏ nhất
trong S.

Bài 30. Gọi c là số thực dương nhỏ nhất sao cho với mọi số nguyên dương n và số thực
dương x1 , . . ., xn , ta có bất đẳng thức
(n3 + k 3 − k 2 n)3/2 √ i3 (4n − 3i + 100)
!
3 + cn5 + 100n4 .
Pn Pn
k=0 q ≤ i=1
x21 + · · · + x2k + xk+1 + · · · + xn xi

Tính b2020cc.

3. Các bài toán chọn lọc từ kỳ thi


3.1. Đại số
Bài 1 (OMO 2015). Cho các số thực dương a,b,c,d thỏa mãn
56
a2 + b2 − c2 − d2 = 0 và a2 − b2 − c2 + d2 = (bc + ad).
53
ab + cd
Tìm giá trị lớn nhất của .
bc + ad
Bài 2 (OMO 2013). Tính giá trị tuyệt đối của tổng sau:

b2013 sin 0◦ c + b2013 sin 1◦ c + · · · + b2013 sin 359◦ c


trong đó bxc là số nguyên lớn nhất không vượt quá x.
(Có thể sử dụng tính chất sin n◦ là số vô tỷ với n là số nguyên dương không chia hết cho
30)

Bài 3 (OMO 2018). Tìm số nguyên dương n nhỏ nhất sao cho đa thức

xn − xn−1 − xn−2 − · · · − x − 1
nhận một nghiệm lớn hơn 1.999.

49
Giới thiệu kỳ thi OMO

Bài 4 (OMO 2017). Cho dãy đa thức Pn (x) thỏa mãn

P0 (x) = 1, P1 (x) = x, Pn+1 (x) = 2xPn (x) − Pn−1 (x).


! !
i i S
Gọi S = 0
P2017 và T = 0
P17

(trong đó i là đơn vị ảo, hay i2 + 1 = 0). Biết là
2 2

T
m
một số hữu tỉ có phần lẻ có thể viết dưới dạng với m,n là hai số nguyên dương nguyên
n
tố cùng nhau. Tìm m.

Bài 5 (OMO 2015). Cho các số thực x1 , x2 , . . . , x42 thỏa mãn

5xi+1 − xi − 3xi xi+1 = 1


với mọi i = 1, 2, . . . , 42 và x1 = x43
Tìm tất cả giá trị có thể của x1 + x2 + · · · + x43 .

Bài 6 (OMO 2019). Cho a1 , a2 , . . . , a5 là các số thực thỏa mãn



a
1 a2 + a2 a3 + a3 a4 + a4 a5 + a5 a1 = 20
a
1 a3 + a2 a4 + a3 a5 + a4 a1 + a5 a2 = 22.

Tìm giá trị nhỏ nhất của a21 + a22 + a23 + a24 + a25 .

Bài 7 (OMO 2019). Cho n là một số nguyên dương và P (x) là một đa thức monic bậc n
hệ số thực bất kì. Đặt Q(x) = (x + 1)2 (x + 2)2 . . . (x + n + 1)2 . Xét mn là giá trị nhỏ nhất
X i2 P (i2 )2
n+1
của . Khi đó tồn tại các hằng số dương a, b, c sao cho khi n tiến về dương vô
i=1 Q(i)
cùng, tỉ số giữa mn và a2n n2n+b c tiến về 1. Tìm các hằng số này.

3.2. Tổ hợp
Bài 1 (OMO 2018). Có bao nhiêu cách để xóa 24 ký tự từ chuỗi "OMOMO. . . OMO"
(gồm 27 ký tự), sao cho ba ký tự còn lại là O, M, O theo đúng thứ tự đó? Thứ tự xóa
các ký tự là không quan trọng.

Bài 2 (OMO 2019). Cho số nguyên dương k. Sơn và Tường chơi một trò chơi trên môt
lưới ô vuông vô hạn. Ban đầu, có một ô vuông đen, các ô còn lại đều trắng.
Một lượt trong trò chơi gồm các bước sau. Sơn đi trước, và chọn một ô đen kế với nhiều
hơn hai ô trắng. (Hai ô được gọi là kề nhau nếu chúng có chung cạnh.) Ô được chọn sẽ
trở thành màu trắng còn các ô còn kề nó sẽ trở thành mau đen nếu chúng đang trắng.
Sau đó, Tường thực hiện k lần hành động sau: chọn hai ô kề nhau và hoán đổi màu của
chúng (Tường được phép đổi màu hai ô cùng màu, dù làm vậy thì không thay đổi gì).
Như thế là một lượt. Nếu sau một lượt mà Sơn không thể chọn một ô đen kề nhiều hơn
hai ô trắng thì Tường thắng, ngược lại, lượt tiếp theo diễn ra.
Gọi m là giá trị nhỏ nhất của k sao cho Tường chắc chắn có chiến thuật thắng. Với k = m,
gọi t là số nguyên dương nhỏ nhất sao cho Tường chắc chắn thắng sau t lượt. Tìm m và
t.

50
Chuyên đề Toán học Số 12

Bài 3 (OMO 2017). Trên trục số thực, ta đánh dấu các điểm 1, 2, 3, . . . , 11. Một con bọ
bắt đầu ở số 1 và nhảy đến 10 điểm còn lại theo một thứ tự nào đó sao cho nó không đến
điểm nào hai lần rồi quay lại điểm 1. Tìm khoảng cách tối đa con bọ đã nhảy, và số cách
nhảy để đạt được khoảng cách đó.

Bài 4 (OMO 2018). 10 người được đánh số 1, 2, . . . , 10 đang chơi một trò chơi vào giáng
sinh. Ông già Noel sẽ thăm nhà từng người theo các qui tắc sau:

• Ông già Noel thăm người chơi 1 đầu tiên. Sau khi thăm người chơi i ông sẽ thăm
người chơi i + 1 với người chơi 11 chính là người chơi 1.

• Mỗi khi thăm một người, ông già Noel đưa một món quà hoặc một cục than cho
người đó (nhưng không đưa cả hai).

• Tại mọi thời điểm, số món quà và số cục than ông già Noel đã đưa chênh lệch nhau
không vượt quá 3.

• Khi ông già Noel có thể chọn giữa quà và than, ông chọn chúng với xác suất như
nhau.

Tính xác suất người chơi 1 nhận một món quà trước người chơi 2.

Bài 5 (OMO 2016). An và Bình chơi một trò chơi trên bảng ô vuông 10 × 10. Đầu tiên,
An chọn ngẫu nhiên một tập S có 100 phần tử từ tập {1, 2, . . . , 2019}. Rồi Bình viết mỗi
số trong 100 số đó vào một ô của bảng 10 × 10. Sau đó, Bình dựng một hình khối từ
bảng 10 × 10 như sau: với mỗi ô vuông đơn vị, Bình xếp n khối 1 × 1 × 1 thành một cột
trên ô đó, với n là số được điền vào ô đó. Gọi B là diện tích bề mặt lớn nhất có thế của
hình khối trong các cách Bình điền 100 số vào bảng. Tính giá trị kỳ vọng của B.

Bài 6 (OMO 2015). Đặt V0 = ∅ là tập rỗng và Vn+1 là tập gồm 2|Vn | tập con của Vn với
n = 0, 1, . . . . Chẳng hạn:

V2 = {∅, {∅}} và V3 = {∅, {∅}, {{∅}}, {∅, {∅}}}

Một tập x ∈ V5 được gọi là bắc cầu nếu mỗi phần tử của x là một tập con của x. Hỏi có
bao nhiêu tập bắc cầu?

Bài 7 (OMO 2020). Trong những ngày TpHCM giãn cách xã hội, do ở nhà chán quá
không có việc gì để làm, Hy xếp n đồng xu thành một hàng. Cứ mỗi giờ trôi qua, Hy lại
thực hiện thuật toán sau đây, gọi là thuật toán alpha: những đồng xu nào nằm cạnh
đúng 1 đồng xu ngửa trước khi thực hiện thuật toán alpha sẽ được đặt ngửa lên (ở
đúng vị trí của nó), còn các đồng xu còn lại sẽ được đặt úp xuống.
Tìm tất cả n nguyên dương sao cho thuật toán alpha luôn kết thúc sau hữu hạn bước.
Với những n đó, tính số giây trung bình để thuật toán alpha kết thúc trên tất cả các
trạng thái ban đầu.

3.3. Hình học


Bài 1 (OMO 2018). Số giao điểm phân biệt giữa tứ giác và ngũ giác không tự cắt nhiều
nhất là bao nhiêu ?

51
Giới thiệu kỳ thi OMO

Bài 2 (OMO 2015). Cho tam giác ABC có số đo góc là các số nguyên dương (theo độ).
Dựng các hình vuông ABDE và ACF G bên ngoài tam giác. Giả sử D, E, F, G cùng thuộc
một đường tròn, bộ ba (∠A, ∠B, ∠C) có thể nhận bao nhiêu giá trị ?

Bài 3 (OMO 2014). Cho tam giác ABC nội tiếp (O). Các điểm D, E, F lần lượt điểm
chính giữa cung BC, CA, AB không chứa A, B, C và G đối xứng với D qua O. BG cắt
DF tại K, CG cắt DE tại L. Chứng minh rằng hình chiếu của D trên KL là tâm nội
tiếp tam giác ABC.

Bài 4 (OMO 2017). Cho tập S = {(x, y) | −1 ≤ xy ≤ 1}. Tìm giá trị lớn nhất của diện
tích tam giác 4 có ba đỉnh thuộc S.

Bài 5 (OMO 2020). Cho đường tròn Ω và dây cung AB. Điểm C thuộc đoạn AB, đường
thẳng ` qua C cắt Ω tại D, E. Các điểm F, G lần lượt là giao điểm của các cặp đường
thẳng (AD, BE), (AE, BD). M là trung điểm của F G. Chứng minh rằng đường tròn
ngoại tiếp tam giác M DE luôn đi qua hai điểm cố định khi ` thay đổi qua C.

Bài 6 (OMO 2019). Cho tam giác ABC nhọn không cân có đường cao BE, CF cắt nhau
tại H. Đường thẳng qua C vuông góc với CH cắt BH tại K, đường thẳng qua B vuông
góc với BH cắt CH tại L. Gọi I là hình chiếu của H trên KL. Đường thẳng AH và EF
cắt nhau tại D.

(a) Chứng minh rằng DI đi qua trung điểm M của BC.

(b) N là trung điểm AH, M N cắt HB, HC lần lượt tại P, Q. O là tâm đường tròn ngoại
tiếp tam giác HP Q. Chứng minh rằng I, M, O thẳng hàng.

Bài 7 (OMO 2016). Cho tam giác ABC nhọn không cân có M, N lần lượt là trung điểm
AB, AC và ` là đường thẳng Euler. Gọi E, F lần lượt là hình chiếu của B, C trên `. Đường
thẳng đối xứng với ` qua BC cắt AE, AF lần lượt tại P, Q. BP và CQ cắt nhau tại K.

(a) Chứng minh điểm liên hợp đẳng giác với K trong tam giác ABC (gọi là L) nằm trên
`.

(b) Các điểm X, Y, Z lần lượt là hình chiếu của K trên trung trực BC, CA, AB. Các
điểm R, S lần lượt là trung điểm XY và XZ. Gọi T là giao điểm của M R và N S.
Chứng minh rằng T L đi qua trọng tâm của tam giác ABC.

3.4. Số học
Bài 1 (OMO 2017). Một số nguyên dương được gọi là k–tốt (với k là một số nguyên
dương nào đó) nếu như với mọi cách thay đổi các chữ số của số đó sao cho có ít nhất một
chữ số được giữ nguyên, kết quả thu được (sau khi loại bỏ các chữ số 0 đầu tiên) là một
số tự nhiên không chia hết cho k.

(a) Tìm số nguyên dương k nhỏ nhất sao cho tồn tại một số nguyên dương có 4 chữ số
là số k–tốt ?

(b) Với n là một số nguyên dương tùy ý, hãy tìm số tự nhiên k nhỏ nhất sao cho tồn tại
một số nguyên dương có n chữ số là số k–tốt.

52
Chuyên đề Toán học Số 12

Bài 2 (OMO 2019). Gọi S là tập hợp tất cả các số nguyên dương có dạng 10n + 1000,
trong đó n là một số tự nhiên. Tìm số nguyên dương lớn nhất không thể biểu diễn được
dưới dạng tổng của một số phần tử (không nhất thiết phân biệt) của S.

Bài 3 (OMO 2020). Với p, q, r là các số nguyên không âm, đặt

f (p, q, r) = (p!)p (q!)q (r!)r .

Tìm số nguyên dương n nhỏ nhất sao cho với mọi bộ ba số nguyên không âm (a, b, c) và
(x, y, z) thỏa mãn điều kiện a + b + c = 2020 và x + y + z = n, f (x, y, z) chia hết cho
f (a, b, c).

Bài 4 (OMO 2015). Cho p là một số nguyên tố lẻ nhỏ hơn 105 . Hoàng và Danh chơi
một trò chơi. Ban đầu, Hoàng chọn một số c ∈ {2, 3, . . . , p − 1}. Sau đó, Danh chọn 2 số
nguyên d và x, đặt f (t) = ct + d, rồi ghi x lên một mảnh giấy.
Tiếp theo, Hoàng ghi f (x) lên mảnh giấy đó, rồi Danh lại ghi f (f (x)), Hoàng ghi
f (f (f (x))), và cứ tiếp tục như vậy, . . . Trò chơi kết thúc khi có 2 số nào đó trên tờ
giấy có hiệu là một số chia hết cho p, và người ghi số cuối cùng là người chiến thắng.
Tìm tất cả các số p sao cho Danh là người có chiến thuật để thắng trò chơi.

Bài 5 (OMO 2019). Cho dãy số (an ) được xác định như sau : a0 = 2019 và an = a2019 n−1
với mọi số nguyên dương n. Tìm số dư thu được khi chia a0 + a1 + · · · + a51 cho 856.

Bài 6 (OMO 2016). Kí hiệu N là tập hợp tất cả các số nguyên dương. Cho f : N → N là
một hàm số sao cho các điều kiện sau được đồng thời thỏa mãn :

• Với mỗi n ∈ N, ta có f (n)|n2016

• Với mỗi a, b, c ∈ N sao cho a2 + b2 = c2 , ta có f (a)f (b) = f (c)

Hãy tính số lượng giá trị có thể của f (2014) + f (2) − f (2016), xét trên tất cả các hàm số
f thỏa mãn các điều kiện trên.

Bài 7 (OMO 2014). Tồn tại duy nhất một số nguyên tố p sao cho biểu diễn thập phân
1
của tuần hoàn theo chu kì là 294. Biết rằng p > 1050 , tìm số dư thu được khi chia p
p
cho 109 .

53
Lời giải đề thi chọn đội dự tuyển các năm
Ban biên tập
Giới thiệu. Cuối học kỳ I, các bạn học sinh lớp 10 tham gia kỳ thi chọn đội
dự tuyển. Như tên gọi, kỳ thi chọn ra 15 bạn để học các chuyên đề nâng cao
vào học kỳ II và hè để tạo nguồn cho đội tuyển. Ngoài ra, ba bạn điểm cao nhất
trong kỳ thi sẽ được chọn thi Olympic 30/4. Đề thi gồm 4 câu, thời gian làm
bài là 2 tiếng. Do đối tượng của kỳ thi là học sinh lớp 10, chưa học quá nhiều
kiến thức nên có một số câu chỉ cần nắm vững kiến thức lớp 9 có thể làm được.

1. Đề thi
1.1. Năm 2018
Bài 1. Tìm tất cả hàm số f : R → R sao cho với mọi số thực x và y,

f (3f (x) + 2y) = 10x + f (2f (y) − x).

Bài 2. Cho tam giác ABC nhọn không cân, các điểm D, E thay đổi trên cạnh BC sao cho
∠BAD = ∠CAE và D nằm giữa B, E. Đường tròn ngoại tiếp các tam giác ABD, ACE
cắt nhau tại M khác A.
(a) Chứng minh rằng phân giác góc DM E luôn đi qua một điểm cố định.

(b) Gọi I và K lần lượt là tâm đường tròn nội tiếp tam giác ABM và ACM. Chứng
minh rằng đường thẳng IK luôn đi qua một điểm cố định.
Bài 3. Cho số nguyên n ≥ 3 và 2n số dương x1 , x2 , . . . , xn , y1 , y2 , . . . , yn thỏa mãn đồng
thời các điều kiện
(i) 0 < x1 y1 < x2 y2 < · · · < xn yn .

(ii) x1 + x2 + · · · + xk ≥ y1 + y2 + · · · + yk với mọi k ∈ {1, 2, . . . , n}.


1 1 1 1 1 1
Chứng minh rằng + + ··· + ≤ + + ··· + .
x1 x2 xn y1 y2 yn
Bài 4. Cho S là tập hợp khác rỗng. Gọi P(S) là tập hợp tất cả các tập con của S. Giả
sử f : P(S) → P(S) là ánh xạ có tính chất sau: với mọi X, Y ∈ P(S), nếu X ⊂ Y thì
f (X) ⊂ f (Y ). Chứng minh rằng tồn tại T ∈ P(S) sao cho f (T ) = T.

1.2. Năm 2019


Bài 1. Tìm tất cả các hàm f : R → R thỏa mãn các điều kiện

f (−x) = −f (x) và f (f (x) − y) = 2x + f (f (y) + x) với mọi x, y ∈ R.

Bài 2. Tìm tất cả các bộ số tự nhiên (a, b, c) sao cho a2 + 2b + c, b2 + 2c + a, c2 + 2a + b


đều là các số chính phương.

54
Chuyên đề Toán học Số 12

Bài 3. Cho tập hợp X = {1, 2, . . . , 396}. Gọi S1 , S2 , . . . , Sk là các tập con khác nhau của
X thỏa mãn đồng thời hai điều kiện:

(i) |S1 | = |S2 | = · · · = |Sk | = 198.

(ii) |Si ∩ Sj | ≤ 99 với mọi i, j ∈ {1, 2, . . . , k} mà i 6= j.

Chứng minh rằng k ≤ 650 .

Bài 4. Cho tam giác ABC nhọn. Đường tròn thay đổi qua B, C cắt các cạnh AB, AC
lần lượt tại D, E.

(a) Gọi H, K lần lượt là hình chiếu của B trên CD và DE. Chứng minh rằng HK luôn
đi qua một điểm cố định.

(b) Gọi Q là hình chiếu của C trên DE. Đường tròn ngoại tiếp tam giác BDK cắt BC
tại M , đường tròn ngoại tiếp tam giác CEQ cắt BC tại N . KM cắt QN tại X.
Chứng minh rằng X thuộc một đường thẳng cố định.

1.3. Năm 2020


Bài 1. Tìm giá trị nhỏ nhất của biểu thức

a4 + b 4 + 2
P = với a, b ∈ R.
(a2 − a + 1)(b2 − b + 1)

Bài 2. Tìm tất cả các hàm số f : Q+ → Q+ sao cho với mọi số hữu tỉ dương x và y,

f (x2 f (y)2 ) = f (x)2 f (y).

Bài 3. Cho x1 , x2 , x3 . . . là dãy số nguyên thỏa mãn đồng thời hai điều kiện

1 = x1 < x2 < x3 < . . . và xn+1 6 2n với n = 1, 2, 3, . . .

Chứng minh với mọi số nguyên dương k, tồn tại các số nguyên i > j sao cho xi − xj = k.

Bài 4. Cho tam giác ABC cân tại A, nội tiếp đường tròn (O). Gọi M là điểm trên cạnh
−−→ 1 −→
AB sao cho AM = AB. Đường tròn tâm M bán kính M B cắt (O) tại điểm thứ hai là D.
3
−−→ 2 −→
Một đường thẳng qua M song song với AD cắt AC tại N . Chứng minh rằng AN = AC.
3

1.4. Năm 2021


Bài 1. Cho các số thực không âm a, b, c thoả a2 + b2 + c2 = 1. Tìm giá trị lớn nhất của
7 √ √ √
P = a + (1 − a)( a + b + c) + a2 b2 c2 .
2
Bài 2. Tìm tất cả các hàm số f : R → R sao cho với mọi số thực x và y,

f (x − f (y)) = 4f (x) + 3x + f (y).

55
Lời giải đề thi chọn đội dự tuyển các năm

Bài 3. Cho số nguyên dương n và A là tập gồm 8n + 1 số tự nhiên sao cho nếu m ∈ A
thì (m, 6) = 1 và m < 30n. Chứng minh rằng tồn tại hai số phần biệt a, b ∈ A mà a | b.
Bài 4. Cho điểm M di động trên đường thẳng d cố định và O là điểm cố định nằm ngoài
đường thẳng d. Gọi A là hình chiếu của O trên d, H là hình chiếu của A trên OM , D là
trung điểm HM và K là hình chiếu của H trên AD. Chứng minh rằng HK luôn đi qua
một điểm cố định khi M di động trên d.

1.5. Năm 2022


Bài 1. Cho các số thực không âm a, b, c thoả a2 + b2 + c2 = 1. Tìm giá trị lớn nhất và
giá trị nhỏ nhất của
P = ab + bc + ca − 2(a + b + c).
Bài 2. Cho các số nguyên dương k, n thỏa k < n. Tính số đơn ánh f : {1, 2, . . . , 2k +1} →
{1, 2, . . . , 2n} thoả mãn f (1) < · · · < f (k) < f (k + 1) > f (k + 2) > · · · > f (2k + 1) và
f (k + 1) 6= 2n − 2.
Bài 3. Với mỗi số nguyên dương n, đặt a(n) = 1 + 2 + · · · + n và b(n) = 1 + 2 + · · · + n2 .
Hỏi có tồn tại số n sao cho 2(n + 1)a(n) + 3b(n) − 1 là số chính phương?
Bài 4. Cho tam giác ABC có 2∠A = 5∠B = 10∠C. Phân giác trong BD cắt trung tuyến
CM tại I. Một đường thẳng d đi qua D vuông góc với AC cắt BC và AI lần lượt tại E
và K. AE cắt CK tại F . Chứng minh M F song song BK.

2. Lời giải
2.1. Năm 2018

Bài 1. Tìm tất cả hàm số f : R → R sao cho với mọi số thực x và y,

f (3f (x) + 2y) = 10x + f (2f (y) − x).

2f (x)
Lời giải. Thay y = − , ta có
3
2f (x)
! !
f (0) = 10x + f f − −x .
3
nên f toàn ánh. Giả sử tồn tại a, b ∈ R sao cho f (a) = f (b), do f toàn ánh nên tồn tại
a+b
c ∈ R sao cho f (c) = . Thay (x, y) lần lượt bởi (a, c) và (b, c) vào phương trình hàm
2
ban đầu, ta nhận được

f (3f (a) + 2c) = 10a + f (2f (c) − a) = 10a + f (b)


f (3f (b) + 2c) = 10b + f (2f (c) − b) = 10b + f (a).

56
Chuyên đề Toán học Số 12

Suy ra a = b dẫn tới f đơn ánh. Thay x = 0, ta thu được


3f (0)
f (3f (0) + 2y) = f (2f (y)) ⇒ f (y) = y + , ∀y ∈ R.
2
Thay lại phương trình hàm ban đầu, ta được

3f (x) + 2y = 10x + 2f (y) − x, ∀x, y ∈ R.

Thay y bởi x, ta được f (x) = 7x, mâu thuẫn. Tóm lại, không tồn tại hàm số f thỏa mãn.

Nhận xét. Có thể chứng minh f đơn ánh như sau : nếu f (a) = f (b), thay y bởi a, b
vào phương trình hàm ban đầu kết hợp với f toàn ánh để suy ra f tuần hoàn với chu kỳ
T = |a − b| mà do hàm 10x không tuần hoàn nên T = 0. Một số bài toán tương tự :

Bài tập 1 (IMO SL 2002). Tìm tất cả hàm số f : R → R thỏa mãn

f (f (x) + y) = 2x + f (f (y) − x), ∀x, y ∈ R.

Bài tập 2 (Trường Đông 2016). Tìm tất cả hàm số f : R → R thỏa mãn

f (f (x) + 2y) = 10x + f (f (y) − 3x), ∀x, y ∈ R.

Bài tập 3. Tìm tất cả số thực α sao cho tồn tại hàm số f : R → R thỏa mãn

f (3f (x) + 2y) = 10x + f (2f (y) + αx), ∀x, y ∈ R.

Bài 2. Cho tam giác ABC nhọn không cân, các điểm D, E thay đổi trên cạnh BC
sao cho ∠BAD = ∠CAE và D nằm giữa B, E. Đường tròn ngoại tiếp các tam giác
ABD, ACE cắt nhau tại M khác A.

(a) Chứng minh rằng phân giác góc DM E luôn đi qua một điểm cố định.

(b) Gọi I và K lần lượt là tâm đường tròn nội tiếp tam giác ABM và ACM. Chứng
minh rằng đường thẳng IK luôn đi qua một điểm cố định.

Lời giải. (a) Dựng điểm N sao cho tam giác ABM và ACN đồng dạng cùng hướng.
Điều này kéo theo tam giác AM N và tam giác ABC đồng dạng, khi đó

∠CM N = ∠AM C − ∠AM N = ∠AEC − ∠ABC = ∠BAE,

∠CN M = ∠AN C − ∠AN M = ∠AM B − ∠ACB = ∠ADB − ∠ACB = ∠CAD.


Từ giả thiết, ta suy ra ∠BAE = ∠CAD hay ∠CM N = ∠CN M , tức CM = CN. Suy ra
MB MB AB
= = nên nếu gọi X là chân đường phân giác góc A của tam giác ABC
MC NC AC
thì M X là phân giác ∠BM C. Ngoài ra,

∠BM D = ∠BAD = ∠CAE = ∠CM E

nên ∠DM E và ∠BM C có chung phân giác hay phân giác ∠DM E luôn đi qua X.

57
Lời giải đề thi chọn đội dự tuyển các năm

E C

BM CM
(b) Do = nên phân giác ∠ABM, ∠ACM cắt nhau tại J nằm trên AM . Nếu
BA CA
gọi Y là chân đường phân giác ngoài góc A, tam giác JBC có
Y B IJ KC AB AJ AC
· · = · · =1
Y C IB KJ AC AB AJ
nên theo định lý Menelaus đảo, I, K, Y thẳng hàng hay IK luôn đi qua Y.
Nhận xét. Ngoài cách tiếp cận trên, ta có thể dùng nhiều cách khác cho câu a như dùng
tính chất đẳng giác, phép nghịch đảo, . . . Nguồn gốc bài toán có lẽ lấy từ một bài hình
trong đề IMO 1996.
Bài tập 1 (IMO 1996). Cho P là điểm nằm trong tam giác ABC sao cho
∠AP B − ∠ACB = ∠AP C − ∠ABC.
Các điểm D, E lần lượt là tâm nội tiếp của các tam giác AP B, AP C. Chứng minh rằng
ba đường thẳng AP, BD, CE đồng quy.

Bài 3. Cho số nguyên n ≥ 3 và 2n số dương x1 , x2 , . . . , xn , y1 , y2 , . . . , yn thỏa mãn


đồng thời các điều kiện

(i) 0 < x1 y1 < x2 y2 < · · · < xn yn .

(ii) x1 + x2 + · · · + xk ≥ y1 + y2 + · · · + yk với mọi k ∈ {1, 2, . . . , n}.


1 1 1 1 1 1
Chứng minh rằng + + ··· + ≤ + + ··· + .
x 1 x2 xn y1 y2 yn

Lời giải. Với x1 , . . . , xn , y1 , . . . , yn thỏa đồng thời các điều kiện trên, ta chứng minh
n
X 1 X n
1 (y1 + · · · + yn ) − (x1 + · · · + xn )
− ≤
i=1 xi i=1 yi xn y n
bằng quy nạp theo n.
Với n = 1, mệnh đề hiển nhiên đúng. Giả sử mệnh đề đúng với n = k, khi đó
k+1
1 k+1
X 1 (y1 + · · · + yk ) − (x1 + · · · + xk ) yk+1 − xk+1
+
X
− ≤
i=1 xi i=1 yi xk y k xk+1 yk+1
(y1 + · · · + yk + yk+1 ) − (x1 + · · · + xk + xk+1 )

xk+1 yk+1
nên mệnh đề quy nạp đúng. Từ đây, ta suy ra đpcm.

58
Chuyên đề Toán học Số 12

Nhận xét. Cách tiếp cận trên dựa vào việc ghép cặp các số hạng ở hai vế và thử sử
dụng điều kiện (i), trình bày theo cách quy nạp để lời giải rõ ràng chặt chẽ hơn. Một cách
tiếp cận khác có thể kể đến là khai triển Abel, đây là một công cụ hiệu quả đối với các
bài toán mà các tổng được so sánh với nhau. Bài toán này có nguồn gốc từ đề vòng quốc
gia của Romania năm 1999.

Bài tập 1 (Romania 1999).

(a) Cho các số thực dương x1 , . . . , xn , y1 , . . . , yn thỏa mãn đồng thời các điều kiện

(i) x1 y1 < x2 y2 < · · · < xn yn .


(ii) x1 + x2 + · · · + xk ≥ y1 + y2 + · · · + yk với mọi k ∈ {1, 2, . . . , n}.
1 1 1 1 1 1
Chứng minh rằng + + ··· + ≤ + + ··· + .
x1 x2 xn y1 y2 yn
(b) Cho A là tập con hữu hạn khác rỗng của N∗ thỏa mãn với hai tập con B, C phân
biệt bất kỳ của A thì
x 6=
X X
x.
x∈B x∈C

Chứng minh rằng


1
< 2.
X

x∈A x

Bài 4. Cho S là tập hợp khác rỗng. Gọi P(S) là tập hợp tất cả các tập con của S.
Giả sử f : P(S) → P(S) là ánh xạ có tính chất sau: với mọi X, Y ∈ P(S), nếu X ⊂ Y
thì f (X) ⊂ f (Y ). Chứng minh rằng tồn tại T ∈ P(S) sao cho f (T ) = T.

Lời giải. Xét họ F = {X ∈ P(S) | f (X) ⊂ X} là một họ khác rỗng do có chứa S.


Chọn T = X, với mọi X ∈ F thì T ⊂ X nên f (T ) ⊂ f (X) ⊂ X, như vậy, ta thu
\

X∈F
được f (T ) ⊂ T. Ngoài ra ta cũng có f (f (T )) ⊂ f (T ) nên f (T ) ∈ F, điều này dẫn tới
T ⊂ f (T ). Kết hợp các điều trên, ta được f (T ) = T.

Nhận xét. Ý tưởng của bài toán dựa vào trường hợp S là tập hữu hạn, khi đó f (S) ⊂ S,
thực hiện ánh xạ f nhiều lần, sẽ có thời điểm số phần tử không giảm được (do số phần
tử của f (X) không vượt quá X trong quá trình đó), khi đó chọn T là tập thu được ở thời
điểm đó. Trong trường hợp vô hạn, thuật toán trên phải được điều chỉnh bằng cách lấy
giao của các tập X mà f (X) ⊂ X (tương tự trường hợp hữu hạn). Một số bài tương tự:

Bài tập 1. Cho hàm f : [0, 1] → [0, 1] đơn điệu. Chứng minh rằng f có điểm bất động,
tức tồn tại x ∈ [0, 1] sao cho f (x) = x.

Bài tập 2. Cho S là tập hợp khác rỗng. Gọi P(S) là tập hợp tất cả các tập con của S.
Giả sử f, g : P(S) → P(S) là các ánh xạ có tính chất: với mọi X, Y ∈ P(S), nếu X ⊂ Y
thì f (X) ⊂ f (Y ) và g(X) ⊂ g(Y ). Chứng minh tồn tại A, B ∈ P(S) sao cho f (A) = S \ B
và g(B) = S \ A.

59
Lời giải đề thi chọn đội dự tuyển các năm

2.2. Năm 2019

Bài 1. Tìm tất cả các hàm f : R → R thỏa mãn

f (−x) = −f (x) và f (f (x) − y) = 2x + f (f (y) + x) với mọi x, y ∈ R.

Lời giải. Do f là hàm lẻ nên f (0) = 0. Thay lần lượt x = 0, y = 0 vào điều kiện thứ hai,
ta thu được, với mọi x, y ∈ R,

f (−y) = f (f (y)) và f (f (x)) = 2x + f (x).

Do đó,

f (−x) = f (f (x)) = 2x + f (x) ⇒ f (x) = −x.

Thử lại, f (x) = −x là hàm số thỏa mãn yêu cầu đề bài.

Nhận xét. Đây là một bài phương trình hàm cơ bản, có thể giải được bằng các phép thế
đơn giản. Ngoài ra nếu bỏ điều kiện hàm lẻ thì bài toán vẫn có thể giải quyết được.

Bài tập 1. Tìm tất cả các hàm f : R → R thỏa mãn các điều kiện

f (f (x) − y) = 2x + f (f (y) + x), ∀x, y ∈ R.

Bài 2. Tìm tất cả các bộ số tự nhiên (a, b, c) để a2 + 2b + c, b2 + 2c + a, c2 + 2a + b đều


là các số chính phương.

Lời giải. Không mất tính tổng quát, giả sử (a, b, c) là bộ cần tìm và a = min{a, b, c}.

Trường hợp 1. a = 0. Khi đó 2b + c, b2 + 2c và c2 + b là các số chính phương. Nếu b ≥ c


thì
b2 ≤ b2 + 2c ≤ b2 + 2b < (b + 1)2 .
Do đó b2 + 2c = b2 hay c = 0. Khi đó, b và 2b là các số chính phương, điều này dẫn tới
b = 0. Tương tự, nếu b ≤ c thì ta cũng được nghiệm (0, 0, 0).

Trường hợp 2. a > 0.

• Nếu b ≥ c, khi này do b2 + 2c + a là số chính phương và

b2 < b2 + 2c + a ≤ b2 + 3b < (b + 2)2

nên b2 + 2c + a = (b + 1)2 hay 2c + a = 2b + 1. Ta suy ra b < 2c+a


2
< 32 c, kéo theo

3
c2 < c2 + 2a + b < c2 + 2c + c < (c + 2)2
2
nên c2 + 2a + b = (c + 1)2 hay 2a + b = 2c + 1.

60
Chuyên đề Toán học Số 12

• Nếu c ≥ b, tương tự ta cũng có c2 < c2 + 2a + b < (c + 2)2 nên 2a + b = 2c + 1.


Suy ra c < 23 b kéo theo b2 < b2 + 2c + a < (b + 2)2 nên 2c + a = 2b + 1.

Tóm lại, b2 + 2c + a và c2 + 2a + b là số chính phương tương đương với hệ

5a − 3
 
2b + 1 = 2c + a c =

⇐⇒  2 .
2c + 1 = 2a + b b = 3a − 2

Do a lẻ nên đặt a = 2k + 1 (k ∈ N) thì b = 6k + 1, c = 5k + 1. Ngoài ra,

a2 + 2b + c = (2k + 1)2 + 2(6k + 1) + (5k + 1) = 4k 2 + 21k + 4

là số chính phương. Kiểm tra trực tiếp với k ∈ {0, 1, 2}, ta thấy giá trị thỏa mãn là k = 1,
nên ta được nghiệm (1, 1, 1). Nếu k ≥ 3, ta có

(2k + 4)2 < 4k 2 + 21k + 4 < (2k + 6)2 ⇒ 4k 2 + 21k + 4 = (2k + 5)2 ⇐⇒ k = 21.

Thay vào ta được nghiệm (43, 127, 106).

Tóm lại, các bộ số cần tìm là

(a, b, c) ∈ {(0, 0, 0), (1, 1, 1), (43, 127, 106), (127, 106, 43), (106, 43, 127)}.

Nhận xét. Đây là một bài số học nhẹ nhàng, ý tưởng kẹp giữa hai số chính phương cũng
không mới đối với học sinh cấp 2. Tuy nhiên, bài toán cũng có bẫy ở chỗ: vai trò của
ba số a, b, c không như nhau. Bài toán được lấy trực tiếp từ ELMO Shortlist 2013, chú ý
rằng nếu yêu cầu là tìm các số nguyên a, b, c thì vẫn có thể giải được.

Bài tập 1. Tìm tất cả các bộ số nguyên (a, b, c) để a2 + 2b + c, b2 + 2c + a, c2 + 2a + b


đều là các số chính phương.

Bài 3. Cho tập hợp X = {1, 2, . . . , 396}. Gọi S1 , S2 , . . . , Sk là các tập con khác nhau
của X thỏa mãn đồng thời hai điều kiện:

(i) |S1 | = |S2 | = · · · = |Sk | = 198.

(ii) |Si ∩ Sj | ≤ 99 với mọi i, j ∈ {1, 2, . . . , k} mà i 6= j.

Chứng minh rằng k ≤ 650 .

Lời giải. Ta đếm số cặp (A, i), trong đó A là một tập con 100 phần tử của X, i là số
nguyên dương không vượt quá k và A ⊂ Si bằng hai cách :

• Đếm theo A: Có tổng cộng C396 100


cách chọn A. Hơn nữa, từ (ii), có tối đa một chỉ số
i ∈ {1, 2, . . . , k} sao cho A ⊂ Si . Do đó, số cặp không vượt quá C396100
.

• Đếm theo i: Có k cách chọn i và C198


100
cách chọn A nên số cặp là kC198
100
.

61
Lời giải đề thi chọn đội dự tuyển các năm

Từ đây, ta suy ra được kC198


100 100
≤ C396 . Viết lại, ta được
100
C396 49
(i + 297)(396 − i)
k ≤ 100 =
Y
.
C198 i=0 (i + 99)(198 − i)

Với mỗi i ∈ {0, 1, . . . , 49}, ta lại có

(i + 297)(396 − i) − 6(i + 99)(198 − i) = 5i2 − 495i = 5i(i − 99) ≤ 0.

Suy ra
49
(i + 297)(396 − i)
≤ 650 .
Y
k≤
i=0 (i + 99)(198 − i)

Nhận xét. Đây là một bài toán đẹp, cần sự tinh tế trong bước đánh giá, số 50 và lũy
thừa trong yêu cầu bài toán cũng là một gợi ý đến vai trò số 100. Bài toán có nguồn gốc
từ đề SMO 2006.
Bài tập 1 (SMO 2006). Cho tập hợp X = {1, 2, . . . , 4n}. Gọi S1 , S2 , . . . , Sk là các tập
con khác nhau của X thỏa mãn đồng thời hai điều kiện:
(i) |S1 | = |S2 | = · · · = |Sk | = 2n.

(ii) |Si ∩ Sj | ≤ n với mọi i, j ∈ {1, 2, . . . , k} mà i 6= j.


n+1
Chứng minh rằng k ≤ 6 2 .
Bài tập 2. Thầy Vũ giao 10 bài tập cho các bạn học sinh lớp 10 Toán, mỗi bạn chọn 4
bài để giải và nộp cho thầy. Thầy Vũ thấy rằng hai bạn bất kỳ chỉ nộp giống nhau tối đa
2 bài. Chứng minh rằng lớp 10 Toán có không quá 30 học sinh.
Bài tập 3. Cho X = {1, 2, . . . , n} và S1 , S2 , . . . , St là các tập con của X thỏa mãn đồng
thời hai điều kiện
(i) |S1 | = |S2 | = · · · = |St | = k,

(ii) Với mọi tập con A có m phần tử của X thì tồn tại i sao cho Si ⊂ A.
Cnk
Chứng minh rằng t ≥ k
.
Cm

Bài 4. Cho tam giác ABC nhọn. Đường tròn thay đổi qua B, C cắt các cạnh AB, AC
lần lượt tại D, E.

(a) Gọi H, K lần lượt là hình chiếu của B trên CD và DE. Chứng minh rằng HK
luôn đi qua một điểm cố định.

(b) Gọi Q là hình chiếu của C trên DE. Đường tròn ngoại tiếp tam giác BDK cắt
BC tại M , đường tròn ngoại tiếp tam giác CEQ cắt BC tại N . KM cắt QN tại
X. Chứng minh rằng X thuộc một đường thẳng cố định.

62
Chuyên đề Toán học Số 12

Lời giải. (a) HK là đường thẳng Simson ứng với điểm B trong tam giác CDE nên luôn
đi qua chân đường cao hạ từ B lên CA.

(b) ∠XKQ = ∠ABC và ∠XQK = ∠ACB (do các tứ giác BKDM và CQEN nội tiếp)
nên 4XKQ v 4ABC. Ngoài ra, ∠ADE = ∠ACB, do đó DE song song với một đường
−−→
thẳng cố định. Hệ quả là BK, CQ là các đường thẳng cố định và vector KQ không đổi.
−−→
Từ 4XKQ v 4ABC, ta suy ra v = KX không đổi. Vậy X ∈ ` với ` là ảnh của đường
thẳng BK qua phép tịnh tiến theo vector v. 

Nhận xét. Ý a của bài toán nhẹ nhàng, nhắc lại về đường thẳng Simson. Khó khăn chủ
yếu ở ý b nằm ở việc nhận ra đường cố định. Tùy vào cách phát hiện ra đường cố định
mà ta còn có các cách khác, chẳng hạn dựng ra các vị trí tổng quát và quy về chứng minh
thẳng hàng, đồng quy. Ngoài ra, ta có thể cho đường tròn qua B, C trở thành đường tròn
ngoại tiếp tam giác ABC, khi đó ta biết được ` đi qua chân đường cao của A lên BC.

Bài tập 1. Cho tứ giác ABCD nội tiếp đường ω. Giả sử đường tròn đường kính AB cắt
BC và AD tại E, F , đường tròn đường kính CD cắt BC và AD tại G, H. Đường thẳng
EF cắt GH tại K. Chứng minh rằng đường thẳng qua K vuông góc với BC luôn đi qua
một điểm cố định.

2.3. Năm 2020

Bài 1. Tìm giá trị nhỏ nhất của biểu thức

a4 + b 4 + 2
P = với a, b ∈ R.
(a2 − a + 1)(b2 − b + 1)

63
Lời giải đề thi chọn đội dự tuyển các năm

2
Lời giải. Với mọi x ∈ R, ta có bất đẳng thức x4 + 1 ≥ (x2 − x + 1)2 . Thật vậy,
9
2 1
x4 + 1 − (x2 − x + 1)2 = (x + 1)2 (7x2 − 10x + 7) ≥ 0.
9 9
Sử dụng bất đẳng thức trên và bất đẳng thức AM-GM,
2 (a2 − a + 1)2 + (b2 − b + 1)2 4
P ≥ . ≥ .
9 (a2 − a + 1)(b2 − b + 1) 9

Dấu bằng xảy ra khi a = b = −1.

Bài 2. Tìm tất cả các hàm số f : Q+ → Q+ sao cho với mọi số hữu tỉ dương x và y,

f (x2 f (y)2 ) = f (x)2 f (y).

Lời giải. Giả sử f là hàm số thỏa mãn

f (x2 f (y)2 ) = f (x)2 f (y), ∀x, y ∈ Q+ . (1)

Thay x bởi f (x) vào (1), ta được

f (f (x)2 f (y)2 ) = f (f (x))2 f (y), ∀x, y ∈ Q+ . (2)

Từ (2), ta suy ra

f (f (x))2 f (y) = f (f (x)2 f (y)2 ) = f (f (y))2 f (x), ∀x, y ∈ Q+ . (3)


f (f (1))2
Thay y bởi 1 vào (3), đặt k = f (1)
, ta được

f (f (1))2
f (f (x))2 = f (x) = kf (x), ∀x ∈ Q+ . (4)
f (1)
Với x ∈ Q+ , xét (xn ) xác định bởi x1 = f (x), xn+1 = f (xn ) với n ∈ N∗ , từ (4), ta được

x2n+1 = kxn , ∀n ∈ N∗ . (5)

Chia k 2 cho hai vế trong công thức truy hồi, ta được, với mọi n ∈ N∗ ,
2 4 2n−1
x1 x2 x3 xn
  
= = = ··· = .
k k k k
Điều này dẫn tới trong phân tích chuẩn tắc của xk1 , các mũ đều là bội của 2n−1 với mọi
n nên chỉ các mũ này chỉ có thể bằng 0. Suy ra x1 = k nên ta được f (x) = k, ∀x ∈ Q+ .
Thử lại, ta được k 3 = k hay k = 1. Vậy các hàm số cần tìm là f (x) = 1.

Nhận xét. Đây là bài toán tương đối kĩ thuật với học sinh lớp 10, việc thực hiện phép
thế x → f (x) vào đẳng thức (4) để nhận ra dãy số (xn ) và xử lí là không dễ. Đây là một
bài toán trong IMO Shortlist 2018.

64
Chuyên đề Toán học Số 12

Bài 3. Cho x1 , x2 , x3 . . . là dãy số nguyên thỏa mãn đồng thời hai điều kiện

1 = x1 < x2 < x3 < . . . và xn+1 6 2n với n = 1, 2, 3, . . .

Chứng minh với mọi số nguyên dương k, tồn tại các số nguyên i > j sao cho xi −xj = k.

Lời giải. Với mọi số nguyên dương n, các số x1 , x2 , . . . , xn+1 nằm trong n tập
{1, n + 1}, {2, n + 2}, . . . , {n, 2n}
nên có hai số thuộc vào cùng một tập hay luôn tìm được hai số trong dãy có hiệu bằng n.
Nhận xét. Bài toán này đơn thuần là phát biểu lại một bài toán cũ và quen thuộc. Một
số bài toán tương tự và tổng quát hơn:
Bài tập 1. Cho m, n là các số nguyên dương. S là tập gồm (m − 1)n + 1 số nguyên dương
không lớn hơn mn. Chứng minh rằng tồn tại x1 , x2 , . . . , xm+1 ∈ S sao cho
xm+1 = x1 + x2 + · · · + xm .
Bài tập 2. Cho số nguyên dương m và (xn ) là dãy số nguyên thỏa mãn
0 < x1 < x2 < x3 < . . . và xn+1 6 mn với n = 1, 2, 3, . . .
Chứng minh rằng tồn tại các số nguyên dương y1 , y2 , . . . , ym để mọi số nguyên l đều biểu
diễn được dưới dạng l = xi − xj + yk với i, j, k ∈ N∗ và k ≤ m.

Bài 4. Cho tam giác ABC cân tại A, nội tiếp đường tròn (O). Gọi M là điểm trên
−−→ 1 −→
cạnh AB sao cho AM = AB. Đường tròn tâm M bán kính M B cắt (O) tại điểm thứ
3
hai là D. Một đường thẳng qua M song song với AD cắt AC tại N . Chứng minh rằng
−−→ 2 −→
AN = AC.
3
Lời giải. Tứ giác ADN M nội tiếp do
∠DAN = ∠DBC = ∠ABC − ∠ABD = ∠ADB − ∠M DB = ∠ADM = ∠DM N.

2 2
Nên ADN M là hình thang cân, suy ra AN = M D = M B = AB = AC. 
3 3
65
Lời giải đề thi chọn đội dự tuyển các năm

2.4. Năm 2021

Bài 1. Cho các số thực không âm a, b, c thoả a2 + b2 + c2 = 1. Tìm giá trị lớn nhất của
7 √ √ √
P = a + (1 − a)( a + b + c) + a2 b2 c2 .
2

7 7
Lời giải. Giá trị lớn nhất của P là , khi a = 1, b = c = 0. Ta chứng minh P ≤ . Từ
2 2
giả thiết, ta thu được a, b, c ≤ 1. Như vậy,
7 7 √ √ √
P − = − (1 − a) + (1 − a)( a + b + c) + a2 b2 c2
2 2
7 √ √ √ a2 (1 − a2 )2
≤ − (1 − a) + (1 − a)( a + b + c) + (AM – GM)
2 4
7 √ √ √ a2 (1 − a2 )(1 + a)
" #
= (1 − a) − + a + b + c +
2 4
7 1+a −3
 
≤ (1 − a) − + 3 + ≤ (1 − a) ≤ 0. (AM – GM)
2 16 8
Nhận xét. Bài bất đẳng thức không quá khó và không cần sử dụng các công cụ bất đẳng
thức mạnh nhưng phải nhận ra việc rút nhân tử 1 − a ra và sau đó đưa về một bất đẳng
thức rất yếu, vấn đề tới đây gần như đã được giải quyết.

Bài 2. Tìm tất cả các hàm số f : R → R sao cho với mọi số thực x và y,

f (x − f (y)) = 4f (x) + 3x + f (y).

Lời giải. Đặt g(x) = f (x) + x, ta được

g(x + y − g(y)) = 4g(x), , ∀x, y ∈ R. (1)

Đặt g(0) = a. Thay y bởi 0 vào điều kiện ban đầu, ta được

g(x − a) = 4g(x), , ∀x ∈ R. (2)


a
Thay x bởi a vào (2), ta được g(a) = . Thay y bởi a vào (1), ta được
4
3a
 
g x+ = 4g(x) , ∀x ∈ R. (3)
4
Áp dụng (2) và (3), ta được

g(x) = 43 g(x + 3a) và g(x + 3a) = 44 g(x)

nên g(x) = 0 hay f (x) = −x. Thử lại, ta thấy f (x) = −x là hàm số thỏa mãn yêu cầu.
Nhận xét. Ngoài cách thế như trên, ta có thể giải trực tiếp mà không cần thông qua các
bước đặt ẩn, chẳng hạn : tính f (0), chứng minh {f (x)−4f (y) | x, y ∈ R} = R.

66
Chuyên đề Toán học Số 12

Bài 3. Cho số nguyên dương n và A là một tập hợp bao gồm 8n + 1 số nguyên dương
nguyên tố cùng nhau với 6 và nhỏ hơn 30n. Chứng minh rằng tồn tại hai số phân biệt
a, b ∈ A sao cho a chia hết cho b.

Lời giải. Với n ∈ N∗ và (n, 6) = 1, ta có thể viết n = 5k m với (m, 30) = 1. Trong các số
không vượt quá 30n, có nϕ(30) = 8n số nguyên tố cùng nhau với 30. Do đó, theo nguyên
lí Dirichlet, luôn tồn tại hai số phân biệt a, b thuộc A có dạng a = 5x m và b = 5y m. Như
vậy, trong hai số a, b có một số chia hết cho số còn lại. 
Nhận xét. Ý tưởng bài toán dựa vào kết quả kinh điển của Erdos : Trong n+1 số nguyên
dương nhỏ hơn 2n luôn có hai số mà số lớn chia hết cho số bé. Bài toán được lấy từ đề
chọn đội tuyển JBMO của Romania năm 2019.

Bài 4. Cho điểm M di động trên đường thẳng d cố định và O là điểm cố định nằm
ngoài đường thẳng d. Gọi A là hình chiếu của O trên d, H là hình chiếu của A trên
OM , D là trung điểm HM và K là hình chiếu của H lên AD. Chứng minh rằng HK
luôn đi qua một điểm cố định khi M thay đổi trên d.

Lời giải. Đường thẳng HK và AO cắt nhau tại B, gọi N là trung điểm AH.

Ta có DN k d do DN là đường trung bình trong 4HAM , kết hợp với d ⊥ OA, ta được
DN ⊥ OA. Vậy N là trực tâm tam giác OAD (giao điểm hai đường cao) nên ON ⊥ AD.
Khi đó ON k HB nên O là trung điểm AB. Do đó HK luôn đi qua B cố định.

2.5. Năm 2022

Bài 1. Cho a, b, c ≥ 0 thỏa mãn a2 + b2 + c2 = 1. Tìm giá trị lớn nhất và giá trị nhỏ
nhất của biểu thức P = ab + bc + ca − 2(a + b + c)

Lời giải. Ta có
(a + b + c)2 1
P = ab + bc + ca − 2(a + b + c) = − 2(a + b + c) −
2 2
1 5
= (a + b + c − 2)2 − .
2 2

67
Lời giải đề thi chọn đội dự tuyển các năm

(a+b+c)2
Ta lại có 3
≤ a2 + b2 + c2 ≤ (a + b + c)2 nên từ giả thiết ta được

√ √
1≤a+b+c≤ 3 ⇒ −1 ≤ a + b + c − 2 ≤ 3 − 2 < 0.

Do đó ta được
1 √ 5 1 5
( 3 − 2)2 − ≤ P ≤ · 12 − .
2 2 2 2
√ √
• P = 12 ( 3 − 2)2 − 5
2
= 1 − 2 3 chẳng hạn khi a = b = c = √1 .
3

• P = 1
2
· 12 − 5
2
= −2 chẳng hạn khi a = 1, b = c = 0.

Vậy giá trị lớn nhất và giá trị nhỏ nhất của P lần lượt là −2 và 1 − 2 3.

Nhận xét. Đây là một bài toán khá dễ và quen thuộc, học sinh cấp 2 cũng có thể giải
quyết được.

Bài 2. Cho k, n ∈ Z+ thoả k < n. Hãy tính số đơn ánh f : {1, 2, 3, . . . , 2k + 1} →


{1, 2, 3, . . . , 2n} thoả mãn điều kiện f (1) < f (2) < . . . f (k) < f (k + 1) > f (k + 2) >
· · · > f (2k) > f (2k + 1) và f (k + 1) 6= 2n − 2.

Lời giải. Trước hết, ta sẽ chọn tập ảnh của f . Từ đề bài, ta có

max{f (i)|1 ≤ i ≤ 2k + 1} = f (k + 1) 6= 2n − 2.

Có C2n
2k+1
cách chọn các số vào tập ảnh, trong số đó có C2n−3
2k
cách chọn để phần tử lớn
nhất của tập ảnh là 2n − 2, suy ra số tập ảnh hợp lệ là C2n − C2n−3
2k+1 2k
. Ứng với mỗi tập
ảnh, số lớn nhất của tập đó sẽ là f (k + 1). Đặt

A = {f (i)|1 ≤ i ≤ k}, B = {f (i)|k + 2 ≤ i ≤ 2k + 1}.

Khi này, có C2k


k
cách chọn A và Ckk cách chọn B sau khi chọn A. Ứng với mỗi cách chọn
A, B, ta có đúng 1 cách xếp các số trong A và trong B theo thứ tự từ bé đến lớn, do đó
cũng có đúng 1 cách xác định f (1), f (2), . . . , f (k), f (k + 2), f (k + 3), . . . , f (2k + 1).
Từ đây, ta suy ra số đơn ánh thoả mãn yêu cầu đề bài là (C2n 2k+1 2k
− C2n−3 ) · C2k
k
.

Nhận xét. Lời giải trên tuy có vẻ đơn giản nhưng lại đòi hỏi nhiều kỹ thuật đếm không
tầm thường. Thêm vào đó, điều kiện f (k + 1) 6= 2n − 2 có thể khiến cho một số bạn sa
đà vào việc xét từng trường hợp đối với f (k + 1), do đó không thu được kết quả ở dạng
rút gọn. Vì thế bài toán này thật sự là một thách thức với các bạn lớp 10.

Bài tập 1 (Ý tưởng từ St. Petersburg 2016). Cho các số nguyên dương m, n thoả mãn
3m ≤ n. Xét một hình bao gồm m hàng, mỗi hàng có 3 hình vuông, đồng thời các hình
vuông ở mỗi hàng được dịch sang bên phải một ô so với hàng ở ngay phía trên nó (ví dụ

68
Chuyên đề Toán học Số 12

với m = 4, ta thu được hình dưới đây).

Điền vào trong các ô vuông của hình này mỗi ô một số nguyên dương không vượt quá n
sao cho :

i) Các số được điền trong các ô là đôi một khác nhau.

ii) Số trong mỗi hàng tăng dần theo thứ tự từ trái sang phải và số trong mỗi cột tăng
dần theo thứ tự từ trên xuống dưới.

Hỏi có tổng cộng bao nhiêu cách điền?

Bài 3. Với mỗi số nguyên dương n, đặt a(n) = 1 + 2 + · · · + n và b(n) = 1 + 2 + · · · + n2 .


Hỏi có tồn tại số n sao cho 2(n + 1)a(n) + 3b(n) − 1 là số chính phương?

Lời giải. Ta có

n(n + 1) n(n + 1)(2n + 1)


a(n) = 1 + 2 + · · · + n = , b(n) = 1 + 2 + · · · + n2 = .
2 6
Suy ra

n(n + 1)(2n + 1) n(n + 1)(4n + 3)


2(n + 1)a(n) + 3b(n) − 1 = (n + 1)2 n + −1= − 1.
2 2

Nếu n(n+1)(4n+3)
2
− 1 = x2 (x ∈ N), suy ra n(n + 1)(4n + 3) = 2(x2 + 1). Khi này, 4n + 3
có một ước nguyên tố p có dạng 4k + 3. Với p này, ta có p | 2(x2 + 1), vô lí vì 2(x2 + 1)
không có ước nguyên tố dạng 4k + 3.
Từ điều vô lí nêu trên, ta suy ra không tồn tại số n thoả mãn yêu cầu đề bài.

Nhận xét. Đây là một bài toán có phát biểu khá lạ. Có lẽ vì vậy nên không nhiều bạn
nhận ra rằng điểm máu chốt của bài toán chính là tính chất quen thuộc: Một số có dạng
x2 + 1 (x ∈ N) thì không có ước nguyên tố nào có dạng 4k + 3, dẫn đến không giải quyết
được trọn vẹn bài toán.

Bài 4. Cho tam giác ABC có 2∠A = 5∠B = 10∠C. Phân giác trong BD cắt trung
tuyến CM tại I. Một đường thẳng d đi qua D vuông góc với AC cắt BC và AI lần
lượt tại E và K. AE cắt CK tại F . Chứng minh M F song song BK.

69
Lời giải đề thi chọn đội dự tuyển các năm

Lời giải. Ta tính được ∠BAC = 112.5◦ , ∠BCA = 22.5◦ , ∠ABC = 45◦ .

A
D
J
M
I

B E C
G

F
K

Gọi AI cắt BC tại G, F G cắt AC tại H. Do KD, CG, AF đồng quy nên ta được
HA DA BA
(HDAC) = −1 ⇒ = = .
HC DC BC
Mặt khác, áp dụng định lý Menelaus cho tam giác ABG có M, I, C thẳng hàng với
M ∈ AB, I ∈ AG, C ∈ BG ta được
M A IG CB CG IG BG CG BC
· · =1⇒ = = ⇒ =
M B IA CG CB IA BA BG BA
Do đó trong tam giác ABC có BG · M A · HC = 1, suy ra H, M, G, F thẳng hàng.
CG M B HA

Ta sẽ chứng minh G là trung điểm AK, từ đó có được điều phải chứng minh.
Thật vậy, hạ đường cao AG0 . Ta có
CG0 AB · cos 45◦ AB · sin 45◦ AB AC AB GB
= = = · = = .
BG 0 AC · cos 22.5 ◦ AC · sin 112.5◦ AC BC BC GC
Vậy G trùng G0 , hay AI là đường cao của tam giác ABC.
Ta có ∠BAD = ∠BED = 112.5◦ và BD là phân giác ∠ABE nên BD vuông góc AE tại
trung điểm AE là J, suy ra tứ giác BAJG nội tiếp và

∠BJG = ∠BAG = 90◦ − ∠ABG = 45◦ .

Ngoài ra, ta cũng có

∠BDC = ∠ABD + ∠BAD = 22.5◦ + 112.5◦ = 135◦ ⇒ ∠BDE = ∠BDC − 90◦ = 45◦ .

Kết hợp với điều ngay trên ta có ∠BJG = ∠BDE nên JG k EK, suy ra G là trung điểm
AK. Ta có điều phải chứng minh.

3. Lời kết
Ban biên tập xin cảm ơn các bạn học sinh đã đóng góp để hoàn thiện lời giải :

70
Chuyên đề Toán học Số 12

• Lớp Toán 17 – 20 : Bá Dũng, Kế Huy, Thái Tài, Nam Trung.

• Lớp Toán 18 – 21 : Nam Hưng, Gia Hy, Vũ Tường.

• Lớp Toán 19 – 22 : Tiến Khải, Hoàng Sơn.

• Lớp Toán 20 – 23 : Thanh Danh.

• Lớp Toán 21 – 24 : Thụ Nghĩa, Ngọc Huyền.

71
Lời giải đề thi chọn đội tuyển các năm
Ban biên tập
Giới thiệu. Bài viết sẽ giới thiệu đến bạn đọc lời giải đề thi chọn đội tuyển các
năm 2019, 2020, 2021 cùng những nhận xét, mở rộng thú vị liên quan đến các
bài toán này.

1. Đề thi
1.1. Năm 2019
Bài 1. Số thực α được gọi là điểm tụ của dãy số (un ) nếu tồn tại ít nhất một dãy con
của dãy (un ) hội tụ về α.

a) Hãy chỉ ra một dãy số có vô hạn điểm tụ.

b) Chứng minh rằng nếu mọi dãy con của dãy (un ) đều hội tụ thì dãy (un ) hội tụ.

1, nếu n ∈ S \ {0}
c) Gọi S là tập tất cả số chính phương. Dãy số (an ) thỏa an = n
.
1
n2
, nếu n 6∈ S
n
Xét sự hội tụ của các dãy (an ) và (bn ) với bn = ai .
X

i=1

Bài 2. Tìm tất cả các hợp số n thỏa nσ(n) ≡ 2 (mod ϕ(n)) với σ(n) là tổng các ước
nguyên dương của n, ϕ(n) là số các số nguyên dương không lớn hơn n và nguyên tố cùng
nhau với n.

Bài 3. Tìm tất cả các hàm f : R → R thỏa mãn với mọi số thực x, y,

f (f (x) + y) + f (x)f (f (y)) = xf (y) + x + y.

Bài 4. Cho tam giác ABC nhọn nội tiếp đường tròn (O) với B, C cố định và A thay đổi
trên cung lớn BC. Các đường tròn bàng tiếp góc A, B, C lần lượt tiếp xúc với BC, CA, AB
tại D, E, F.

a) Gọi L 6= A là giao điểm của (ABE) và (ACF ). Chứng minh AL luôn qua một điểm
cố định.

b) (BCF ) cắt (BAD) tại M, B và (CAD) cắt (CBE) tại N, C. Gọi K, I, J theo thứ tự
là trung điểm của AD, BE, CF . Chứng minh KL, IM, JN đồng qui.

Bài 5. Cho a, b, c là các số thực dương thỏa điều kiện 8(a2 + b2 + c2 ) = 9(ab + bc + ca).
b+c c+a a+b
Tìm giá trị lớn nhất và giá trị nhỏ nhất của P = + + .
a b c
Bài 6. Tìm tất cả các hàm f : N∗ → N∗ thỏa mãn đồng thời các điều kiện sau :

(i) mf (m) + nf (n) + 2mf (n) là số chính phương với mọi số nguyên dương m, n.

72
Chuyên đề Toán học Số 12

(ii) f (mn) = f (m)f (n) với mọi số nguyên dương m, n.

(iii) Với mọi số nguyên tố p, f (p) không chia hết cho p2 .

Bài 7. Một trường phổ thông có n học sinh. Các học sinh tham gia vào nhiều câu lạc bộ
khác nhau và có tất cả m câu lạc bộ là A1 , A2 , . . . , Am .

a) Chứng minh rằng nếu mỗi câu lạc bộ có đúng 4 học sinh tham gia và hai học sinh
n(n − 1)
bất kỳ cùng tham gia chung nhiều nhất là một câu lạc bộ thì m ≤ .
12
b) Giả sử tồn tại k > 0 sao cho Ai 6= Aj , |Ai ∩ Aj | = k với mọi i < j và có một câu lạc
bộ At mà |At | = k. Chứng minh m ≤ n.

Bài 8. Cho tam giác ABC nội tiếp đường tròn (O). Đường tròn (I) tiếp xúc với các cạnh
BC, CA, AB tại D, E, F. Gọi J là tâm đường tròn bàng tiếp góc A và H là hình chiếu
vuông góc của D trên EF.

a) Chứng minh giao điểm của AH và JD thuộc OI.

b) DH cắt (I) tại K 6= D, IK cắt đường tròn (T ) ngoại tiếp tam giác IEF tại L 6= I.
Chứng minh AD và KH cắt nhau trên đường tròn (T ).

1.2. Năm 2020


Bài 1. Với mỗi số nguyên dương n, tìm số thực Mn > 0 lớn nhất sao cho với mọi số
dương x1 , x2 , . . . , xn , ta luôn có
 2
n
1 1 1 n
1 
+ + P
X X
2 !2 ≥ Mn  n .
k=1 xk n
P k=1 xk x k
xk k=1
k=1

Bài 2. Cho 2021 số nguyên khác 0. Biết rằng tổng của một số bất kỳ trong chúng với
tích của tất cả 2020 số còn lại luôn âm.

a) Chứng minh rằng với mọi cách chia 2021 số này thành hai nhóm và nhân các số cùng
nhóm với nhau thì tổng của hai tích thu được cũng luôn âm.

b) Một bộ số thỏa mãn điều kiện đề bài có thể có nhiều nhất bao nhiêu số âm ?

Bài 3. Cho hai hàm số f, g : R → R thỏa mãn g(2020) > 0 và với mọi số thực x, y,

f (x − g(y)) = f (−x + 2g(y)) + xg(y) − 6
g(y) = g(2f (x) − y)

a) Chứng minh rằng g là hàm hằng.

b) Chứng minh rằng đồ thị hàm số h(x) = f (x) − x nhận đường thẳng x = 1 là trục
đối xứng.

73
Lời giải đề thi chọn đội tuyển các năm

Bài 4. Cho tam giác ABC nhọn nội tiếp (O) có H là trực tâm. AH, BH, CH lần lượt
cắt BC, CA, AB tại D, E, F và I, M, N là trung điểm BC, HB, HC. BH, CH cắt (O) tại
L, K (L 6= B, K 6= C); KL cắt M N tại G.

a) Trên EF lấy điểm T sao cho AT ⊥ IH. Chứng minh GT ⊥ OH.

b) DE, DF lần lượt cắt M N tại P, Q. Gọi S là giao điểm của BQ với CP . Chứng minh
rằng HS đi qua trung điểm EF.
1
 
Bài 5. Cho số nguyên n > 1. Chứng minh rằng với mọi số thực a ∈ 0, và mọi đa
n
thức P (x) bậc 2n − 1 thỏa điều kiện P (0) = P (1) = 0, luôn tồn tại các số thực x1 , x2
thuộc [0, 1] sao cho P (x1 ) = P (x2 ) và x2 − x1 = a.

Bài 6. Tìm các số nguyên dương x, y thỏa phương trình


 
3x+1 3x+1
(x + 3)
2
(x + 3)
2
+ 1 + x2 + y = x2 y.

Bài 7. Cho các số nguyên n > k > t > 0 và X = {1, 2, . . . , n}. Gọi F là họ gồm các tập
con k phần tử của X sao cho |F ∩ F 0 | ≥ t với mọi F, F 0 thuộc F. Giả sử không có tập
con t phần tử nào được chứa trong tất cả các tập F ∈ F. Chứng minh rằng

a) Tồn tại tập hợp B với |B| < 3k thỏa |B ∩ F | ≥ t + 1 với mọi F ∈ F.

b) |F| < C3k


t+1
· Cnk−t−1 .

Bài 8. Tam giác ABC nội tiếp đường tròn (O) với B, C cố định và A thay đổi trên cung
lớn BC. Dựng hình bình hành ABDC, AD cắt (BCD) tại K 6= D.

a) Gọi R1 , R2 là bán kính của (KAB), (KAC); chứng minh R1 R2 không đổi.

b) (T ) là đường tròn qua K tiếp xúc với BD tại B và (T1 ) là đường tròn qua K tiếp
xúc với CD tại C. (T ) và (T1 ) cắt nhau tại L 6= K. Chứng minh AL luôn đi qua
một điểm cố định.

1.3. Năm 2021


Bài 1. Tìm hàm số f : R → R thoả

f (xf (y) + f (x)) = f (x) + xy + x + 1, ∀x, y ∈ R.


un un−1
r
Bài 2. Cho dãy số (un ) thoả u1 = 2, u2 = 1 và un+1 = với mọi n ≥ 2. Xét dãy
n
số (vn ) xác định bởi vn := u1 + u2 + · · · + un , ∀n ≥ 1. Chứng minh dãy (vn ) hội tụ.

Bài 3. Cho p là số nguyên tố lẻ và n là một số nguyên dương lớn hơn p. Gọi A là tập
hợp các đa thức P (x) = xn + an−1 xn−1 + · · · + a1 x + a0 thoả mãn ai ∈ {1, 2, . . . , n!} với
mọi i = 0, 1, 2, . . . , n − 1, đồng thời P (m) chia hết cho p với mọi số nguyên dương m.
n−1
$ %
a) Chứng minh tổng a1 + ap + a2p−1 + · · · + a1+k(p−1) , trong đó k = , chia hết
p−1
cho p (xem an = 1).

74
Chuyên đề Toán học Số 12

b) Tính số phần tử của A theo n và p.

(Với mỗi số thực x, bxc là số nguyên lớn nhất không vượt quá x.)

Bài 4. Cho tam giác ABC có (I) là đường tròn nội tiếp. Một đường thẳng qua A cắt (I)
tại M, N . Gọi T là giao điểm của các tiếp tuyến với (I) tại M, N .

a) Chứng minh rằng nếu AT k BC thì M N đi qua trung điểm K của BC.

b) Gọi D là tiếp điểm của (I) với AB và E là giao điểm của DM với AC. Trên EN lấy
điểm F thoả T F vuông góc AI. Chứng minh rằng khi đường thẳng AM N thay đổi,
giao điểm P của M F và DN thuộc một đường thẳng cố định.

Bài 5. Cho n số thực x1 , x2 , . . . , xn thoả hiệu giữa số lớn nhất và số nhỏ nhất trong chúng
là 1. Ta xây dựng
x1 + x2 x1 + x2 + · · · + xn
y1 = x1 , y2 = , . . . , yn = .
2 n
Đặt T = max yi − min yj . Tìm giá trị lớn nhất của T .

Bài 6. Cho tập X = {1, 2, . . . , 20}. Tập con A của X được gọi là tập tránh 2 nếu với
mọi x, y ∈ A thì |x − y| khác 2. Tìm số tập con tránh 2 của X có 5 phần tử.

Bài 7. Cho tam giác ABC và điểm D trên cạnh BC. Các đường tròn (ABD), (ACD)
lần lượt cắt AC, AB tại E, F . Gọi I là tâm đường tròn (AEF ).

a) Chứng minh ID vuông góc BC.

b) Gọi H là giao điểm của ID với EF và K là điểm thoả mãn HBK


\ = HCK \ = 900 .
Các đường tròn (IBK), (ICK) lần lượt cắt IC, IB tại M , N . Chứng minh tâm J
của đường tròn (IM N ) thuộc trung trực BC.

Bài 8. Cho p là số nguyên tố. Với mọi số nguyên a, đặt

q = 1 + a + a2 + · · · + ap−1 .

Chứng minh rằng (1 − a)(1 − a2 ) . . . (1 − ap−1 ) − p chia hết cho q.

75
Lời giải đề thi chọn đội tuyển các năm

2. Lời giải
2.1. Năm 2019

Bài 1. Số thực α được gọi là điểm tụ của dãy số (un ) nếu tồn tại ít nhất một dãy
con của dãy (un ) hội tụ đến α.

a) Hãy chỉ ra một dãy số có vô hạn điểm tụ.

b) Chứng minh rằng nếu mọi dãy con của dãy (un ) đều hội tụ thì dãy (un ) hội tụ.
1
c) Gọi S là tập các số chính phương. Dãy số (an ) thỏa an = nếu n ∈ S \ {0} và
n
1 n
an = nếu Xét tính hội tụ của các dãy (a ) và (b ) với = ai .
X
n ∈
6 S. n n b n
n2 i=1

Lời giải.

a) Xét dãy un xác định bởi u1 = 0, un = p(n) với p(n) là ước nguyên tố lớn nhất của n
với mọi n > 1. Khi đó mọi số nguyên tố p đều là điểm tụ của dãy (do upk = p).

b) Chọn dãy con (un ) là (un ), ta suy ra điều phải chứng minh.
1 1
c) Do 0 < an < với mọi n nguyên dương và limn→+∞ = 0 nên theo định lý kẹp,
n n
lim an = 0.
Dẽ thấy (bn ) là dãy tăng, ta chứng minh (bn ) bị chặn. Ta có
2
n n
1 X n
1
= ai 2 + +
X X X
bn2 ai < 2 2
.
i=1 i6∈S,i<n2 i=1 i i=1 i

Với mọi n nguyên dương,


n
1 n
1 1
<1+ =2− <2
X X

i=2 i(i − 1)
2
i=1 i n

nên bn2 < 4 với mọi n nguyên dương, kết hợp (bn ) tăng, ta suy ra (bn ) bị chặn trên
bởi 4 nên (bn ) hội tụ.

Nhận xét. Đây là một bài toán khá cơ bản. Ý a tương đối mở, ý b thì quá hiển nhiên,
còn ý c thì khá gần gũi với các bài toán cấp 2. Ở ý a, ngoài cách xây dựng như trên, ta
cũng có thể chọn un = v2 (n), un = S(n), . . . Xin giới thiệu đến bạn đọc một mở rộng nhỏ
cho ý này.

Bài tập 1. Cho số thực dương L > 1. Hỏi có tồn tại hay không một dãy gồm các số
nguyên dương sao cho dãy này có vô hạn điểm tụ, đồng thời với mọi số nguyên dương n,
trung bình cộng của n số hạng đầu tiên của dãy nhỏ hơn L?

76
Chuyên đề Toán học Số 12

Bài 2. Tìm tất cả các hợp số n > 0 thỏa nσ(n) ≡ 2 (mod ϕ(n)) với σ(n) là tổng các
ước nguyên dương của n và ϕ(n) là số các số nguyên dương không lớn hơn n và nguyên
tố cùng nhau với n.

Lời giải. Nếu n là lũy thừa của số nguyên tố, n = pk (k ≥ 2), khi đó, ϕ(n) = pk − pk−1
chia hết cho p, suy ra 2 ≡ 0 (mod p) hay p = 2. Nếu k > 2 thì ϕ(2k ) chia hết cho 4, suy
ra vô lí. Vậy k = 2 hay n = 4.
Nếu n có ít nhất hai ước nguyên tố phân biệt, xét phân tích ra thừa số của

n = 2s pa11 pa22 . . . pakk

với s ∈ N, k, a1 , a2 , . . . , ak là các số nguyên dương, p1 , . . . , pk là các số nguyên tố lẻ.


Suy ra ai = 1, vì nếu ai > 1 thì ta suy ra ϕ(n) chia hết cho pi nên pi | 2 (vô lí).
Nếu k ≥ 2, do σ(n) = (2s+1 − 1)(p1 + 1) . . . (pk + 1) chia hết cho 4 và ϕ(n) = 2s−1 (p1 −
1) . . . (pk − 1) chia hết cho 4 nên mâu thuẫn. Nên k = 1 và s ≥ 1. Nếu s ≥ 2 thì
ϕ(n) = 2s−1 (p1 − 1) chia hết cho 4 mà n chia hết cho 4 nên mâu thuẫn. Vậy s = 1.
Ta quy bài toán về tìm số nguyên tố p sao cho 2pσ(2p) ≡ 2 (mod ϕ(2p)), tương đương

6p(p + 1) ≡ 2 (mod p − 1) ⇒ 10 ≡ 0 (mod p − 1).

Ta tìm được p = 3 hoặc p = 11, tức n = 6 hoặc n = 22.


Vậy tập các số nguyên dương n cần tìm là {4, 6, 22}.

Nhận xét. Tuy bài toán có dạng hơi lạ (có đến hai hàm số học xuất hiện trong cùng
một phương trình đồng dư) nhưng ý tưởng giải quyết cũng khá quen thuộc, chủ yếu xoay
quanh kĩ thuật chặn.

Bài tập 1. Gọi S là tập hợp tất cả các số nguyên dương n thỏa mãn điều kiện
s
n3
τ (n)ϕ(n) ≥ ,
3

trong đó τ (n) là số ước nguyên dương của n.

a) Chứng minh rằng nếu n không thuộc S và p là một số nguyên tố lẻ thì np cũng
không thuộc S.

b) Chứng minh rằng S có đúng 10 phần tử. Hãy xác định 10 phần tử này.

Bài 3. Tìm tất cả các hàm f : R → R thỏa

f (f (x) + y) + f (x)f (f (y)) = xf (y) + x + y

với mọi số thực x, y.

Lời giải. Xét P (x, y) tương ứng với phép thay x, y vào điều kiện của đề bài.
P (0, 0) : f (f (0))(f (0) + 1) = 0. Đặt f (0) = a, ta thu được f (a) = 0 hoặc a = −1.

77
Lời giải đề thi chọn đội tuyển các năm

• Nếu f (a) = 0, P (a, 0) : a = a2 + a hay a = 0. P (0, x) : f (x) = x (thử lại thỏa).

• Nếu a = −1, P (x, 0) : f (f (x)) + f (x)f (−1) = 0. P (x, f (y) :

f (f (x) + f (y)) + f (x)f (y)f (−1)2 = −xf (y)f (−1) + x + f (y).

Đổi vai trò x, y, ta thu được −xf (y)f (−1) + x + f (y) = −yf (x)f (−1) + y + f (x).
Thay y = 0 vào biểu thức trên, ta được f (x) = x(f (−1) + 1) − 1, thay x = −1, ta
được f (−1) = −1 hay f (x) = −1, tuy nhiên, đây không là nghiệm hàm thỏa mãn.

Tóm lại, f (x) = x là hàm số duy nhất thỏa mãn.

Bài 4. Cho tam giác ABC nhọn nội tiếp đường tròn (O) với B, C cố định và A thay
đổi trên cung lớn BC. Các đường tròn bàng tiếp góc A, B, C lần lượt tiếp xúc với
BC, CA, AB tại D, E, F.

a) Gọi L 6= A là giao điểm của (ABE) và (ACF ). Chứng minh AL luôn qua một
điểm cố định.

b) (BCF ) cắt (BAD) tại M, B và (CAD) cắt (CBE) tại N, C. Gọi K, I, J theo thứ
tự là trung điểm của AD, BE, CF . Chứng minh KL, IM, JN đồng qui.

Lời giải.

F
K

N O E
J

D
B C
L

a) Theo tính chất vị tự quay, 4LBF v 4LEC mà BF = CE nên LB = LE, suy ra


AL là phân giác ∠BAC nên AL đi qua điểm chính giữa cung nhỏ BC của (O).

78
Chuyên đề Toán học Số 12

b) Chú ý rằng L là điểm chính giữa cung BE, CF không chứa A của (ABE), (ACF ).
Tương tự, ta cũng có M, N lần lượt là điểm chính giữa cung AD không chứa B, C
của (ADB), (ADC) nên M N là trung trực AD, suy ra K ∈ M N. Tương tự, ta cũng
B B
KM KD · tan tan
có J ∈ LM, I ∈ LN. Ngoài ra, = 2 = 2 , nên theo định lý Ceva
KN C C
KD · tan tan
2 2
trong tam giác LM N, ta suy ra LK, IM, JN đồng qui.

Bài 5. Cho a, b, c là các số thực dương thỏa điều kiện

8(a2 + b2 + c2 ) = 9(ab + bc + ca).

b+c c+a a+b


Tìm giá trị lớn nhất và giá trị nhỏ nhất của P = + + .
a b c

Lời giải. Chuẩn hóa ab + bc + ca = 8, suy ra a2 + b2 + c2 = 9 và (a + b + c)2 = 25 hay


a + b + c = 5. Ta có
(a + b + c)(ab + bc + ca) 40
P +3= = .
abc abc
Lại có

(b + c)2 2a2 + (5 − a)2


9 = a2 + b 2 + c 2 ≥ a2 + = ⇔ 0 ≥ (a − 1)(3a − 7).
2 2
7
 
Vậy ta suy ra a, b, c ∈ 1, . Ta có
3
• (a − 1)(b − 1)(c − 1) ≥ 0 ⇒ abc ≥ 4. Nên max P = 7, đạt được khi a = 1, b = c = 2.
7 7 7 112 91
   
• a− − c− ≤ 0 ⇒ abc ≤ . Nên min P = , đạt được khi a =
3 3 3 27 14
7 4
,b = c = .
3 3

Bài 6. Tìm tất cả các hàm f : N∗ → N∗ thỏa mãn đồng thời các điều kiện sau :

(i) mf (m) + nf (n) + 2mf (n) là số chính phương với mọi m, n.

(ii) f (mn) = f (m)f (n) với mọi m, n.

(iii) Với mọi số nguyên tố p, f (p) không chia hết cho p2 .

Lời giải. Thay m = n vào điều kiện đầu tiên, ta suy ra nf (n) là số chính phương.
Từ điều kiện thứ hai, ta suy ra f (1) = 1. Từ đây, lần lượt thay m và n trong điều kiện
(i) bởi 1, ta suy ra được mf (m) + 1 + 2m và mf (m) + 1 + 2f (m) là số chính phương với
mọi m nguyên dương. Nếu f (m) 6= m, ta xét hai trường hợp sau đây

79
Lời giải đề thi chọn đội tuyển các năm

• m < f (m) q
Khi này, ta có A = mf (m) + 1 + 2m < ( mf (m) + 1)2 .
• m > f (m) q
Tương tự như trên, ta lại có B = mf (m) + 1 + 2f (m) < ( mf (m) + 1)2 .
tuy nhiên điều đó lại mâu thuẫn, do A, B là các số chính phương lớn hơn mf (m).
Vậy f (m) = m với mọi m nguyên dương.
Nhận xét. Nhìn chung, điều kiện cuối cùng là điều kiện thừa, chỉ có tác dụng gợi ý
việc thay số nguyên tố vào hai điều kiện còn lại. Nếu chỉ giữ lại điều kiện đầu tiên thì
f (m) = m không phải là đáp số duy nhất cho bài toán.
Bài tập 1. Cho hàm số f : N∗ → N∗ thoả mãn điều kiện mf (m) + nf (n) + 2mf (n) là số
chính phương với mọi m, n nguyên dương. Chứng minh rằng f (1) là số chính phương và
f (n) = nf (1) với mọi số nguyên dương n.
Một bài toán tương tự từng xuất hiện trong đề thi chọn đội tuyển IMO của Iran năm
2011.
Bài tập 2 (Iran TST 2011). Giả sử f : N∗ → N∗ là một hàm số thoả mãn điều kiện giá
trị của biểu thức mf (m) + nf (n) + 2mn là một số chính phương với mọi số nguyên dương
m, n. Chứng minh rằng f (m) = m với mọi m nguyên dương.

Bài 7. Một trường phổ thông có n học sinh. Các học sinh tham gia vào nhiều câu lạc
bộ khác nhau và có tất cả m câu lạc bộ là A1 , A2 , . . . , Am .

a) Chứng minh rằng nếu mỗi câu lạc bộ có đúng 4 học sinh tham gia và hai học sinh
n(n − 1)
bất kỳ cùng tham gia chung nhiều nhất là một câu lạc bộ thì m ≤ .
12
b) Giả sử tồn tại k > 0 sao cho Ai 6= Aj , |Ai ∩ Aj | = k với mọi i < j và có một câu
lạc bộ At mà |At | = k. Chứng minh m ≤ n.

Lời giải.
a) Gọi S là số bộ (i, j, Ak ) thỏa mãn học sinh thứ i, j tham gia câu lạc bộ Ak .
• Vì mỗi câu lạc bộ có đúng 4 học sinh tham gia và có m câu lạc bộ nên S = mC42 .
• Chọn hai học sinh bất kì thì có tối đa một câu lạc bộ mà hai học sinh này tham
gia, nên S ≤ Cn2 .
Cn2 n(n − 1) n(n − 1)
Suy ra m ≤ = = .
C42
2×6 12
b) Do |Ai ∩ At | = k nên At ⊂ Ai với mọi i 6= t, đặt Bi = Ai \ At , suy ra Bi ∩ Bj = ∅ với
mọi i, j. Do Ai 6= Aj với mọi i 6= j, nên Bi 6= Bt = ∅ với mọi i 6= t. Dẫn tới

m m
[
n−k = = |Bi | ≥ m − 1 ⇒ n ≥ m + k − 1 ≥ m.
X

Bi
i=1 i=1

80
Chuyên đề Toán học Số 12

Bài 8. Cho tam giác ABC nội tiếp đường tròn (O). Đường tròn (I) tiếp xúc với các
cạnh BC, CA, AB tại D, E, F. Gọi J là tâm đường tròn bàng tiếp góc A và H là hình
chiếu vuông góc của D trên EF.

a) Chứng minh giao điểm của AH và JD thuộc OI.

b) DH cắt (I) tại K 6= D, IK cắt đường tròn (T ) ngoại tiếp tam giác IEF tại
L 6= I. Chứng minh AD và LH cắt nhau trên đường tròn (T ).

Lời giải.

a) Gọi Ib , Ic lần lượt là tâm đường tròn bàng tiếp góc B, C.

Ib

Ic
E
H
F

I
O

B D C

Dễ thấy 4DEF và 4JIb Ic có các cạnh đôi một song song nên tồn tại phép vị tự
H : D → J, H → A (A là hình chiếu của J lên Ib Ic ), I → O0 với O0 là tâm đường
tròn ngoại tiếp tam giác JIb Ic . Mặc khác I là trực tâm tam giác JIb Ic , O là tâm

81
Lời giải đề thi chọn đội tuyển các năm

đường tròn Euler của tam giác JIb Ic nên I, O, O0 thẳng hàng, suy ra AH, JD, OI
đồng quy tại tâm của phép vị tự H.

b) Dễ thấy A ∈ T , AD cắt lại (T ) tại S. Ta có

180o − ∠DSL = ∠ASL = ∠AIL = ∠AIK = ∠DKI = 180o − ∠DKL

nên tứ giác DSKL nội tiếp đường tròn (ω). Xét ba đường tròn (ω), (I), (T ) :

Ib

L
Ic
E
K
F H
I
S
O

B D C

• DK là trục đẳng phương của (ω) với (I),


• SL là trục đẳng phương của (ω) với (T ),
• EF là trục đẳng phương của (I) với (T ).

Nên DK, SL, EF đồng quy tại tâm đẳng phương của (ω), (I), (T ), suy ra H, S, L
thẳng hàng hay HL cắt AD tại S ∈ (T ).

82
Chuyên đề Toán học Số 12

2.2. Năm 2020

Bài 1. Với mỗi số nguyên dương n, tìm số thực Mn > 0 lớn nhất sao cho với mọi số
dương x1 , x2 , . . . , xn , ta luôn có
 2
n
1 1 1 1 
n
+ + P
X X
2 !2 ≥ Mn  n .
k=1 xk n
P k=1 xk x k
xk k=1
i=1

1 1
 2
Lời giải. Chọn x1 = x2 = · · · = xn = 1, ta thu được n + 2 ≥ Mn n + , suy ra
n n
n3 + 1 n3 + 1
Mn ≤ 2 . Ta chứng minh Mn = 2 là giá trị cần tìm.
(n + 1)2 (n + 1)2
Áp dụng bất đẳng thức Cauchy – Schwarz, ta được
 2     
n
1 1  n
1 n n
1  n2 + 1  Xn
1 n
+P + 1+ = +
X X X

n ≤ 
2 !2 
2 !2 .
k=1 xk xk k=1 xk n
P k=1 n n k=1 xk n
P
k=1 xk xk
i=1 i=1

Áp dụng bất đẳng thức AM – GM :


n
X 1 n n3
2
≥ q ≥ !2 .
k=1 xk x21 . . . x2n
n n
P
xk
i=1

Suy ra
 2  
n
1 1  n3 + 1  Xn
1 n
+ P +
X
Mn  n ≤ 3+n 2 !2 
k=1 x k xk n k=1 x k n
P
k=1 xk
i=1
n
1 1 n−1 X n
1 n3
!
= +
X
!2 − 3 −
n + n k=1 x2k
2 !2
k=1 xk n
P n
P
xk xk
i=1 i=1
n
1 1
+
X
≤ 2 !2 .
k=1 xk n
P
xk
i=1

Nhận xét. Bài toán này là phiên bản tổng quát của bài toán sau đây trong đề thi Iran
MO, năm 2010:
Cho các số thực dương a, b, c. Chứng minh rằng
1 1 1 1 7 1 1 1 1
 2
+ 2+ 2+ ≥ + + + .
a 2 b c (a + b + c) 2 25 a b c a + b + c

Ý tưởng chính ở đây là coi vế trái như "tổng các bình phương" và vế phải là "bình phương
của tổng", cộng thêm dựa vào trường hợp xảy ra đẳng thức để đưa ra các biến đổi phù

83
Lời giải đề thi chọn đội tuyển các năm

hợp. Sử dụng ý tưởng này, ta cũng có thể giải quyết bài toán sau đây, thu được khi thay
số mũ 2 trong bài nêu trên bởi số nguyên dương m tuỳ ý (chú ý khi m = 1 thì vế trái và
vế phải bằng nhau).
Bài tập 1. Cho các số nguyên dương m, n. Tìm số thực Mm,n > 0 lớn nhất sao cho với
mọi số thực dương x1 , x2 , . . . , xn , ta luôn có
 m
n
1 1 n
1 1 
+ + P
X X
 n m ≥ Mm,n  n .
m
k=1 xk k=1 xk xk
P
xk
k=1 k=1

Bài 2. Cho 2021 số nguyên khác 0. Biết rằng tổng của một số bất kỳ trong chúng với
tích của tất cả 2020 số còn lại luôn âm.

a) Chứng minh rằng với mọi cách chia 2021 số này thành hai nhóm và nhân các số
cùng nhóm với nhau thì tổng của hai tích thu được cũng luôn âm.

b) Một bộ số thỏa mãn điều kiện đề bài có thể có nhiều nhất bao nhiêu số âm ?

Lời giải. Gọi T là tích của 2021 số nguyên này, dễ thấy T < 0 vì nếu T > 0 thì có ít
nhất một số dương, khi đó tổng của số dương đó với tích các số còn lại dương (vô lí).
a) Giả sử, ta chia hai 2021 số nguyên này thành hai nhóm là a1 , a2 , . . . , ak và ak+1 , . . . , a2021
và k > 1. Do (a1 . . . ak )(ak+1 . . . a2021 ) < 0 nên có một nhóm có tích là số âm, giả
sử a1 . . . ak < 0, không mất tính tổng quát, giả sử a1 < 0 (tức a2 . . . ak > 0 nên
a2 . . . ak ≥ 1). Suy ra

a1 a2 . . . ak + ak+1 . . . a2021 < a1 + a2 . . . a2021 < 0.

b) Giả sử có hai số âm là a và b. Khi đó a2 + T > 0, b2 + T > 0. Suy ra a2 b2 > T 2 hay


T
ab > −T. Ngoài ra, ab + < 0 nên a2 b2 < −T < ab hay 0 < ab < 1 (vô lí).
ab
Do đó có tối đa một số âm. Hơn nữa, dễ thấy bộ số (−2, 1, . . . , 1) thỏa mãn yêu cầu
đề bài. Từ đây, ta suy ra một bộ số thỏa đề sẽ có nhiều nhất 1 số âm.
Nhận xét. Đây là một bài toán đại số – tổ hợp khá nhẹ nhàng, học sinh cấp 2 cũng có
thể giải quyết dễ dàng. Nếu 2021 số ban đầu không phải là số nguyên mà là các số thực
khác 0 tuỳ ý, kết quả thu được ở các ý a và b không còn đúng nữa. Có thể dễ dàng kiểm
chứng điều này với bộ số − 12 , − 12 , . . . , − 12 . Một số bài toán tương tự.

Bài tập 1. Cho các số nguyên dương m, n và m + n số nguyên khác 0. Biết rằng nếu lấy
tích của m số bất kỳ cộng với tích của n số còn lại, kết quả thu được luôn là một số âm.
Hỏi trong m + n số này có thể có nhiều nhất bao nhiêu số âm?
Bài tập 2 (Áo TST 2018). Ta viết 2018 số nguyên trên một vòng tròn sao cho mỗi số
lớn hơn tổng của hai số liền trước nó theo chiều kim đồng hồ. Hỏi có tối đa bao nhiêu số
nguyên dương?

84
Chuyên đề Toán học Số 12

Bài 3. Cho hai hàm số f : R → R và g : R → R thỏa g(2020) > 0 và



f (x − g(y)) = f (−x + 2g(y)) + xg(y) − 6
g(y) = g(2f (x) − y)

a) Chứng minh rằng g là hàm hằng.

b) Chứng minh rằng đồ thị hàm số h(x) = f (x) − x nhận đường thẳng x = 1 là trục
đối xứng.

Lời giải.

a) Thay y = 2020 vào phương trình đầu, ta suy ra với mọi x ∈ R, tồn tại u, v ∈ R sao
cho x = f (u) − f (v). Từ phương trình thứ hai, ta được

g(y) = g(2f (v) − y) = g(2f (u) − 2f (v) + y),

z−y
suy ra với mọi y, z ∈ R, g(y) = g(z), vì ta có thể chọn f (u) − f (v) = x = .
2

3 3
b) Giả sử g(y) = α = g(2020) > 0, thay x = α, ta được α2 = 6 hay α = 2.
2 2
Vậy ta được f (x − 2) = f (−x + 4) = 2x − 6, thay x = x + 2, ta được

h(x) = f (x) − x = f (2 − x) − (2 − x) = h(2 − x),

nên h(x) nhận đường thẳng x = 1 là trục đối xứng.

Bài 4. Cho tam giác ABC nhọn nội tiếp (O) có H là trực tâm. AH, BH, CH lần lượt
cắt BC, CA, AB tại D, E, F và I, M, N là trung điểm BC, HB, HC. BH, CH cắt (O)
tại L, K (L 6= B, K 6= C); KL cắt M N tại G.

a) Trên EF lấy điểm T sao cho AT ⊥ IH. Chứng minh GT ⊥ OH.

b) DE, DF lần lượt cắt M N tại P, Q. Gọi S là giao điểm của BQ với CP . Chứng
minh rằng HS đi qua trung điểm EF.

Lời giải.

a) Gọi EF cắt BC tại T 0 , theo kết quả quen thuộc, AT 0 ⊥ IH nên T 0 ≡ T.

85
Lời giải đề thi chọn đội tuyển các năm

O
K F

M H N
G
T
B D C

Ta có ∠HM N = ∠HBC = ∠HKL nên tứ giác M N LK nội tiếp. Gọi (O0 ) là đường
tròn Euler của tam giác ABC, khi đó

PG\(O) = GK · GL = PG\(M N LK) = GM · GN = PG\(O0 )


PT \(O) = T B · T C = PT \(BC) = T E · T F = PT \(O0 ) .

Dẫn tới GT là trục đẳng phương của (O), (O0 ), suy ra GT ⊥ OO0 ≡ OH.

b) Ta có ∠QHD = ∠QDH (do M N là trung trực HD) mà DH là phân giác ∠EDF


nên ∠QDH = ∠HDE, suy ra QH k DE.

F
H

S O
M N
Q P

B U D C

86
Chuyên đề Toán học Số 12

HQ cắt BC tại U , thì Q là trung điểm HU , theo bổ đề hình thang thì BQ đi qua
trung điểm DE. Do DE, HC đối song trong ∠HBC nên BQ, BN đẳng giác trong
∠HBC. Vậy BQ là đường đối trung trong tam giác HBC, tương tự CP là đường
đối trung trong tam giác HBC nên HS là đường đối trung trong tam giác HBC.
Do EF, BC đối song trong ∠BHC nên HS đi qua trung điểm EF.

1
 
Bài 5. Cho số nguyên n > 1. Chứng minh rằng với mọi số thực a ∈ 0, và mọi đa
n
thức P (x) bậc 2n − 1 thỏa điều kiện P (0) = P (1) = 0, luôn tồn tại các số thực x1 , x2
thuộc [0, 1] sao cho P (x1 ) = P (x2 ) và x2 − x1 = a.

Lời giải. Xét đa thức Q(x) = P (x + a) − P (x), nếu tồn tại x ∈ [0, 1 − a] sao cho Q(x) = 0
thì bài toán kết thúc, giả sử Q(x) > 0 với mọi x ∈ [0, 1 − a].
Gọi N là số nguyên dương lớn nhất sao cho N a < 1, chú ý rằng N ≥ n. Dễ thấy
0 = P (0) < P (a) < · · · < P (N a) và 0 = P (1) > P (1 − a) > · · · > P (1 − N a).
Suy ra 0 < 1 − N a < a < 1 − (N − 1)a < 2a < · · · < 1 − a < N a < 1, tức ta đã chọn
ra được 2N số x1 < x2 < · · · < x2N mà P (xi )P (xi+1 ) < 0 nên có ít nhất 2N − 1 nghiệm
thuộc các đoạn (xi , xi+1 ), suy ra P có ít nhất 2N + 1 nghiệm, vô lí vì deg P < 2N + 1.

Bài 6. Tìm các số nguyên dương x, y thỏa phương trình


 
3x+1 3x+1
(x + 3)
2
(x + 3)
2
+ 1 + x2 + y = x2 y.

 
Lời giải. Phương trình tương đương y(x − 1) = (x + 3)
2 2 3x+1
(x + 3)
2 3x+1
+ 1 + x2 .
Đặt x2 − 1 = z, suy ra 43 (43 + 1) + 1 ≡ 0 (mod z), đặt 43 = m, ta suy ra
x+1 x+1 x+1

x+2
m2 + m + 1 ≡ 0 (mod z) ⇒ m3 ≡ 1 (mod z) ⇐⇒ 43 ≡1 (mod z)

Từ giả thiết, ta suy ra x > 1, nên z > 1, gọi p là ước nguyên tố bất kì của z. h = ordp 4,
suy ra h | 3x+2 . Nếu h = 3x+2 thì x2 − 1 = z ≥ 3x+2 . Dễ dàng kiểm tra bằng quy nạp
3x+2 > x2 nên ta suy ra h < 3x+2 hay h | 3x+1 , vậy m ≡ 1 (mod p) nên 3 ≡ 0 (mod p).
Nên x2 − 1 là lũy thừa của 3, suy ra x = 2. Thay vào phương trình ban đầu, ta được

4927 + 727 + 4
x = 2, y =
3
là cặp nghiệm cần tìm.

Nhận xét. Đây là một bài toán số học khá thú vị. Ý tưởng sử dụng cấp tuy không mới
nhưng đòi hỏi sự khéo léo nhất định. Một số bài toán tương tự.

Bài tập 1 (Đà Nẵng 2021). Tìm tất cả các số nguyên dương a, b và số nguyên tố p thoả
mãn:
(p − 1)b+1 + 1 = a(b + 1)p−1 .

87
Lời giải đề thi chọn đội tuyển các năm

Bài tập 2 (Canada MO, 2018). Cho trước k là một số nguyên dương chẵn. Ban đầu,
Sarah sẽ chọn một số nguyên dương N lớn hơn 1 và thực hiện quá trình sau đây : cứ
mỗi phút trôi qua, Sarah chọn một ước nguyên tố p của giá trị của N tại thời điểm đó,
1
rồi nhân giá trị của N tại thời điểm đó cho pk − để thu được giá trị tiếp theo của N .
p
Chứng minh rằng tồn tại vô số số nguyên dương chẵn k sao cho với mọi cách Sarah thực
hiện quá trình nêu trên, tồn tại một thời điểm nào đó mà N chia hết cho 2018.

Bài 7. Cho các số nguyên n > k > t > 0 và X = {1, 2, . . . , n}. Gọi F là họ gồm các
tập con k phần tử của X sao cho |F ∩ F 0 | ≥ t với mọi F, F 0 thuộc F. Giả sử không có
tập con t phần tử nào được chứa trong tất cả các tập F ∈ F. Chứng minh rằng

a) Tồn tại tập hợp B với |B| < 3k thỏa |B ∩ F | ≥ t + 1 với mọi F ∈ F.

b) |F| < C3k


t+1
· Cnk−t−1 .

Lời giải.

a) Nếu |F ∩ F 0 | ≥ t + 1 với mọi F, F 0 ∈ F thì ta chọn B ∈ F. Ngược lại, giả sử


F1 , F2 ∈ F sao cho |F1 ∩ F2 | = t. Xét F3 ∈ F sao cho F3 không chứa F1 ∩ F2 . Chọn
B = F1 ∪ F2 ∪ F3 . Khi đó |B| < 3k. Giả sử tồn tại tập F ∈ F sao cho |F ∩ B| = t.
Vì |F ∩ Fi | ≥ t với i = 1, 2, 3 nên F ∩ F1 = F ∩ F2 = F ∩ F3 = S, suy ra S = F1 ∩ F2
và S ⊂ F3 (vô lí).

b) Bổ sung vào tập B thêm các phần tử tùy ý để được tập C có 3k phần tử. Với mỗi
tập F ∈ F, tồn tại duy nhất cách phân hoạch F thành 2 tập con (F1 , F2 ) sao cho
F1 là tập con có t + 1 phần tử của C và F2 là tập con có k − t − 1 phần tử của X,
hơn nữa với các tập F khác nhau thì cặp (F1 , F2 ) tương ứng cũng khác nhau nên
t+1
|F| < C3k .Cnk−t−1

Bài 8. Tam giác ABC nội tiếp đường tròn (O) với B, C cố định và A thay đổi trên
cung lớn BC. Dựng hình bình hành ABDC, AD cắt (BCD) tại K 6= D.

a) Gọi R1 , R2 là bán kính của (KAB), (KAC); chứng minh R1 R2 không đổi.

b) (T ) là đường tròn qua K tiếp xúc với BD tại B và (T1 ) là đường tròn qua K tiếp
xúc với CD tại C. (T ) và (T1 ) cắt nhau tại L 6= K. Chứng minh AL luôn đi qua
một điểm cố định.

Lời giải.

88
Chuyên đề Toán học Số 12

B M C
L

a) Gọi M là trung điểm chung của AD, BC. Ta có

M B 2 = M B · M C = PM \(DBC) = M D · M K = M A · M K.

Suy ra 4M BK v 4M AB, nên ∠M KB = ∠ABC, tương tự, ∠M KC = ∠ACB.


Gọi R là bán kính (O), theo định lý hàm số sin, ta có
AB AC AB · AC
2R1 · 2R2 = · = = 4R2
sin ∠AKB sin ∠AKC sin ∠ABC · sin ∠ACB
nên R1 R2 = R2 không đổi.

b) Ta có ∠BLK + ∠CLK = (180o − ∠DBK) + (180o − ∠DCK) = 180o nên L ∈ BC.


Chú ý rằng ∠BKL = ∠DBL = ∠DBC = ∠DKC nên KL, KD đẳng giác trong
∠BKC, điều này dẫn tới
LB KB 2 AB 2
= = ,
LC KC 2 AC 2
suy ra AL, AK đẳng giác trong ∠BAC. Vậy AL là đường đối trung trong tam giác
ABC nên AL đi qua S là giao tiếp tuyến tại B, C của (O).

89
Lời giải đề thi chọn đội tuyển các năm

2.3. Năm 2021

Bài 1. Tìm hàm số f : R → R thỏa

f (xf (y) + f (x)) = f (x) + xy + x + 1, ∀x, y ∈ R.

Lời giải. Trước hết, ta sẽ chứng minh rằng f là đơn ánh. Thật vậy, xét a, b ∈ R sao cho
f (a) = f (b). Thay lần lượt y = a và y = b vào điều kiện đề bài, ta được ax = bx với mọi
x ∈ R. Với x = 1, ta được a = b. Vậy f là đơn ánh.
Tiếp theo, thay y = −x−1x
, ta có

−x − 1 −x − 1
     
f xf + f (x) = f (x) ⇒ xf + f (x) = x. (1)
x x
Trong (1) thay x thành −x−1
x
ta được

−x − 1 −1 −x − 1 −x − 1
   
f +f = , ∀x ∈
/ {0, −1}. (2)
x x+1 x x

Trong (1) thay x thành −1


x+1
ta được

−1 −1 −1
 
f (x) + f = , ∀x ∈
/ {0, −1}. (3)
x+1 x+1 x+1
Giải hệ phương trình (1), (2), (3) ta được f (x) = x + 1 với mọi ∀x ∈
/ {0, −1}.
Thay x = 2, y = −1 vào điều kiện đề bài, ta được

f (2f (−1) + 3) = f (2) + 1 = 4 = f (3) ⇒ 2f (−1) + 3 = 3 ⇒ f (−1) = 0.

Thay x = −1, y = 0 vào điều kiện đề bài, ta được f (−f (0)) = f (−1), suy ra −f (0) = −1
và f (0) = 1, do đó f (x) = x + 1 với mọi x ∈ R. Thử lại, ta thấy hàm này thoả đề.
Vậy f (x) = x + 1 là hàm cần tìm.

q
Bài 2. Cho dãy số (un ) thoả u1 = 2, u2 = 1 và un+1 = un unn−1 với mọi n ≥ 2. Xét
dãy số (vn ) được xác định bởi vn = u1 + u2 + · · · + un với mọi n ≥ 1. Chứng minh dãy
(vn ) hội tụ.

Lời giải. Từ công thức truy hồi, dễ dàng suy ra được u3 = 1 và un > 0 với mọi n ∈ N∗ .
Tiếp theo, bằng phương pháp quy nạp toán học, ta sẽ chứng minh rằng un ≤ 1 với mọi
n ≥ 2. Thật vậy, bằng kiểm tra trực tiếp, dễ thấy mệnh đề đúng tại n = 2, n = 3. Giả sử
mệnh đề đã đúng tới n ≥ 3, ta có
s s
un un−1 1·1 1
r
un+1 = ≤ = < 1.
n 3 3
Từ đây, ta suy ra un ≤ 1 với mọi n ≥ 2.

90
Chuyên đề Toán học Số 12

Tiếp theo, cũng bằng quy nạp, ta sẽ chứng minh un ≤ un−1 ≤ nun với mọi n ≥ 2. Thật
vậy, dễ q
thấy mệnh đề đúng với n = 2 và n = 3. Giả sử mệnh đề đã đúng tới n, ta có
un+1 ≤ nunn un = un (do un > 0). Ta chứng minh un+1 ≥ n+1
un
. Điều này đồng nghĩa với

un un−1 un n2
r
≥ ⇔ un−1 ≥ un .
n n+1 n+1
n2
(Hiển nhiên vì n+1
≥ 1 với mọi n ≥ 2 và un−1 ≥ un .)
q
Vì vậy ta có dãy (un ) không tăng. Với mọi n ≥ 4 thì un+1 ≤ un−1 un−1
4
= un−1
2
. Vì vậy
với n đủ lớn thì
u3 + u4 u3 + u4 u3 + u4
vn ≤ v2n+2 ≤ 3 + (u3 + u4 ) + + + ... < 3 + 2(u3 + u4 )
2 4 2n
nên dãy (vn ) bị chặn trên. Hơn nữa, dễ thấy (vn ) là dãy tăng, suy ra dãy này hội tụ. Ta
có điều cần chứng minh.

Bài 3. Cho p là số nguyên tố lẻ và n là một số nguyên dương lớn hơn p. Gọi A là tập
hợp các đa thức P (x) = xn + an−1 xn−1 + · · · + a1 x + a0 thoả mãn ai ∈ {1, 2, . . . , n!} với
mọi i = 0, 1, 2, . . . , n − 1, đồng thời P (m) chia hết cho p với mọi số nguyên dương m.
j k
a) Chứng minh tổng a1 + ap + a2p−1 + · · · + a1+k(p−1) , trong đó k = n−1
p−1
, chia hết
cho p (xem an = 1).

b) Tính số phần tử của A theo n và p.

(Với mỗi số thực x, bxc là số nguyên lớn nhất không vượt quá x.)

Lời giải.

a) Với mỗi i ∈ {1, 2, . . . , p − 1}, đặt Si = aj .


P
1≤j≤n,j≡i (mod p−1)
Khi này,
n−1
$ %
S1 = a1 + ap + · · · + a1+k(p−1) , k = .
p−1
Áp dụng định lý Fermat nhỏ, ta có

P (m) ≡ S1 m + S2 m2 + · · · + Sp−1 mp−1 (mod p)

với mọi số nguyên dương m. Đặt

Q(x) = S1 x + S2 x2 + · · · + Sp−1 xp−1 .

Theo đề bài, với mọi số nguyên dương m, do P (m) chia hết cho p nên Q(m) cũng
chia hết cho p. Vì thế, Q(1), Q(2), . . . , Q(p) đều chia hết cho p. Do Q(x) là đa thức
có bậc không vượt quá p − 1 nên áp dụng công thức nội suy Lagrange, ta có
(x − j)
Q
p
1≤j≤p,j6=i
Q(x) =
X
Q(i) · .
(i − j)
Q
i=1
1≤j≤p,j6=i

91
Lời giải đề thi chọn đội tuyển các năm

Do các hệ số của (x − j) đều nguyên, Q(i) chia hết cho p còn (i − j)


Q Q
1≤j≤p,j6=i 1≤j≤p,j6=i
không chia hết cho p nên các hệ số của Q(x) đều chia hết cho p. Từ đây, ta suy ra
S1 , S2 , . . . , Sp−1 chia hết cho p, và thu được điều cần chứng minh.

b) Từ câu trên, ta suy ra để P (m) thuộc A, S1 , S2 , . . . , Sp−1 , a0 phải chia hết cho p. Dễ
thấy đây cũng là điều kiện đủ. Ta sẽ chọn các hệ số từ phải sang. Cụ thể:

• Đầu tiên, chọn an−1 , an−2 , . . . , ap là các số tuỳ ý trong {1, 2, . . . , n!}: có (n!)n−p
cách chọn.

• Với mỗi i = 1, p − 1, sau khi đã chọn các số từ ap đến an−1 , có đúng n!


p
cách chọn
ai để Si chia hết cho p.

• Có n!
p
cách chọn a0 để a0 chia hết cho p.

Từ đây, áp dụng quy tắc nhân, ta suy ra


!p−1
n! n! (n!)n
|A| = (n!)n−p · · = p .
p p p

Nhận xét. Đây là một bài toán số học về đa thức. Ý a không quá khó và cũng không
quá lạ lẫm. Ý b có vẻ khó nhưng lại được giải quyết dễ dàng nhờ ý a và một vài phép
đếm đơn giản. Một số bài toán tương tự.

Bài tập 1. Cho đa thức P (x) bậc n, có các hệ số đều nguyên và là đa thức monic (có hệ
số bậc cao nhất bằng 1). Giả sử tồn tại số nguyên a sao cho P (x) chia hết cho a với mọi
số nguyên x. Chứng minh rằng a | n!.

Bài tập 2. Cho số nguyên tố lẻ p. Tìm tất cả các số nguyên dương n sao cho tồn tại một
đa thức P (x) bậc n thoả mãn P (x) chia hết cho p với mọi số nguyên x, đồng thời các hệ
số của P là hoán vị của n + 1 số nguyên dương đầu tiên.

Bài 4. Cho tam giác ABC có (I) là đường tròn nội tiếp. Một đường thẳng qua A cắt
(I) tại M, N . Gọi T là giao điểm của các tiếp tuyến với (I) tại M, N .

a) Chúng minh rằng nếu AT k BC thì M N đi qua trung diểm K của BC.

b) Gọi D là tiếp điểm của (I) với AB và E là giao điểm của DM và AC. Trên EN
lấy điểm F sao cho T F vuông góc AI. Chứng minh rằng khi đường thẳng AM N
thay đổi, giao điểm P của M F và DN thuộc một đường thẳng cố định.

Lời giải.

92
Chuyên đề Toán học Số 12

E
T
M

Y
F
U

B C
K

a) Gọi X, Y lần lượt là tiếp điểm của (I) với AB, AC, U là giao điểm M N, XY . Ta lại
có (M N XY ) = −1 từ giả thiết, suy ra T ∈ XY và

M (T U XY ) = (M N XY ) = −1 ⇒ A(T U BC) = −1.

Nếu AT k BC thì từ trên ta có AU là trung tuyến của tam giác ABC hay M N di
qua trung điểm K của BC. Ta có điều cần chứng minh.

b) Do D là tiếp điểm của (I) với AB, ta suy ra được D ≡ X.


Ta có T ∈ XY (chứng minh trên) mà XY vuông góc AI nên đường thẳng qua T
vuông AI là XY , theo cách dựng điểm F thì F chính là giao diểm của XY và EN .
Gọi G, H lần lượt là giao điểm khác N, M của N F và M F .
Lúc này
(GHXY ) = F (GHXY ) = (N M Y X) = −1.
nên A, H, G thẳng hàng. Ta lại có:



DN cắt M F tại P


DM cắt N G tại E

DD (tiếp tuyến tại D) cắt GH tại A.

Áp dụng định lý Pascal cho bộ sáu điểm (D, N, G, H, M, D), ta được P, E, A thẳng
hàng hay P nằm trên đường thẳng AC cố định.

Nhận xét. Đây là một bài toán hình học không quá khó. Tuy nhiên, các bạn học sinh
dễ mất điểm nếu không đọc kĩ đề (nhầm lẫn D là tiếp điểm trên BC).

93
Lời giải đề thi chọn đội tuyển các năm

Bài 5. Cho n số thực x1 , x2 , . . . , xn thoả hiệu giữa số lớn nhất và số nhỏ nhất trong
chúng là 1. Ta xây dựng
x1 + x 2 x1 + x2 + · · · + xn
y1 = x1 , y2 = , . . . , yn = .
2 n
Đặt T = max yi − min yj . Tìm giá trị lớn nhất của T .

Lời giải. Trước hết, ta có

(x1 − C) + · · · + (xi − C) x1 + x2 + · · · + xi Ci
= − = yi − C.
i i i
Suy ra khi trừ x1 , x2 , . . . , xn cho cùng một số C thì y1 , y2 , . . . , yn cũng bị trừ đi C đơn vị,
và giá trị của biểu thức T không thay đổi. Do đó, không mất tính tổng quát, ta có thể
giả sử max xi = 1, min xi = 0. Bây giờ, với 1 ≤ i < j ≤ n, ta có

1 1 1 n−1
!
i
yi − yj = − (x1 + · · · + xi ) − (xi+1 + · · · + xj ) ≤ 1 − ≤ .
i j j j n

Tương tự, với 1 ≤ j < i ≤ n, ta có

1 1 1 n−1
!
j
yi − yj = (xj+1 + · · · + xi ) − − (x1 + · · · + xj ) ≤ 1 − ≤ .
i j i i n

Từ đây, ta suy ra yi − yj ≤ n−1


n
với mọi i, j ≤ n, i 6= j và T ≤ n−1
n
. Đẳng thức xảy ra,
chẳng hạn, khi x1 = 0, x2 = x3 = · · · = xn = 1.
Vậy giá trị lớn nhất của T là n−1
n
.

Nhận xét. Đây là một bài bất đẳng thức nhiều biến hay và gọn nhẹ. Ý tưởng chính chỉ
là tính các hệ số của x1 , x2 , . . . , xn trong yi − yj rồi dồn các biến có hệ số dương về 1, các
biến có hệ số âm về 0. Một số bài toán tương tự.

Bài tập 1 (Brazil 2018). Esmeralda viết 2n số thực x1 , x2 , . . . , x2n , tất cả đều thuộc
khoảng [0, 1], xung quanh một vòng tròn theo chiều kim đồng hồ và tính tích tất cả các
cặp số kề nhau trên vòng tròn đó. Như vậy, cô ấy thu được các tích p1 = x1 x2 , p2 = x2 x3 ,
. . . , p2n = x2n x1 . Cô ấy tính tổng tất cả các tích với chỉ số chẵn rồi trừ chúng cho tổng
tất cả các tích với chỉ số lẻ. Hỏi số lớn nhất mà Esmeralda có thể thu được là bao nhiêu?

Bài tập 2. Với dãy số x = (xi )ni=1 ta có thể định nghĩa dãy Cesaro C(x) = (yi )ni=1 của
nó bởi công thức
x1 + x2 + · · · + xi
yi = , i = 1, 2, . . . , n.
i
Đại lượng d(x) = n−1
i=1 |xi − xi+1 | được gọi là độ lệch tuyệt đối của dãy số. Chứng minh
P

rằng với mọi dãy số x = (xi )ni=1 , ta có


n−1
d (C(x)) ≤ d(x).
n

94
Chuyên đề Toán học Số 12

Bài 6. Cho tập X = {1, 2, . . . , 20}. Tập con A của X được gọi là tập tránh 2 nếu với
mọi x, y ∈ A thì |x − y| khác 2. Tìm số tập con tránh 2 của X có 5 phần tử.

Lời giải. Phân hoạch X ra làm 2 tập con X1 = {1, 3, . . . , 19}, X2 = {2, 4, . . . , 20}. Dễ
thấy 2 số có hiệu bằng 2 khi và chỉ khi chúng là 2 số lẻ liên tiếp hoặc 2 số chẵn liên tiếp.
Với n ≤ 5 cho trước, ta sẽ đếm số cách chọn n số từ X1 sao cho trong các số được chọn,
không có 2 số lẻ liên tiếp. Thật vậy, gọi n số được chọn là

2a1 − 1, 2a2 − 1, . . . , 2an − 1 (a1 < a2 < · · · < an ).

Khi này, do không có 2 số lẻ liên tiếp cùng được chọn, ta phải có

a1 < a2 − 1 < · · · < an − (n − 1) ≤ 11 − n.

Có C11−n
n
cách chọn n số a1 , a2 − 1, . . . , an − (n − 1) thoả mãn điều kiện trên, suy ra có
C11−n cách chọn n số từ X1 . Tương tự, ta cũng suy ra có C11−n
n n
cách chọn n số từ X2 sao
cho không có 2 số chẵn liên tiếp cùng được chọn. Do đó, với n ≤ 5 cho trước, số tập con
tránh 2 của X có 5 phần tử, trong đó có đúng n phần tử thuộc X1 là C11−n n 5−n
· C6+n . Cho
n chạy từ 0 đến 5, ta suy ra số tập con tránh 2 của X có 5 phần tử là
5
n 5−n
= 4744.
X
C11−n · C6+n
n=0

Nhận xét. Ở đây, ý tưởng phân hoạch ra làm 2 tập hợp theo tính chẵn lẻ đóng vai trò
quyết định, phần còn lại chỉ là xử lý kỹ thuật.

Bài tập 1 (VMO 2006, Bảng B). Một đơn vị kiểm lâm muốn lập lịch đi tuần tra rừng
cho cả năm 2006 với các yêu cầu sau:

i) Số ngày đi tuần tra trong năm nhiều hơn một nửa tổng số ngày của năm ;

ii) Không có hai ngày đi tuần tra nào cách nhau đúng một tuần lễ.

Hỏi có thể lập được tất cả bao nhiêu lịch đi tuần tra rừng như vậy?

Bài 7. Cho tam giác ABC và điểm D trên cạnh BC. Các đường tròn (ABD), (ACD)
lần lượt cắt AC, AB tại E, F . Gọi I là tâm đường tròn AEF .

a) Chứng minh rằng ID vuông góc BC.

b) Gọi H là giao điểm của ID với EF và K là điểm thoả mãn ∠HBK = ∠HCK =
900 . Các đường tròn (IBK), (ICK) lần lượt cắt IC, IB tại M , N . Chứng minh
rằng tâm J của đường tròn (IM N ) thuộc trung trực BC.

Lời giải.

95
Lời giải đề thi chọn đội tuyển các năm

O
F H
E

B C
D

a) Có
BF · BA = BD · BC = IB 2 − IA2 .
Tương tự thì
CE · CA = CD · CB = IC 2 − IA2 .
Lấy hai vế trừ nhau ta thu được

BD · BC − CD · CB = DB 2 − DC 2 = IB 2 − IC 2 .

Vì vậy theo bổ đề 4 điểm thì ID vuông góc BC.

b) Cho BE cắt CF tại X thì

∠BAC = ∠DAB + ∠DAC = ∠DCF + ∠DBE = 1800 − ∠F XE.

Vì vậy tứ giác AF XE nội tiếp (I). Gọi H 0 là giao điểm của F E và AX. Theo định lý
Brocard, H 0 là trực tâm tam giác IBC, suy ra IH 0 vuông góc với BC và H 0 ≡ H. Ta
có HB ⊥ KB, HB ⊥ IC nên KB k IC. Tương tự, ta có KC k IB, suy ra IBKC
là hình bình hành. Ta giải quyết bổ đề sau, cũng là tương đương với bài toán trên.

96
Chuyên đề Toán học Số 12

Bổ đề. Cho hình bình hành ABCD. (ABC), (ADC) lần lượt cắt AD, AB tại M ,
N . Khi này, tâm J của đường tròn (AM N ) nằm trên trung trực BD.

Chứng minh.

O
B D

N C


DA · DM = PD/(ABC) = PD/(J) = JD2 − JA2 .

Tương tự
BA · BN = PB/(ADC) = PB/(J) = JB 2 − JA2 .

Có PD/(ABC) = PB/(ADC) là do đối xứng qua trung điểm của hai đường chéo AC, BD
biến B ↔ D, (ABC) ↔ (ADC). Vì vậy ta có JD2 = JB 2 , suy ra JD = JB và J
nằm trên trung trực BD. Bài toán được giải quyết. 

Nhận xét. Ở câu a (đề Toán chuyên PTNK năm 2017) thì có nhiều cách xử lý như biến
đổi góc và tam giác đồng dạng hoặc là dùng phương tích như trên. Tuy nhiên ở câu b)
nếu không nhận ra H là trực tâm IBC thì bài toán không thể được giải quyết. Chính vì
vậy, cách tiếp cận bài này bằng định lý Brocard ngay từ đầu sẽ giúp chúng ta giải quyết
được cả hai ý một cách dễ dàng. Việc tách mô hình là để nhận ra bài toán này được ghép
từ hai bài toán là định lý Brocard và bổ đề nêu trên.

Bài 8. Cho p là số nguyên tố. Với mọi số nguyên a, đặt

q := 1 + a + a2 + . . . + ap−1 .

Chứng minh (1 − a) (1 − a2 ) . . . (1 − ap−1 ) − p chia hết cho q.

Lời giải. Để giải quyết bài toán trên ta đưa về xét hai đa thức sau

f (x) = (1 − x)(1 − x2 ) . . . (1 − xp−1 ) − p, g(x) = 1 + x + x2 + . . . + xp−1 .

97
Lời giải đề thi chọn đội tuyển các năm


Đặt α = e p ta viết lại đa thức g(x) như sau
 
g(x) = (x − α)(x − α2 ) . . . x − αp−1

có tất cả các nghiệm phức là αn với n thỏa mãn 1 ≤ n ≤ p − 1.


Ta chứng minh tất cả các nghiệm trên đều là nghiệm của f (x).
Đặt X = αk thì với 1 ≤ j ≤ p − 1 ta được X j = αkj và kj không chia hết cho p.
Đồng thời nhận thấy rằng X i 6= X j với i, j thỏa mãn 1 ≤ i < j ≤ p − 1 nên

(X, X 2 , X 3 , . . . , X p−1 ) là hoán vị của αk , k ∈ {1, 2, . . . , p − 1}).

Suy ra  
f (X) = (1 − α)(1 − α2 ) . . . 1 − αp−1 − p = g(1) − p = 0.

Do đó X là một nghiệm của f (x), cho k chạy trong tập {1, . . . , p − 1} ta có tất cả nghiệm
của đa thức g(x) đều là nghiệm của đa thức f (x).
Từ đây ta được đa thức f (x) chia hết cho đa thức g(x).
Vì thế nên tồn tại đa thức h(x) ∈ R[x] sao cho f (x) = g(x)h(x).
Ta biểu diễn hai đa thức f (x) và h(x) như sau

f (x) = a0 + a1 x1 + . . . + adeg f xdeg f , h(x) = b0 + b1 x1 + . . . + bdeg h xdeg h

trong đó ai là các số nguyên với i ∈ {0, . . . , deg f }.


Đồng nhất thức 2 vế và để ý g(x) = 1 + x + x2 + . . . + xp−1 ta được

ai = bi + bi−1 + bi−2 + · · · + bi−(p−1) ∈ Z với bk = 0 nếu k < 0.

Từ đây bằng phép quy nạp lùi ta chứng minh được bi ∈ Z với mọi i ∈ {0, . . . , deg h} hay
h(x) ∈ Z[x] nên h(a) ∈ Z với mọi số nguyên a.
Lúc này f (a) = g(a)h(a) nên bài toán được chứng minh.

Nhận xét. Đây có lẽ là câu khó nhất của ngày 2 . Khi nhìn thấy biểu thức q, ta thường
sẽ nghĩ đến đa thức chia đường tròn, một hướng suy nghĩ khác là dùng bổ đề "Tất cả các
ước nguyên tố của q hoặc là p, hoặc chia p dư 1", nhưng phát sinh vấn đề ở việc xét đồng
dư do có sự xuất hiện của −p. Và hướng suy nghĩ phổ biến còn lại là nhìn bài toán theo
góc nhìn đại số như lời giải trên, cụ thể là xem hai biểu thức như hai đa thức biến a.

Bài tập 1. Với các số nguyên dương a, k, n thoả mãn a > 1 và k ≤ n, định nghĩa:

(an − 1)(an−1 − 1) . . . (an−k+1 − 1)


B(n, k, a) = .
(ak − 1)(ak−1 − 1) . . . (a − 1)

a) Chứng minh rằng B(n, k, a) là số nguyên.

b) Hỏi có tồn tại hay không các số a, k, n, m (k < n < m, a > 1) sao cho B(m, 1, a) là
ước của B(n, k, a)?

98

You might also like